Trauma and Critical Care Flashcards

1
Q

Cricothyroidotomy

A. Should not be performed in children younger than 12 years

B. Should only be performed in patients who are not good
candidates for a tracheostomy

C. Requires the use of an endotracheal tube smaller than 4 mm in diameter

D. Is preferable to the use of percutaneous transtracheal ventilation

A

Answer: A

Patients in whom attempts at intubation have failed or are precluded from intubation due to extensive facial injuries require a surgical airway.

Cricothyroidotomy (Fig. 7-1) and percutaneous transtracheal ventilation arc preferred over tracheostomy in most emergency situations because of their simplicity and safety.

One disadvantage of cricothyroidotomy is the inability to place a tube greater than 6 mm in diameter due to the limited aperture of the cricothyroid space.

Cricothyroidotomy is also relatively contraindicated in patients younger than 12 years because of the
risk of damage to the cricoid cartilage and the subsequent risk of
subglottic stenosis.

How well did you know this?
1
Not at all
2
3
4
5
Perfectly
2
Q

Which of the following is NOT a sign of tension
pneumothorax?

A. Tracheal deviation

B. Decreased breath sounds

C. Respiratory distress with hypertension

D. Distended neck veins

A

Answer: C

The diagnosis of tension pneumothorax is presumed in any patient manifesting respiratory distress and hypotension in combination with any of the following physical signs: tracheal deviation away from the affected side, lack of or decreased breath sounds on the affected side, and subcutaneous emphysema on the affected side.

Patients may have distended neck veins due to impedance of venous return, but the neck veins may be flat due to concurrent systemic hypovolemia.

Tension pneumothorax and simple pneumothorax have similar signs, symptoms, and examination findings, but hypotension qualifies the pneumothorax as a tension pneumothorax.

How well did you know this?
1
Not at all
2
3
4
5
Perfectly
3
Q

Which of the following is a cause of cardiogenic shock in a trauma patient?

A. Hemothorax

B. Penetrating injury to the aorta

C. Air embolism

D. Iatrogenic increased afterload due to pressors

A

Answer: C

In trauma patients the differential diagnosis of cardiogenic shock consists of a short list:

(1) tension pneumothorax,
(2) pericardial tamponade,
(3) myocardial contusion or infarction, and
(4) air embolism.

Tension pneumothorax is the most frequent cause of cardiac failure.

Traumatic pericardial tamponade is most often
associated with penetrating injury to the heart.

As blood leaks out of the injured heart, it accumulates in the pericardial sac.

Because the pericardium is not acutely distensible,
the pressure in the pericardial sac rises to match that of the injured chamber.

Since this pressure is usually greater than that of the right atrium, right atrial filling is impaired and right ventricular preload is reduced.

This leads to decreased right ventricular output and increased central venous pressure (CVP).

Increased intrapericardial pressure also impedes
myocardial blood flow, which leads to subendocardial ischemia and a further reduction in cardiac output.

This vicious cycle may progress insidiously with injury of the vena cava or atria, or precipitously with injury of either ventricle.

With acute tamponade, as little as 100 mL of blood within the pericardial sac can produce life-threatening hemodynamic compromise.

Patients usually present with a penetrating injury in
proximity to the heart, and they are hypotensive and have distended neck veins or an elevated CVP.

The classic findings of Becks triad (hypotension, distended neck, and muffled heart sounds) and pulsus paradoxus are not reliable indicators of acute tamponade.

Ultrasonography (US) in the emergency department (ED) using a subxiphoid or parasternal view is extremely helpful if the findings are clearly positive (Fig. 7-2);
however, equivocal findings are common.

Early in the course of tamponade, blood pressure (BP) and cardiac output will transiently improve with fluid administration.

This may lead the surgeon to question the diagnosis or be lulled into a false sense of security.

How well did you know this?
1
Not at all
2
3
4
5
Perfectly
4
Q

A trauma patient arrives following a stab wound to the left chest with systolic blood pressure (SBP) 85 mmHg, which improves slightly with intravenous (IV) fluid resuscitation.

Chest X-ray demonstrates clear lung fields. What is the most appropriate next step?

A. Computed tomography (CT) scan of the chest

B. Pelvic X-ray

C. Focused abdominal sonography for trauma (FAST) examination

D. Tube thoracostomy of the left chest

A

Answer: C

During the circulation section of the primary survey, four life-threatening injuries must be identified promptly:

(1) massive hemothorax,
(2) cardiac tamponade,
(3) massive hemoperitoneum, and
(4) mechanically unstable pelvic fractures with bleeding.

In this patient hemothorax is unlikely given normal chest X-ray; thus, hemoperitoneum and cardiac tamponade should be suspected.

Cardiac tamponade occurs most commonly
after penetrating thoracic wounds, although occasionally blunt rupture of the heart, particularly the atrial appendage, is seen.

Acutely, <100 mL of pericardial blood may cause pericardial tamponade. The classic Beck’s triad—dilated neck veins, muffled heart tones, and a decline in arterial pressure—is usually not appreciated in the trauma bay because of the noisy environment and associated hypovolemia.

Diagnosis is best achieved by bedside ultrasound of the pericardium, which is one of the four views of the FAST examination.

(See Schwartz 10th cd.,p. 166.)

How well did you know this?
1
Not at all
2
3
4
5
Perfectly
5
Q

Primary repair of the trachea should be carried out with

A. Wire suture

B. Absorbable monofilament suture

C. Nonabsorbable monofilament suture

D. Absorbable braided suture

A

Answer: B

Injuries of the trachea are repaired with a running 3-0 absorbable monofilament suture.

Tracheostomy is not required in most patients.

Esophageal injuries are repaired in a similar fashion. If an esophageal wound is large or if tissue is missing, a sternocleidomastoid muscle pedicle flap is warranted, and a closed suction drain is a reasonable precaution.

The drain should be near but not in contact with the esophageal or any other suture line. It can be removed in 7 to 10 days if the suture line remains secure. Care must be taken when exploring the trachea and esophagus to avoid iatrogenic injury to the recurrent laryngeal nerve.

How well did you know this?
1
Not at all
2
3
4
5
Perfectly
6
Q

In which patient is emergency department thoracotomy
contraindicated?

A. Motor vehicle accident victim, cardiac tamponade seen on ultrasound, SBP decreasing to 50 mm Hg.

B. Motor vehicle accident victim, became asystolic during transport with 5 minutes of cardiopulmonary resuscitation (CPR) with no signs of life.

C. Patient with chest stab wound, SBP decreasing to 50 mm Hg.

D. Patient with chest stab wound, became asystolic during transport with 20 minutes of CPR with no signs of life.

A

Answer: D

The utility of resuscitative thoracotomy (RT) has been debated for decades.

Current indications arc based on 30 years of prospective data, supported by a recent multicenter prospective
study.

RT is associated with the highest survival rate after isolated cardiac injury; 35% of patients presenting in shock and 20% without vital signs (ie, no pulse or obtainable BP) are salvaged after isolated penetrating injury to the heart.

For all penetrating wounds, survival rate is 15%.

Conversely, patient outcome is poor when RT is done for blunt trauma, with 2% survival among patients in shock and <1% survival among those with no vital signs.

Thus, patients undergoing cardiopulmonary resuscitation (CPR) upon arrival to the ED should undergo RT selectively based on injury and transport time.

How well did you know this?
1
Not at all
2
3
4
5
Perfectly
7
Q

A patient with spontaneous eye opening, who is confused and localizes pain has a Glasgow Coma Score (GCS) of

A. 9

B. 11

C. 13

D. 15

A

Answer: C

The Glasgow Coma Score (GCS) should be determined for all injured patients.

It is calculated by adding the scores of the best motor response, best verbal response, and eye opening. Scores range from 3 (the lowest) to 15 (normal). Scores of 13 to 15 indicate mild head injury, 9 to 12 moderate injury, and less than 9 severe injury.

The GCS is useful for both triage and prognosis.

How well did you know this?
1
Not at all
2
3
4
5
Perfectly
8
Q

Neck injuries

A. Less than 15% penetrating injuries require neck exploration, a majority can be managed conservatively.

B. Divided into three zones, with zone I above the angle of the mandible, zone II between the thoracic outlet and angle of mandible, and zone III inferior to the clavicles.

C. All patients with neck injury should receive computed tomography angiogram (CTA) of the neck.

D. Patients with dysphagia, hoarseness, hematoma,
venous bleeding, hemoptysis, or subcutaneous emphy¬
sema should undergo neck exploration.

A

Answer: A

Zone I is inferior to the clavicles encompassing the thoracic outlet structures, zone II is between the thoracic outlet and the angle of the mandible, and zone III is above the angle of the mandible. Patients with symptomatic zone I and III injuries should ideally undergo diagnostic imaging before operation if they remain hemodynamically stable.

Specific symptoms which indicate further imaging include dysphagia, hoarseness, hematoma, venous bleeding, minor hemoptysis, and subcutaneous emphysema.

Symptomatic patients should undergo CTA with further evaluation or operation based upon the imaging findings; less than 15%of penetrating cervical trauma requires neck exploration.

Asymptomatic patients are typically observed for 6 to 12 hours. The one caveat is asymptomatic patients with a transcervical gunshot wound; these patients should undergo CTA to determine the track of the bullet.

CTA of the neck and chest determines trajectory of the injury tract; further studies are performed based on proximity to major structures.

Angiographic diagnosis, particularly of zone III injuries, can then be managed by selective angioembolization.

How well did you know this?
1
Not at all
2
3
4
5
Perfectly
9
Q

Appropriate surgical management of a through-and-through gunshot wound to the lung with minimal bleeding and some air leak is

A. Chest tube only

B. Oversewing entrance and exit wounds to decrease the air leak

C. Pulmonary tractotomy with a stapler and oversewing of vessels or bronchi

D. Wedge resection of the injured lung

A

Answer: C

Pulmonary injuries requiring operative intervention usually result from penetrating injury.

Formerly the entrance and exit wounds were oversewn to control hemorrhage.

This set the stage for air embolism, which occasionally caused sudden death in the operating room or in the immediate postoperative period.

A recent development, pulmonary tractotomy, has been employed to reduce this problem as well as the need for pulmonary resection.

Linear stapling devices arc inserted directly into the injury tract and positioned to cause the least degree of devascularization.

Two staple lines are created and the lung is divided between. This allows direct access to the bleeding vessels and leaking bronchi. No effort is made to close the defect. Lobectomy or pneumonectomy is rarely necessary.

Lobectomy is only indicated for a completely devascularized or destroyed lobe.

Parenchymal injuries severe enough to require pneumonectomy are rarely survivable, and major pulmonary hilar injuries necessitating pneumonectomy are usually lethal in the field.

How well did you know this?
1
Not at all
2
3
4
5
Perfectly
10
Q

What is true regarding the evaluation of blunt abdominal trauma?

A. Patients with abdominal wall rigidity and negative abdominal CT should undergo diagnostic peritoneal
lavage (DPL) to rule out small bowel injury.

B. If FAST examination is negative in a hemodynamically unstable patient then DPL is indicated to rule out abdominal bleeding.

C. FAST examination cannot detect intraperitoneal fluid if the total volume is <1000 mL

D. Bowel injury can be ruled out in hemodynamically
stable patients with abdominal CT scanning.

A

Answer: B

The presence of abdominal rigidity and hemodynamic compromise is an undisputed indication for prompt surgical exploration.

Blunt abdominal trauma is evaluated initially by
FAST examination in most major trauma centers, and this has largely supplanted diagnostic peritoneal lavage (DPL).

FAST is not 100% sensitive, however, so diagnostic peritoneal aspiration is warranted in hemodynamically unstable patients without a defined source of blood loss to rule out abdominal hemorrhage. This method is exquisitely sensitive for detecting intraperitoneal fluid of >250 mL Patients with fluid on FAST examination, considered a ‘’positive FAST” who do not have immediate indications for laparotomy and are hemodynamically stable undergo CT scanning to quantify their injuries.

CT also is indicated for hemodynamically stable patients for whom the physical examination is unreliable. Despite the increasing diagnostic accuracy of multidetector CT scanners, identification of intestinal injuries remains a limitation.

Patients with free intraabdominal fluid without solid organ injury are closely monitored for evolving signs of peritonitis; if patients have a significant closed head injury or cannot be serially examined, DPL should be performed to exclude bowel injury.

After placement of the catheter, a 10-mL syringe is connected and the abdominal contents aspirated (termed a diagnostic peritoneal aspiration). The aspirate is considered to show positive findings if >10 mL of blood is aspirated.

If <10 mL is withdrawn, a liter of normal saline is instilled, the effluent is withdrawn via siphoning and sent to the laboratory for red blood cell (RBC) count, white blood cell (WBC) count, and determination of amylase, bilirubin, and alkaline phosphatase levels.

TABLE 7-2
Criteria for “positive” finding on diagnostic peritoneal lavage

Abdominal Trauma
Red blood cell count >100,000.'mL
White blood cell count >500/mL
Amylase level >19 OL
Alkaline phosphatase level >2 IU-'L
Bilirubin level >0.01 mg/dL 
Thoracoabdominal Stab Wounds
Red blood cell count >10,000'mL
White blood cell count >500'mL
Amylase level >19IU'L
Alkaline phosphatase level >2WL
Bilirubin level >0.01 mg/dL
How well did you know this?
1
Not at all
2
3
4
5
Perfectly
11
Q

After an automobile accident, a 30-year-old woman is discovered to have a posterior pelvic fracture. Hypotension and tachycardia respond marginally to volume replacement. Once it is evident that her major problem is free intraperitoneal bleeding and a pelvic hematoma in association with the fracture, appropriate management
would be

A. Application of medical antishock trousers with inflation of the extremity and abdominal sections.

B. Arterial embolization of the pelvic vessels.

C. Celiotomy and ligation of the internal iliac arteries
bilaterally.

D. Celiotomy and pelvic packing.

E. External fixation application to stabilize the pelvis.

A

Answer: D

Severe pelvic bleeding is a major problem in the trauma patient. Neither external fixation nor the use of medical anti-shock trousers control free intra-abdominal hemorrhage regardless of its source.

In the unstable patient, celiotomy is mandatory. If there is a ruptured retroperitoneal hematoma bleeding into the peritoneal cavity, control is a major problem.

Internal iliac artery ligation has been abandoned as it is
rarely effective.

Angiography and arterial embolization may be effective with an arterial bleeding problem, but most severe pelvic hemorrhage is venous in origin.

If the hematoma is stable, it is best to leave it undisturbed. However, if the hematoma has ruptured into the peritoneal cavity, pelvic packing offers the best hope of control.

How well did you know this?
1
Not at all
2
3
4
5
Perfectly
12
Q

Which is true of vascular injuries of the extremities?

A. In the absence of hard signs of vascular injury, if the difference between SBP in an injured limb is within
15% of the uninjured limb, no further evaluation is needed.

B. Occult profunda femoris injuries can result in compartment syndrome and limb loss.

C. All patients with significant hematoma should be surgically explored.

D. Vascular injury repair should be performed prior to
realignment of bony fractures or dislocations.

A

Answer: B

Physical examination often identifies arterial injuries, and findings are classified as cither hard signs or soft signs of vascular injury (Table 7-3).

In general, hard signs constitute indications for operative exploration, whereas soft signs are
indications for further testing or observation.

Bony fractures or knee dislocations should be realigned before definitive vascular examination.

In management of vascular trauma, controversy exists regarding the treatment of patients with soft signs of injury, particularly those with injuries in proximity
to major vessels. It is known that some of these patients will have arterial injuries that require repair.

The most common approach has been to measure SBP using Doppler ultrasonography and compare the value for the injured side with that for the uninjured side, termed the A-A index. If the pressures are within 10% of each other, a significant injury is unlikely and no further evaluation is performed.

If the difference is >10%, CTA or arteriography is indicated.

Others argue that there are occult injuries, such as pseudoaneurysms or injuries of the profunda femoris or peroneal arteries, which may not be detected with this technique.

If hemorrhage occurs from these injuries, compartment syndrome and limb loss may occur.

Although busy trauma centers continue to debate this issue, the surgeon who is obliged to treat the occasional injured patient may be better served by performing CTA in selected patients with soft signs.

Hard Signs (Operation Mandatory):
Pulsatile hemorrhage
Absent pulses
Acute ischemia

Soft Signs (Further Evaluation Indicated):
Proximity to vasculature
Significant hematoma
Associated nerve injury
A-A index (systolic blood pressure on the injured side compared with that on the uninjured side) of <0.9
Thrill or bruit

How well did you know this?
1
Not at all
2
3
4
5
Perfectly
13
Q

Which of the following statements about blunt carotid injuries is true?

A. Magnetic resonance imaging is the diagnostic modality of choice in patients at risk.

B. Approximately 50% of patients have a delayed
diagnosis.

C. The mechanism of injury is usually cervical flexion
and rotation.

D. Such injuries are always treated operatively when
identified.

A

Answer: B

Blunt injury to the carotid or vertebral arteries may cause dissection, thrombosis, or pseudoaneurysm.

More than one half of patients have a delayed diagnosis.

Facial contact resulting in hypertension and rotation appears to be the mechanism.

To reduce delayed recognition, the authors employ CTA in patients at risk, to identify these injuries before neurologic symptoms develop. The injuries frequently occur at or extend into the base of the skull and are usually not surgically accessible.

Currently accepted treatment for thrombosis and dissection is anticoagulation with heparin followed by warfarin for 3 months.

Pseudoaneurysms also occur near the base of
the skull. If they are small, they can be followed with repeat angiography.

If enlargement occurs, consideration should be
given to the placement by an interventional radiologist of a stent across the aneurysm.

Another possibility is to approach the intracranial portion of the carotid by removing the overlying bone and performing a direct repair. This method has only recently been described and has been performed in a limited
number of patients.

How well did you know this?
1
Not at all
2
3
4
5
Perfectly
14
Q

Massive transfusion protocols

A. Should include transfusion of plasma and platelets in addition to packed RBCs

B. Should only be initiated after blood typing, but cross match is not needed

C. Should be initiated in patients with tachycardia despite administration of 3.5 Lof crystalloid fluids

D. Should include testing for coagulopathies, present in 5% of patients requiring massive transfusion

A

Answer: A

In the critically injured patient requiring large amounts of blood component therapy, a massive transfusion protocol should be followed.

This approach calls for administration of various components in a specific ratio during transfusion
to achieve restoration of blood volume and correction of coagulopathy.

Although the optimal ratio is yet to be determined, current scientific evidence indicates a presumptive 1:2 RBC: plasma ratio in patients at risk for massive transfusion.

Because complete typing and cross-matching takes up to 45 minutes, patients requiring emergent transfusions are given type O, type-specific, or biologically compatible RBCs.

Blood typing, and to a lesser extent cross-matching, is essential to
avoid life-threatening intravascular hemolytic transfusion reactions.

Injured patients with life-threatening hemorrhage develop an acute coagulopathy of trauma (ACOT). Activated protein C is a key element, although the complete mechanism remains
to be elucidated.

Fibrinolysis is an important component of the ACOT; present in only 5% of injured patients requiring
hospitalization, but 20% in those requiring massive transfusion.

How well did you know this?
1
Not at all
2
3
4
5
Perfectly
15
Q

T e most appropriate treatment for a duodenal hematoma that occurs from blunt trauma is

A. Exploratory laparotomy and bypass of the duodenum.

B. Exploratory laparotomy and evacuation of the
hematoma.

C. Exploratory laparotomy to rule out associated
injuries.

D. Observation.

A

Answer: D

Duodenal hematomas are caused by a direct blow to the abdomen and occur more often in children than adults.

Blood accumulates between the seromuscular and submucosal layers, eventually causing obstruction.

The diagnosis is suspected by the onset of vomiting following blunt abdominal trauma; barium examination of the duodenum reveals either the coiled spring sign or obstruction.

Most duodenal hematomas in children can be managed nonopcratively with nasogastric suction and parenteral nutrition.

Resolution of the obstruction occurs in the majority of patients if this therapy is continued for 7 to 14 days.

If surgical intervention becomes necessary, evacuation of the hematoma is associated with equal success but fewer complications than bypass procedures.

Despite few existing data on adults, there is no reason to believe that their hematomas should be treated differently from those of children.

A new approach is laparoscopic evacuation if the obstruction persists more than 7 days.

How well did you know this?
1
Not at all
2
3
4
5
Perfectly
16
Q

Damage control surgery (DCS)

A. Limits enteric spillage by rapid repair of partial small bowel injuries with whipstitch, and complete transection with a GIA stapling device.

B. Aims to control surgical bleeding and identify injuries that can be managed conservatively or with interventional radiology.

C. Is indicated when patients develop intraoperative refractory hypothermia, scrum pH >7.6, or refractory coagulopathy.

D. Abdominal wall should be closed with penetrating towel clips.

A

Answer: A

The goal of damage control surgery (DCS) is to control surgical bleeding and limit gastrointestinal (GI) spillage. The operative techniques used are temporary measures, with definitive repair of injuries delayed until the patient is physiologically replete.

Small GI injuries (stomach, duodenum, small intestine, and colon) may be controlled using a rapid whipstitch of 2-0 polypropylene.

Complete transection of the bowel or segmental damage is controlled using a GIA stapler, often with resection of the injured segment.

Before the patient is returned to the surgical intensive care unit (SICU), the abdomen must be temporarily closed.

Originally, penetrating towel clips were used to approximate the skin; however, the ensuing bowel edema often produces a delayed abdominal compartment syndrome.

Instead, the bowel is covered with a fenestrated subfascial sterile drapes (45 x 60 cm Steri-Drape 3M
Health Care), and two Jackson-Pratt drains are placed along the fascial edges; this is then covered using an Ioban drape, which allows closed suction to control reperfusion-related ascitic fluid egress while providing adequate space for bowel expansion to prevent abdominal compartment syndrome.

How well did you know this?
1
Not at all
2
3
4
5
Perfectly
17
Q

Therapy for increased intracranial pressure (ICP) in a patient with a closed head injury is instituted when the

ICP is greater than

A. 10

B. 20

C. 30

D. 40

A

Answer: B

In patients with abnormal findings on CT scans and GCS scores of <8, intracranial pressure (ICP) should be monitored using fiberoptic intraparenchymal devices or intraventricular catheters.

Although an ICP of 10 mm Hg is believed to be the
upper limit of normal, therapy generally is not initiated until ICP is >20 mm Hg.

Indications for operative intervention to remove space-occupying hematomas are based on the clot volume, amount of midline shift, location of the clot, GCS score, and ICP.

A shift or>5 mm typically is considered an indication for evacuation, but this is not an absolute rule.

How well did you know this?
1
Not at all
2
3
4
5
Perfectly
18
Q

Cerebral perfusion pressure (CPP)

A. Equals the SBP minus ICP

B. Should be targeted to be greater than 100 mmHg

C. Is lowered with sedation, osmotic diuresis, paralysis,
ventricular drainage, and barbiturate coma

D. Can be increased by lowering ICP and avoiding
hypotension

A

Answer: D

The goal of resuscitation and management in patients with head injuries is to avoid hypotension (SBP of <100 mm Hg) and hypoxia (partial pressure of arterial oxygen of <60 or arterial oxygen saturation of <90).

Attention, therefore, is focused on maintaining cerebral perfusion rather than merely lowering ICP.

Resuscitation efforts aim for a euvolemic state and
an SBP of >100 mm Hg.

Cerebral perfusion pressure (CPP) is equal to the mean arterial pressure minus the ICP, with a target range of >50 mm Hg.

CPP can be increased by either lowering ICP or raising mean arterial pressure.

Sedation, osmotic diuresis, paralysis, ventricular drainage, and barbiturate coma are used in sequence, with coma induction being the last resort.

How well did you know this?
1
Not at all
2
3
4
5
Perfectly
19
Q

An 18-year-old man is admitted to the ED shortly after being involved in an automobile accident. He is in a coma (GCS = 7). His pulse is barely palpable at a rate of 140 beats per minute, and BP is 60/0. Breathing is rapid and shallow, aerating both lung fields. His abdomen is moderately distended with no audible peristalsis. There are closed fractures of the right forearm and the left lower leg.

After rapid IV administration of 2 L of lactated Ringer solution in the upper extremities, his pulse is 130 and BP
70/0. The next immediate step should be to

A. Obtain cross-table lateral X-rays of the cervical spine.

B. Obtain head and abdominal CT scans.

C. Obtain supine and lateral decubitus X-rays of the abdomen.

D. Obtain an arch aortogram.

E. Explore the abdomen.

A

Answer: E

Ideally, a patient seriously injured in an automobile accident should undergo X-rays of the cervical spine, the chest, and the abdomen.

When he has a GCS of 7, CT scans of the head are
certainly desirable. If the chest X-ray shows a widened mediastinum, arch aortograms are indicated. However, this patient has had no response to a rapid fluid challenge, and if he is to survive, bleeding must be controlled immediately.

The head injury, although severe, is not responsible for his hypotension and tachycardia.

The most likely problem is uncontrolled abdominal hemorrhage. Immediate abdominal exploration offers the best chance for survival.

How well did you know this?
1
Not at all
2
3
4
5
Perfectly
20
Q

A 36-year-old patient arrives in the trauma bay with a stab wound to the left chest. After placement of a left thoracostomy tube and fluid resuscitation, his breathing is stable with BP 160/74 mm Hg and heart rate of 110 beats per minute. CT scanning reveals a descending thoracic pscudoaneurysm and no intracranial or intra-abdominal injury. What is the most appropriate next step?

A. Open repair with partial left heart bypass

B. Endovascular repair with stent

C. Esmolol drip

D. Admission to SICU with repeat CT in 24 hours

A

Answer: C

Descending thoracic aortic injuries may require urgent if not emergent intervention.

However, operative intervention for intracranial or intra-abdominal hemorrhage or unstable pelvic fractures takes precedence.

To prevent aortic rupture, pharmacologic therapy with a selective ß-antagonist, esmolol, should be instituted in the trauma bay, with a target SBP of <100 mm Hg and heart rate of <100 beats per minute.

Endovascular stenting is now the mainstay of treatment, but open operative reconstruction is warranted, or necessary, in select patients.

Endovascular techniques are particularly appropriate in patients who cannot tolerate single lung ventilation,
patients older than 60 years who are at risk for cardiac decompensation with aortic clamping, or patients with uncontrolled intracranial hypertension.

How well did you know this?
1
Not at all
2
3
4
5
Perfectly
21
Q

A patient with penetrating injury to the chest should
undergo thoracotomy if

A. There is more than 500 mL of blood which drains from the chest tube when placed.

B. There is more than 200 mL/h of blood for 3 hours
from the chest tube.

C. There is an air leak that persists for >48 hours.

D. There is documented lung injury on CT scan.

A

Answer: B

The most common injuries from both blunt and penetrating thoracic trauma are hemothorax and pneumothorax.

More than 85% of patients can be definitively treated with a chest tube. The indications for thoracotomy include significant initial or ongoing hemorrhage from the tube thoracostomy and specific imaging-identified diagnoses.

One caveat concerns the patient who presents after a delay. Even when the initial chest tube output is 1.6 L, if the output ceases and the lung is re-expanded, the patient may be managed non operatively, if hemodynamically stable.

Indications for operative treatment of thoracic injuries:

  • Initial tube thoracostomy drainage of >1000 mL (penetrating injury) or >1500 mL (blunt injury)
  • Ongoing tube thoracostomy drainage of >200 mL/h or 3 consecutive hours in noncoagulopathic patients
  • Caked hemothorax despite placement of two chest tubes
  • Selected descending torn aortas
  • Great vessel injury (endovascular techniques may be used in selected patients)
  • Pericardial tamponade
  • Cardiac herniation
  • Massive air leak rom the chest tube with inadequate ventilation
  • Tracheal or mainstem bronchial injury diagnosed by endoscopy or imaging
  • Open pneumothorax
  • Esophageal per oration
  • Air embolism
How well did you know this?
1
Not at all
2
3
4
5
Perfectly
22
Q

After sustaining a gunshot wound to the right upper quadrant of the abdomen, the patient has no signs of peritonitis.

Her vital signs are stable, and CT scan shows a grade III
liver injury. What is the next step in management?

A. Exploratory laparotomy with control of hepatic
parenchymal hemorrhage.

B. Admission to SICU with serial complete blood count.

C. Admission to SICU with repeat CT in 24 hours.

D. Hepatic angiography.

A

Answer: B

The liver’s large size makes it the organ most susceptible to blunt trauma, and it is frequently involved in upper torso penetrating wounds.

Nonoperative management of solid organ injuries is pursued in hemodynamically stable patients who do not have overt peritonitis or other indications for laparotomy.

Patients with more than grade II injuries should be admitted to the SICU with frequent hemodynamic monitoring, determination of hemoglobin, and abdominal examination.

The only absolute contraindication to nonoperative management is hemodynamic instability.

Factors such as high injury grade, large hemoperitoneum, contrast extravasation, or pseudoaneurysms may predict complications or failure of nonoperative management.

Angioembolization and endoscopic retrograde
cholangiopancreatography (ERCP) are useful adjuncts that can improve the success rate of nonoperative management.

The indication for angiography to control hepatic hemorrhage is transfusion of 4 units of RBCs in 6 hours or 6 units of RBCs in 24 hours without hemodynamic instability.

How well did you know this?
1
Not at all
2
3
4
5
Perfectly
23
Q

A 25-year-old man has multiple intra-abdominal injuries after a gunshot wound. Celiotomy reveals multiple injuries to small and large bowel and major bleeding from the liver. After repair of the bowel injuries, the abdomen is closed with towel clips, leaving a large pack in the injured liver. Within 12 hours, there is massive abdominal swelling with edema fluid, and intra-abdominal pressure exceeds 35 mm Hg. The immediate step in managing this problem is to

A. Administer albumin intravenously

B. Give an IV diuretic

C. Limit IV fluid administration

D. Open the incision to decompress the abdomen

A

Answer: D

Cardiac, pulmonary, and renal problems develop when invasive ascites compresses the diaphragm and the inferior vena cava. Dialysis, diuresis, and increasing scrum oncotic pressure will not correct this problem rapidly enough to save the patient’s life.

Opening the incision relieves the intra-abdominal
pressure. There are few reports of sudden hypotension after this maneuver, but volume loading has largely eliminated this problem.

How well did you know this?
1
Not at all
2
3
4
5
Perfectly
24
Q

Which of the following statements is correct regarding traumatic spleen injury?

A. An elevation in WBC to 20,000/mm3 and platelets to 300,000/mm3 on postoperative day 7 is a common benign finding in postsplenectomy patients.

B. Delayed rebleeding or rupture will typically occur within 48 hours of injury.

C. Common complications after splenectomy include
subdiaphragmatic abscess, pancreatic tail injury, and gastric perforation.

D. Postsplcncctomy vaccines against encapsulated bacteria is optimally administered preoperatively or
immediately postoperative.

A

Answer: C

Unlike hepatic injuries, which usually rebleed within 48 hours, delayed hemorrhage or rupture of the spleen can occur up to weeks after injury.

Indications for early intervention include
initiation of blood transfusion within the first 12 hours and
hemodynamic instability.

After splenectomy or splenorrhaphy, postoperative hemorrhage may be due to loosening of a tie
around the splenic vessels, an improperly ligated or unrecognized short gastric artery, or recurrent bleeding from the spleen if splenic repair was used.

An immediate postsplenectomy increase in platelets and WBCs is normal; however, beyond postoperative day 5, a WBC count above 15,000/mm3 and a platelet/WBC ratio of <20 arc strongly associated with sepsis and should prompt a thorough search for underlying infection.

A common infectious complication after splenectomy is a subphrenic abscess, which should be managed with percutaneous drainage.

Additional sources of morbidity include a concurrent but unrecognized iatrogenic injury to the pancreatic tail during rapid splenectomy resulting in pancreatic ascites or fistula, and a gastric perforation during short gastric ligation.

Enthusiasm for splenic salvage was driven by the rare, but often fatal, complication of overwhelming postsplenectomy sepsis.

Overwhelming postsplenectomy sepsis is caused by encapsulated bacteria, Streptococcus pneumoniae, Haemophilus influenzae, and Neisseria meningitidis, which are resistant to antimicrobial
treatment.

In patients undergoing splenectomy, prophylaxis
against these bacteria is provided via vaccines administered optimally at 14 days.

How well did you know this?
1
Not at all
2
3
4
5
Perfectly
25
Q

The most appropriate treatment for a duodenal hematoma that occurs from blunt trauma is

A. Exploratory laparotomy and bypass of the duodenum

B. Exploratory laparotomy and evacuation of the
hematoma

C. Exploratory laparotomy to rule out associated injuries

D. Observation

A

Answer: D

Duodenal hematomas arc caused by a direct blow to the abdomen and occur more often in children than adults. Blood accumulates between the seromuscular and submucosal layers, eventually causing obstruction.

The diagnosis is suspected by the onset of vomiting following blunt abdominal trauma; barium examination of the duodenum reveals either the coiled spring sign or obstruction.

Most duodenal hematomas in children can be managed nonoperatively with nasogastric suction and parenteral nutrition.

Resolution of the obstruction occurs in the majority of patients if this therapy is continued for 7 to 14 days.

If surgical intervention becomes necessary, evacuation of the hematoma is associated with equal success but fewer complications than bypass procedures.

Despite few existing data on adults, there is no reason to believe that their hematomas should be treated differently from those of children.

A new approach is laparoscopic evacuation if the obstruction persists more than 7 days.

How well did you know this?
1
Not at all
2
3
4
5
Perfectly
26
Q

A 19-year-old man fell of his skateboard, reporting blunt injury to his upper abdomen.

Abdominal CT and magnetic resonance cholangiopancreatography (MRCP) confirmed he suffered transection of the main pancreatic duct at the middle of the pancreatic body.

Which of the following would be the most appropriate next step in management?

A. Nonoperative treatment

B. Endoscopic retrograde cholangiopancreatography
(ERCP) with stenting of pancreatic duct

C. Distal pancreatectomy with splenic preservation

D. Primary repair of pancreatic duct with closed suction drainage

A

Answer: C

Optimal management of pancreatic trauma is determined by where the parenchymal damage is located and whether the intrapancreatic common bile duct and main pancreatic duct remain intact.

Patients with pancreatic contusions (defined as injuries that leave the ductal system intact) can be treated nonoperatively or with closed suction drainage if undergoing laparotomy for other indications.

Patients with proximal pancreatic injuries, defined as those that lie to the right of the superior mesenteric vessels, are also managed with closed suction drainage.

In contrast, distal pancreatic injuries are managed based upon ductal integrity.

Pancreatic duct disruption can be identified through direct exploration of the parenchymal laceration, operative pancreatography, endoscopic retrograde pancreatography (ERCP), or magnetic resonance
cholangiopancreatography (MRCP).

Patients with distal ductal disruption undergo distal pancreatectomy, preferably with splenic preservation.

An alternative, which preserves both the spleen and distal transected end of the pancreas, is either
a Roux-en-Y pancreaticojejunostomy or pancreaticogastrostomy.

If the patient is physiologically compromised, distal
pancreatectomy with splenectomy is the preferred approach.

How well did you know this?
1
Not at all
2
3
4
5
Perfectly
27
Q

The most appropriate treatment for a gunshot wound to the hepatic flexure of the colon that cannot be repaired
primarily is

A. End colostomy and mucous fistula.

B. Loop colostomy.

C. Exteriorized repair.

D. Resection of the right colon with ilcocolostomy.

A

Answer: D

Numerous large retrospective and several prospective studies have now clearly demonstrated that primary repair is safe and effective in the majority of patients with penetrating injuries.

Colostomy is still appropriate in a few patients, but the current dilemma is how to select them.

Exteriorized repair is probably no longer indicated since most patients who were once candidates for this treatment are now successfully managed by primary repair.

Two methods have been advocated that result in 75 to 90% of penetrating colonic injuries being safely treated by primary repair.

The first is to repair all perforations not requiring resection. If resection is required due to the local extent of the injury, and it is proximal to the middle colic artery, the proximal portion of the right colon up to and including the injury is resected and an ileocolostomy performed.

If resection is required distal to the middle colic
artery, an end colostomy is created and the distal colon oversewn and left within the abdomen.

The theory behind this approach is that an ileocolostomy heals more reliably than colocolostomy, because in the trauma patient who has suffered shock and may be hypovolemic, assessing the adequacy of the blood supply of the colon is much less reliable than in elective procedures.

The blood supply of the terminal ileum is never a problem. The other approach is to repair all injuries regardless of the extent and location (including colocolostomy), and reserve colostomy for patients with protracted shock and extensive contamination.

The theory used to support this approach is that systemic factors are more important than local factors in determining whether a suture line will heal.

Both of these approaches are reasonable and result in the majority of patients being treated by primary repairs.

When a colostomy is required, regardless of the theory used to reach that conclusion, performing a loop colostomy proximal to a distal repair should be avoided because a proximal colostomy does not protect a distal suture line.

All suture lines and anastomoses are performed with the running single-layer technique.

How well did you know this?
1
Not at all
2
3
4
5
Perfectly
28
Q

Which of the following statements is FALSE regarding traumatic genitourinary injury?

A. If exploratory laparotomy is performed for trauma, all blunt and penetrating wounds to the kidneys should be explored.

B. Renal vascular injuries are common after penetrating trauma, and can be deceptively tamponaded by surrounding fascia.

C. Success of renal artery repair after blunt trauma is slim, but can be attempted if injury occurred within 5 hours or patient does not have any reserve renal function (solitary kidney or bilateral injury).

D. Suspected ureteral injuries in patients with penetrating trauma or pelvic fractures can be evaluated intraoperatively with methylene blue or indigo carmine administered intravenously.

E. Bladder injuries with extraperitoneal extravasation can be managed with Foley decompression for 2 weeks.

A

Answer: A

When undergoing laparotomy for trauma, the best policy is to explore all penetrating wounds to the kidneys.

However, over 90% of blunt injuries are treated nonoperatively; the indications for surgery include parenchymal injuries leading to hypotension and evidence of renovascular injury.

If laparotomy is performed in the setting of blunt kidney injury for other reasons, expanding or pulsatile perinephric hematomas should be explored.

Injuries to the ureters are uncommon but may occur in patients with pelvic fractures and penetrating trauma.

An injury may not be identified until a complication (ie, a urinoma) becomes apparent.

If an injury is suspected during operative exploration but is not clearly identified, methylene blue or indIgo carmine is administered IV with observation for extravasation.

Bladder injuries are subdivided into those with intraperitoneal extravasation and those with extraperitoneal extravasation.

Extraperitoneal ruptures are treated nonoperatively with bladder decompression for 2 weeks, whereas injuries with intraperitoneal extravasation can be closed primarily.

Urethral injuries are managed by bridging the defect with a Foley catheter, with or without direct suture repair.

Strictures are not uncommon but can be managed electively.

How well did you know this?
1
Not at all
2
3
4
5
Perfectly
29
Q

At what pressure is operative decompression of a compartment mandatory?

A. 15 mmHg

B. 25mmHg

C. 35mmHg

D. 45mmHg

A

Answer: D

In comatose or obtundcd patients, the diagnosis is more difficult to secure.

A compatible history, firmness of the compartment to palpation, and diminished mobility of the joint are suggestive.

The presence or absence of a pulse distal to the affected compartment is notoriously unreliable in the diagnosis of a compartment syndrome.

A frozen joint and myoglobinuria are late signs and suggest a poor prognosis.

As in the abdomen, compartment pressure can be measured. The small, hand-held Stryker device is a convenient tool for this purpose.

Pressures greater than 45 mm Hg usually require operative intervention.

Patients with pressures between 30 and 45 mm
Hg should be carefully evaluated and closely watched.

(See Schwartz 10th ed.,p. 215.)

How well did you know this?
1
Not at all
2
3
4
5
Perfectly
30
Q

Which is true regarding trauma in geriatric patients?

A. Admission GCS score after severe head injury is a good predictor of outcome.

B. Rib fractures are associated with pulmonary contusion in 35% of patients, and complicated by pneumonia in 10 to 30% of patients.

C. Approximately 10% of patients older than 65 years
will sustain a rib fracture from a fall <6 ft.

D. Chronologic age older than 65 years is associated with higher morbidity and mortality after trauma.

A

Answer: B

Mortality in patients with severe head injury more than doubles after the age of 55 years.

Moreover, 25% of patients with a normal GCS score of 15 had intracranial bleeding, with an associated mortality of 50%.

Just as there is no absolute age that predicts outcome, admission GCS score is a poor predictor of individual outcome.

Therefore, the majority of trauma centers advocate an initial aggressive approach with re-evaluation at the 72-hour mark to determine subsequent care.

Secondly, one of the most common sequelae of blunt thoracic trauma is rib fractures. In fact, in one study, 50% of patients older than 65 years sustained rib fractures from a fall of <6 ft, compared with only 1% of patients younger than 65 years.

Concurrent pulmonary contusion is noted in up to 35% of patients, and pneumonia complicates the injuries in 10 to 30% of patients with rib fractures.

How well did you know this?
1
Not at all
2
3
4
5
Perfectly
31
Q

A 76-year-old man with hypertension, chronic renal insufficiency, and Child class A cirrhosis is admitted to the intensive care unit (ICU) after emergency exploratory laparotomy for ruptured appendicitis. His vital signs are a temperature of 97.3°F, heart rate (HR) of 129 beats/min, blood pressure (BP) of 220/90 mmHg, respiratory rate (RR) of 30 breaths/min, and oxygen saturation (Sao2) of 90%. The patient is agitated and trying to pull his drains and nasogastric tube. He does not appear to respond to commands. Select the best choice to sedate this patient.

A. Lorazepam, 5 mg intravenously (IV)

B. Four-point restraints while trying to reason with the patient

C. Morphine delivered by patient-controlled anesthesia (PCA) with settings of 1 mg every 6 min and a 30-mg 4-h lockout

D. Diprivan and fentanyl drip

E. Placement of an epidural catheter for analgesia

A

ANSWER: D

COMMENTS: In the ICU, management of pain can be difficult and is often complicated by an inability to communicate with the patient and by the patient’s physiologic instability, comorbid conditions, or delirium.

Several methods have been developed to help assess sedation, including the Riker Sedation-Agitation Scale and the Ramsay Scale.

This patient has both renal and hepatic dysfunction, which makes lorazepam an incorrect choice. It has a slow onset and intermediate half-life.

In this situation, a faster-acting drug is preferable because the patient is obviously agitated.

A Diprivan and fentanyl drip is the best answer because Diprivan is a general anesthetic agent with a rapid onset and ultrashort duration of action.

Side effects of this medication include a risk for hypotension, high cost, pain on injection, and potential for hypertriglyceridemia.

It has no analgesic effect and therefore additional medication is required to control the pain.

Fentanyl is a better choice for analgesia because of the patient’s renal failure and its rapid onset of action relative to morphine, which can take 5 to 10 min.

The use of four-point restraints without additionally sedating the patient is not a good option.

Again, PCA is not a good option for a patient intubated and needing further sedation because of agitation.

Moreover, morphine and its active metabolites (morphine-3-glucuro- nide and morphine-6-glucuronide) can accumulate in patients with renal insufficiency.

Finally, placing an epidural catheter in an agitated patient would be difficult and dangerous to the patient and staff.

How well did you know this?
1
Not at all
2
3
4
5
Perfectly
32
Q

A 53-year-old man with coronary artery disease, Child class B alcoholic cirrhosis, and chronic renal insufficiency is admitted to the ICU after undergoing exploratory laparotomy and resection of necrotic small bowel from an incarcerated ventral hernia. He is septic and continues to require mechanical ventilation. Arterial blood gas analysis revealed a pH of 7.59, Pco2 of 20 mmHg, Po2 of 59 mmHg, HCO3 of 21 mEq/L, base deficit of −2, and Sao2 of 88%. The nurse calls because the ventilator alarms continue to go off. The patient is actually breathing at a rate of 43 breaths/min. After adequately sedating him, he is still dyssynchronous with the ventilator. Which paralytic agent is the most appropriate for this patient?

A. Pancuronium

B. Cisatracurium

C. Vecuronium

D. Succinylcholine

E. Rocuronium

A

ANSWER: B

COMMENTS: The best choice is cisatracurium, a nondepolarizing neuromuscular blocker and one of the most commonly used paralytics in the ICU.

It, along with atracurium, is metabolized by plasma ester hydrolysis and Hofmann elimination and is therefore the best choice in this patient with both hepatic and renal dysfunction.

Pancuronium is long acting but contraindicated in patients with coronary artery disease because it has a vagolytic effect and induces tachycardia.

Vecuronium is intermediate acting (30 min) but is cleared by the kidney and liver.

Succinylcholine is a short- acting depolarizing neuromuscular blocker, usually for facilitation of endotracheal intubation.

Rocuronium has a rapid onset and intermediate duration thus making it a better choice for short procedures, as opposed to the needs of this patient, who must be sedated for a longer period.

How well did you know this?
1
Not at all
2
3
4
5
Perfectly
33
Q

Which of the following statements concerning radial artery cannulation is true?

A. Aortic systolic pressure is higher than radial systolic pressure.

B. The Allen test is an outdated mode of assessing the collateral flow of the ulnar and radial arteries.

C. The incidence of infection is higher with catheters placed by surgical cutdown.

D. The catheter should be replaced every 3 days.

E. Intermittent flushing to keep the catheter free of clots is desirable.

A

ANSWER: C

COMMENTS: The incidence of complications after arterial catheterization seems to be operator independent, unlike the case with pulmonary artery (PA) catheterization.

Known risk factors include intermittent punctures, age younger than 10 years, prolonged catheterization (>4 days), anticoagulant therapy, and use of a catheter larger than 20 gauge or made of polypropylene rather than Teflon.

The radial artery is the site most frequently used for catheterization, provided that the ulnar artery and palmar arterial arch are patent.

Therefore the Allen test should be performed before attempting radial artery catheterization.

A normal test result consists of a palmar blush within 7s after the ulnar artery is released.

Most patients with arterial thrombosis remain asymptomatic. Symptoms can be minimized by placing lines in arteries with good collateral circulation.

Most thrombi (43%) are present at the time of catheter removal and another 30% develop within 24 h.

A higher incidence of thrombosis occurs within the first 24h when surgical cutdown is performed (48% vs. 23% with percutaneous placement), but the incidence of thrombosis at 1 week is the same for both methods of placement.

Brachial artery cannulation has a high incidence of embolic occlusion of the distal arteries (5%–41%) and should therefore be avoided.

Infection remains the most common complication.

Predisposing factors are prolonged catheterization, surgical cutdown, local inflammation, pre- existing bacteremia, and failure to change the saline flush fluid, transducer, and flush tubing every 48 h.

The need for intermittent arterial catheter replacement is not established and indeed is controversial.

The aortic mean arterial pressure (MAP) and diastolic arterial pressure are slightly higher than the radial MAP and diastolic arterial pressure.

However, systolic pressure is consistently higher in the radial artery than in the aorta. This discrepancy increases with distal progression, smaller arterial caliber, and age and is explained by the reflection of pressure waves from capillary beds, which results in augmentation of the systolic and reduction of the diastolic values measured.

How well did you know this?
1
Not at all
2
3
4
5
Perfectly
34
Q

A 70-kg, 72-year-old man known to suffer from congestive heart failure (CHF), arthritis, diabetes mellitus, and a first-degree heart block is intubated in the ICU on postoperative day 2 after exploratory laparotomy for perforated sigmoid diverticulitis. His urine output has dropped to 10 mL/h for the last shift, and he is hypotensive despite several fluid boluses. A PA catheter is placed through the right internal jugular vein with some difficulty. As the line is advanced to 50 cm, the patient has a 14-beat run of ventricular tachycardia, which resolves when the catheter is pulled back. It is finally advanced to 62 cm, and the balloon is inflated with 3cc of air. As the line is being secured, a large amount of blood is noted in the endotracheal tube and the patient becomes hypotensive. Select the best intervention for this patient:

A. Place external pacing wires and administer lidocaine to treat the ventricular tachycardia.

B. Place a double-lumen endotracheal tube and occlude the appropriate bronchus with a Fogarty catheter.

C. Pull the PA catheter back 2 cm with the balloon inflated.

D. Suction the endotracheal tube while deflating the balloon by 2 cc of air.

E. Obtain a chest radiograph to confirm the correct placement of the line.

A

ANSWER: B

COMMENTS: The indications for PA catheters and their value in patients with sepsis or hemodynamic instability are uncertain, but they may be useful in the management of patients unresponsive to the use of fluids and vasoactive agents.

Dysrhythmias occur in 12%–67% of patients undergoing catheterization but are usually self-limited, premature ventricular contractions.

Complete heart block can develop in patients with preexisting left bundle branch block.

A prophylactic pacing wire should be used in these patients.

Prophylactic lidocaine and full inflation of the balloon may prevent ventricular ectopy.

Hemoptysis in patients with a PA catheter sug- gests the diagnosis of perforation or rupture.

Mechanisms involved in PA rupture include

(1) overinflation of the balloon,
(2) incomplete balloon inflation (<75%) with the exposed tip being forced through the wall, and
(3) pulmonary hypertension

An “overwedge” pattern suggests eccentric balloon inflation, overdistention, or both. If hemoptysis develops, the catheter should be pulled back, with the balloon deflated.

Massive hemoptysis necessitates placement of a double-lumen endotracheal tube and occlusion of the bronchus on the side of the rupture with a Fogarty catheter.

Emergency thoracotomy is needed. Looping or knotting of the catheter may occur in the right ventricle during insertion and can be avoided if no more than 10 cm of the catheter are inserted after a ventricular tracing is identified and before a PA tracing appears.

Although catheter-related sepsis occurs in only up to 2% of insertions, bacterial colonization takes place in 5%–35% of catheterizations.

Infections are more common when the catheter is left in place for more than 72 h or when it is inserted via an antecubital vein.

How well did you know this?
1
Not at all
2
3
4
5
Perfectly
35
Q

A 51-year-old morbidly obese female who has a known history of symptomatic cholelithiasis is admitted to the ICU after presenting to the emergency department (ED) with severe epigastric pain. Her temperature is 100.5°F; HR, 115 beats/min; and BP, 123/84 mmHg. Her laboratory values are significant for a white blood cell (WBC) count of 15,000/ mm3 and a lipase of 1547. A computed tomography (CT) scan shows peripancreatic inflammation with a small fluid collection.

What is the next best step in the management of this patient?

A. Central line placement for monitoring of central venous pressure (CVP)

B. Broad-spectrum antibiotics

C. Placement of an enteral feeding tube and initiation of tube feeding

D. Percutaneous drain placement

E. Cholecystectomy

A

ANSWER: C

COMMENTS: Acute pancreatitis may have a variable presentation dependent on both etiology and severity.

If the patient is presenting with signs of shock and organ hypoperfusion, initial interventions must be targeted toward the improvement of oxygen delivery.

Additionally, CT is helpful in determining if pancreatic necrosis with or without infection is present.

Typically, infected pancreatic necrosis will present with air within the peripancreatic fluid collection, necessitating drainage or debridement.

The role of prophylactic antibiotics in severe pancreatitis has been debated without consensus.

A small number of randomized trials found benefit with fewer infectious complications when antibiotics are given prophylactically. However, this has not been widely repro- ducible.

Cholecystectomy should be performed prior to the patient’s discharge from the hospital, but not in the acute setting.

Early enteral feeding, typically beyond the second portion of the duodenum, is preferable over parenteral nutrition and is associated with fewer infectious complications.

How well did you know this?
1
Not at all
2
3
4
5
Perfectly
36
Q

Of the following parameters, which is the best predictor of
successful extubation?

A. Increase in Paco2 of less than 10 mmHg during a spontaneous breathing trial (SBT)

B. Spontaneous tidal volume (Vt)

C. 10-s head raise

D. Rapid shallow breathing index (RSBI)

E. Minute ventilation

A

ANSWER: D

COMMENTS: Many measures are utilized in order to determine the appropriateness for a trial of extubation.

The RSBI is RR divided by Vt.

A value of more than 105 predicts failure of extuba- tion in over 95% of patients, while an RSBI less than 105 predicts success in over 80% of patients.

Other factors can be useful in the decision to extubate; however, they are individually not as reliable as the RSBI.

The importance of reliable and objective indicators for successful liberation from the ventilator is tied to the avoidance of complications such as ventilator-associated pneumonia (VAP), along with a decreased requirement for sedative medications.

How well did you know this?
1
Not at all
2
3
4
5
Perfectly
37
Q

A 44-year-old male heroin user is intubated in the surgical intensive care unit (SICU) after undergoing debridement of a lower extremity wound. His vital signs are a temperature of 102.3°F, HR of 134 beats/min, and BP of 80/40 mmHg with a MAP of 55 mmHg. A triple lumen catheter is placed, and CVP reads 12 mmHg. Norepinephrine is started, and despite being at the upper limit of the recommended dose, the MAP increases to 62 mmHg. What is the next best step?

A. Fluid bolus

B. Echocardiography

C. Dobutamine

D. Epinephrine

E. Vasopressin

A

ANSWER: D

COMMENTS: This patient remains in septic shock postoperatively from debridement of an infected wound.

We can presume the source to be the wound and therefore source control has been obtained.

He should remain on broad-spectrum antibiotics and achieve certain parameters as directed by goal-directed therapy.

He is intubated, which mandates a goal CVP of 10 to 12 mmHg.

If he were extubated, a lower CVP of 8 to 10 mmHg would be appropriate, given the absence of positive pressure ventilation.

This indicates he is volume resuscitated or has an adequate preload.

If you are concerned about cardiac function, or need an indicator other than CVP to determine the volume status, echocardiography is indicated.

However, this patient remains hypotensive despite norepinephrine, and something must be done in the interim.

The first vasopressor used in septic shock should be norepinephrine. If a second vasopressor is required, it is now recommended that epi- nephrine be added to norepinephrine to achieve a MAP greater than 65 mmHg.

Vasopressin may be added subsequently, but the dose should not exceed 0.03 units/min.

How well did you know this?
1
Not at all
2
3
4
5
Perfectly
38
Q

Which of the following is an indication to give calcium gluconate in a patient with hyperkalemia?

A. Serum potassium of 6.2

B. Electrocardiogram (ECG) changes consistent with hyperkalemia

C. The patient does not have preexisting renal disease

D. Both A and B

E. All of the above

A

ANSWER: B

COMMENTS: While mild hyperkalemia is well tolerated, severe hyperkalemia results in a predictable progression of cardiac effects.

Classically, peaked T waves are the first to appear on ECG, fol- lowed by a widened QRS complex, loss of P wave, appearance of “sine” waves, and eventually ventricular fibrillation. Emergent treatment of hyperkalemia focuses first on the antagonization of the depolarizing effect that a high level of potassium has on the cardiac membranes.

Calcium should be administered

(1) to any patient with a serum potassium level of 6.5 or greater or
(2) to any patient with ECG changes consistent with hyperkalemia regardless of the potas- sium level.

Other treatment of acute hyperkalemia includes redistribution of potassium to the intracellular space with insulin or β-agonists (albuterol).

Ultimately, the elimination of excess potassium from the body is achieved by administering loop diuretics, sodium bicarbonate, Kayexalate, or hemodialysis.

How well did you know this?
1
Not at all
2
3
4
5
Perfectly
39
Q

You are asked to see a 24-year-old male with no medical problems who sustained a gunshot wound to the right thigh 1 h prior to arrival. He is afebrile with an HR of 136 beats/min and a BP of 90/60 mmHg. He is initially alert and asking for water but becomes confused during your brief interview. He is in which class of hemorrhagic shock?

A. I

B. II

C. III

D. IV

E. More information is needed

A

ANSWER: C

COMMENTS: This patient presents with hemorrhagic shock with altered mental status, tachycardia, and hypotension.

Driven to maintain adequate oxygen delivery, early changes in vital signs are representative of the body’s attempt to preserve homeostasis after the rapid loss of a significant amount of its circulating volume.

Early care is directed toward eliminating ongoing sources of exsanguination, along with restoration of intravascular volume.

The degree of derangement is reliably linked to the volume of blood loss and classified according to the following table.

This patient’s constellation of findings places him within class III shock, and he has an estimated blood loss of 1.5 to 2 L.

This estimate may help in guiding volume resuscitation to give an understanding of how much volume has been lost.

The choice of fluid replacement has been the subject of much debate and investigation. Generally, for moderate-to-severe shock, resuscitation is carried out with a com- bination of both crystalloid and colloid solutions. In the acute setting, hemoglobin is an unreliable indicator; rather, therapy should be aimed at restoring adequate intravascular volume and normalized hemodynamics.

(See ATLS, Chapter on Shock)

How well did you know this?
1
Not at all
2
3
4
5
Perfectly
40
Q

A 33-year-old unhelmeted male is brought to the ED after a motorcycle crash. His temperature is 99.4°F, HR is 95 beats/ min, BP is 110/65 mmHg, and RR is 10 breaths/min with an Sao2 of 94% on room air. He makes incomprehensible sounds, withdraws extremities, and opens eyes to painful stimuli. He is intubated in the ED and placed on the ventilator for transfer to the ICU. On head CT, he is found to have small subarachnoid hemorrhage and a nondepressed skull fracture. The remainder of his trauma workup is negative. In addition to head-of-bed elevation and frequent monitoring of neurologic status, which of the following is indicated?

A. IV mannitol bolus

B. Hypertonic saline infusion

C. Craniotomy

D. Hypothermia

E. Insertion of intracranial pressure (ICP) monitor at bedside

A

ANSWER: E

COMMENTS: Normal ICP ranges from 5 to 15 mmHg. Because of the bony calvarium, any increase in ICP results in an equivalent decrease in cerebral perfusion pressure (CPP), which is the difference between MAP and ICP.

Goals for treatment of closed head injuries are to maintain adequate CPP through the use of various maneuvers to lower ICP.

This patient has suffered a severe head injury, as evidenced by his Glasgow coma score (GCS) of 8.

He has an abnormal head CT with intracranial hemorrhage and has a significant risk (50%–60%) of elevations of ICP.

For all patients with head injury, abnormal CT scan, and GCS less than 9, and ICP monitoring should be considered.

One should also consider the placement of a monitor if the patient will be unable to be examined for a prolonged period [i.e., in the operating room (OR)].

There are conflicting data regarding the potential mortality benefit of ICP monitors; however, numerous data published since 2012 indicate improved outcomes when they are placed in these patients.

Mannitol and hypertonic saline are indicated for the treatment of elevated ICP, indicated by invasive monitoring or a change in physical examination (unilateral blown pupil).

Hypothermia was found in a recent study to have no benefit in patients with traumatic brain injury and elevated ICP.

How well did you know this?
1
Not at all
2
3
4
5
Perfectly
41
Q

When comparing early tracheostomy (<10 days after endotracheal intubation) versus late tracheostomy (>10 days after endotracheal intubation), which of the following is true?

A. Incidence of VAP is the same

B. Decreased mortality in those undergoing early tracheostomy at 28 days

C. Shorter ICU length of stay

D. No difference in sedation requirements

E. Improved patient satisfaction

A

ANSWER: C

COMMENTS: The timing of tracheostomy continues to be a debated topic in both the critical care and trauma literature.

The general division between early and late tracheostomy creation is at 10 days after endotracheal intubation.

For patients predicted to have prolonged ventilator requirements, the optimal timing of tracheostomy remains uncertain.

Early trials suggested a mortality benefit; however, this result has not been reproduced in more recently published data.

There does however seem to be a decrease in the incidence of VAP, along with decreased sedation requirements, more ventilator-free days, and shorter ICU length of stay.

Trials with longer follow-up are still ongoing; however, there is some evidence that there may be a long-term mortality benefit for early tracheostomy, though more data are needed to establish this conclusion.

While pulmonary hygiene and ease of care are improved with tracheostomy, there are no definitive data suggesting improved patient satisfaction with early versus late tracheostomy creation.

How well did you know this?
1
Not at all
2
3
4
5
Perfectly
42
Q

A 53-year-old woman with a history of metastatic lung cancer is admitted to the ICU after video-assisted resection of the right middle lobe. Initial vitals are an HR of 104 beats/ min, BP of 64/43 mmHg, and RR of 34 breaths/min. After multiple fluid boluses, the patient remains hypotensive, so a PA catheter is placed and secured at 43 cm. The following values were determined: PA pressure, 38/27 mmHg; CVP, 26 mmHg; PA occlusion pressure (PAOP), 27 mmHg; and cardiac index, 2.0 L/min/m2.

Which of the following explains the clinical scenario?

A. CHF from sepsis

B. Malignant pleural effusion

C. Cardiac tamponade

D. Hypovolemia

E. Pneumothorax

A

ANSWER: C

COMMENTS: There are many potential causes of pericardial tamponade, with lung cancer, renal failure, tuberculosis, breast carcinoma, and lymphoma and leukemia being among the most common.

Hemodynamic monitoring with a PA catheter can help determine the diagnosis by showing equalization of right ventricular diastolic pressure, a PA diastolic pressure, and a PAOP within 2 to 3 mmHg of each other, along with the elevated mean right atrial pressure.

With small effusions most patients are asymptomatic, but with fluid in excess of 500 mL, patients can experience the onset of dyspnea, cough, chest pain, tachycardia, and jugular venous distention.

Pulsus paradoxus, hypotension, cardiogenic shock, and paradoxical movement of the jugular venous pulse are also signs to be noted.

This patient has pericardial tamponade and is unlikely to improve with medical management.

Pericardial drainage is recommended in all patients with large effusions because of recurrence rates in the 40%–70% range.

Administration of a fluid bolus is an appropriate measure but likely to be temporary.

For immediate decompression, one can perform bedside pericardiocentesis.

Definitive treatment of persistent symptomatic pericardial effusions is surgical pericardiectomy.

How well did you know this?
1
Not at all
2
3
4
5
Perfectly
43
Q

Which of the following treatments of a hypotensive patient is correct?

A. Pericardiocentesis in a 54-year-old man after myocardial infarction (MI) with adequate volume status and hypotension refractory to inotropic agents

B. Cardiac pacing in a 73-year-old woman taking digitalis with atrial fibrillation on the ECG, a ventricular response rate of 40, and an adequate volume status

C. Intraaortic balloon pump (IABP) in a 47-year-old woman with sepsis from pyelonephritis, good volume, and an echocardiogram showing no mechanical defects

D. Inotropic agents in a 68-year-old woman with metastatic breast cancer, distended neck veins, and PA catheter readings showing normalization of right and left heart pressure

E. Clamping of the infrarenal aorta in a patient with a gunshot wound to the chest and low right and left atrial pressure

A

ANSWER: B

COMMENTS: Cardiogenic shock most commonly occurs as a consequence of acute left ventricular infarction.

However, it may also be due to right ventricular infarction, ruptured papillary muscle, ruptured ventricular wall, acute aortic valvular insufficiency, mitral regurgitation, and a ventricular septal defect.

However, before assuming that the hypotension is caused by a cardiogenic mechanism, one must be sure that there is adequate blood volume.

Therefore a patient who is hypotensive with low right and left atrial pressure should undergo fluid administration as the initial management.

If cardiac performance improves with fluid administration alone, cardiogenic shock is probably not present.

If adequate filling pressures are attained and the hypotension persists in the absence of mechanical defects, arrhythmia, and sepsis, a primary pump problem probably exists and should be managed with inotropic agents.

One form of cardiogenic shock is cardiac tamponade, which is seen in traumatized patients, postoperative cardiac patients, and those suffering from uremia and certain malignancies.

Pericardial tamponade has a trend toward equalization of pressures in the right and left sides of the heart.

In a patient who is overdigitalized or hypokalemic, a very low ventricular rate in response to atrial fibrillation or flutter may result in hypotension and should be managed with cardiac pacing.

If a patient remains in cardiogenic shock despite adequate blood volume, appropriate HR, absence of a mechanical or valvular defect, appropriate administration of inotropic agents, and restoration of pressure and coronary blood flow, support via IABP counterpulsation may be needed.

IABP counterpulsation is most beneficial in patients with severe left ventricular dysfunction.

It assists in left ventricular systolic unloading by directly reducing stroke work, which in turn reduces myocardial oxygen consumption during the cardiac cycle, and in diastolic augmentation, which raises arterial BP and provides better coronary arterial perfusion during diastole and improved delivery of oxygen to the myocardium.

Patients with hemodynamic compromise secondary to right ventricular MI require fluid resuscitation and inotropic support.

Any preload reducers must be avoided.

Afterload reducers in the presence of hypotension are not warranted.

How well did you know this?
1
Not at all
2
3
4
5
Perfectly
44
Q

The alarm on the cardiac monitor continues to go off on a 73-year-old man with CHF and diabetes mellitus who was recently transferred to the ICU. He appears calm and is sitting up in bed watching a baseball game. His vital signs are an HR of 155 beats/min, BP of 125/84 mmHg, RR of 18 breaths/min, and Sao2 of 96%. An ECG taken 7 days ago is normal. The most recent one, taken 24 h previously, shows that his previously distinct P waves have been replaced with rapid, polymorphic, irregular P waves that are irregular and occurring at a rate greater than 300/min. The ECG is repeated and confirms the presence of an arrhythmia. At this point, which is the best initial intervention for this patient?

A. Anticoagulation with a
heparin drip

B. Cardioversion with paddles and settings at 260 J up to three times

C. Repeat ECG in 48 h

D. Restoration of sinus rhythm by pharmacologic means such as amiodarone or diltiazem

E. Morphine, 4 mg by IV push, to alleviate the pain

A

ANSWER: D

COMMENTS: The most common sustained dysrhythmia is atrial fibrillation, which has a prevalence of 5% in persons older than 65 years.

There are numerous causes that may trigger new-onset atrial fibrillation, including ischemia, MI, hypertension, electrolyte imbalance, pulmonary embolism (PE), and digoxin toxicity.

Initially, an ECG should be obtained, and if the arrhythmia is symptomatic, it should be treated aggressively.

New-onset atrial fibrillation with a duration of less than 48 h is a clear indication to restore sinus rhythm by either electrical or pharmacologic means.

This can be performed with IV calcium channel blockers, amioda- rone, or β-blockers, which are usually effective in rapid conversion.

Cardioversion should be performed in patients who are hemodynamically instable.

Cardioversion in patients with atrial fibrillation for longer than 48 h is contraindicated until they are fully anticoagulated.

Acute intervention may not be necessary in patients with a history of well-tolerated arrhythmia.

How well did you know this?
1
Not at all
2
3
4
5
Perfectly
45
Q

A 58-year-old woman is found to have meningococcemia and sepsis. On examination, she is confused, agitated, and in respiratory distress. She is intubated and placed on assist- control (AC) ventilation. A central line is placed and several fluid boluses are given but she is still hemodynamically unstable. A continuous drip of a vasoactive drug is started. After administration, her HR remains at 105 beats/min, MAP rises to 70 from 45 mmHg, cardiac output (CO) drops to 2.8 from 3.3 L/min, and systemic vascular resistance increases to 1150 from 500 dynes⋅s/cm5. Based on the changes observed, which drug was most likely administered?

A. Dobutamine

B. Dopamine

C. Phenylephrine

D. Epinephrine

E. Milrinone

A

ANSWER: C

COMMENTS: Inotropic agents increase cardiac contractility by increasing the concentration and availability of intracellular calcium.

Catecholamines act by binding to adrenergic receptors. Each type of receptor controls a particular cardiovascular function (Table 10.1).

Epinephrine, norepinephrine, dopamine, and dobutamine are all catecholamines.

The α1-receptor mediates arterial vasoconstriction by causing contraction of the vascular smooth muscle, and the α2-receptor induces constriction of venous capacitance vessels.

The β1-receptor stimulates myocardial contractility, and the β2-receptor causes relaxation of bronchial smooth muscle and relaxation of vascular smooth muscle in skeletal muscle beds.

The dopamine receptors cause relaxation of the vascular smooth muscle. The dopamine-1 receptor induces relaxation of renal and splanchnic vascular smooth muscle, and the dopamine-2 receptor inhibits the uptake of norepinephrine at the sympathetic nerve terminal, which results in a prolonged action of norepinephrine at the motor end plate.

The effects of dopamine are unpredictable, and the side effects might be significant. Thus its use in the ICU has been ebbing.

The response to catecholamines in normal individuals is different from that in critically ill patients. Receptor populations change over short periods, and upregulation and downregulation can occur depending on the disease state.

It is important that catecholamines be administered for a predetermined effect. If the effect is not attained with the particular catecholamine chosen, the dose should be adjusted or another agent used.

How well did you know this?
1
Not at all
2
3
4
5
Perfectly
46
Q

You are consulted for a long-term enteral feeding access in a patient with a recent stroke. Despite speech therapy, he is unable to maintain adequate nutrition with by-mouth (PO) intake alone. Which of the following is an absolute contraindication to percutaneous endoscopic gastrostomy (PEG) placement?

A. Active infection

B. Hepatitis with large ascites

C. History of abdominal surgery

D. Peptic ulcer disease

E. Expected survival of less
than 6 months

A

ANSWER: B

COMMENTS: Placement of a PEG tube is a viable and safe option for many critically ill patients or those who are unable to maintain adequate PO intake.

There are few contraindications and complications when performed on a properly selected patient population.

The technique of PEG placement relies on the principles of transillumination of the entry site with a lighted gastroscope and 1:1 palpation.

These steps are critical to ensure the stomach and anterior abdominal wall are sufficiently opposed, and there are no other organs (i.e., transverse colon) that may be injured during tube insertion.

Contraindications to PEG include irreversible coagulopathy, hemodynamic instability, oropharyngeal or esophageal mass unable to be passed by a gastroscope, a history of total gastrectomy, massive ascites, or portal hypertension with varices.

There are reports of successful PEG placement with small-to-moderate ascites and the concurrent use of paracentesis; however, the complication rate in this population is much higher suggesting the risks of the procedure outweigh the benefits.

How well did you know this?
1
Not at all
2
3
4
5
Perfectly
47
Q

Which of the following conditions is not usually associated with elevated dead space ventilation?

A. 42-year-old female after MI with CHF and a CO of 1.5 L/min

B. 28-year-old woman on post partum [?] day 1 with
shortness of breath, a Pao of 60 mmHg, and segmental 2 clots bilaterally in the pulmonary arteries

C. 52-year-old Hispanic immigrant with a long-standing ventricular septal defect and PA pressure of 80/52 mmHg

D. 22-year-old man after multiple gunshot wounds, massive transfusions, and a mean arterial to inspired oxygen ratio (Pao2/FiO2) of 180

E. 62-year-old woman smoker with the following ventilator settings: controlled mandatory ventilation at a rate of 12 breaths/min, FiO2 of 60%, Vt of 600 mL, and positive end-expiratory pressure (PEEP) of 5 cm H2O

A

ANSWER: E

COMMENTS: The most common causes of increased dead space in critically ill patients are decreased CO, PE, pulmonary hypertension, acute respiratory distress syndrome (ARDS), and excessive PEEP, all of which directly cause decreased blood flow to the pulmonary vasculature.

In dead space ventilation with a high ventilation/perfusion (V/Q) ratio, there is decreased blood flow to ventilated areas, which primarily affects the elimination of carbon dioxide.

In ARDS, some areas of the lung are perfused but not ventilated. Alveoli may be filled with secretions, exudate, blood, or edema, thereby increasing the shunt fraction. Other areas of the lung may be ventilated but not perfused, which accounts for the dead space ventilation.

PEEP can cause dead space ventilation by decreasing CO and stenting alveoli open, which causes the surrounding capillaries to collapse and thereby decreases alveolar perfusion.

Carbon dioxide production and the dead space–tidal volume ratio (Vds/Vt) determine minute ventilation.

The anatomic dead space includes the volume of the airways to the level of the bronchiole (150 mL).

Dead space can also include alveoli that are well ventilated but poorly perfused.

When combined, the anatomic and alveolar dead space constitutes the physiologic dead space, which is essentially the volume of gas moved during each tidal breath that does not participate in gas exchange.

How well did you know this?
1
Not at all
2
3
4
5
Perfectly
48
Q

A 17-year-old female with asthma is brought to the OR for ruptured ectopic pregnancy. Postoperatively, she is dyspneic and in acute respiratory failure. She is intubated and transferred to the surgical ICU where her ventilator is set to AC mode, RR of 18 breaths/min, FiO2 of 0.80, Vt of 600 mL, and PEEP of 0 mmHg. She was sedated and paralyzed for the intubation and is not breathing over the ventilator settings. After examining the patient and the flow pattern on the ventilator, changes in the settings are made. A change in which ventilator setting would best limit the intrinsic PEEP?

A. Increase in Vt

B. Decrease in the inspiratory flow rate

C. Increase in PEEP

D. Decrease in RR

E. Change from AC mode to synchronized intermittent mandatory ventilation (SIMV)

A

ANSWER: D

COMMENTS: Intrinsic PEEP (commonly known as auto-PEEP) is a state at end exhalation in which there is incomplete gas emptying, which can elevate alveolar volume and pressure.

It is the threshold pressure needed to be overcome to initiate inspiratory flow.

Severe bronchospasm increases the expiratory time needed, and patients in status asthmaticus or severe chronic obstructive pulmonary disease (COPD) are at risk for intrinsic PEEP.

If combined with narrowed airways, such as in asthma, and parenchymal noncompliance, the inspiratory work of breathing is increased.

Therefore there is an imbalance of respiratory muscle strength and work of breathing leading to respiratory failure.

During mechanical ventilation, when the expiratory time is insufficient to allow full exhalation of a ventilator breath, expiratory flow is still occurring when the next ventilator breath is delivered.

To best limit intrinsic PEEP, one can decrease the RR, thereby giving the patient more time to exhale between breaths. In addition, decreasing Vt will allow minimal improvement.

One should also limit the inspiratory time to leave more time in the respiratory cycle for exhalation.

Avoidance of hyperinflation and overdistention at the expense of minute ventilation, otherwise known as permissive hypercapnia, is an important method of ventilatory management in asthmatics.

How well did you know this?
1
Not at all
2
3
4
5
Perfectly
49
Q

A 65-year-old male who is 7 days out from a three-vessel coronary artery bypass remains intubated for respiratory failure despite normal hemodynamics. He is sedated with propofol and failed an SBT earlier this morning. The nurse asks if you would like to hold his sedation. Which of the following is true with regard to daily sedation awakening trials (SATs)?

A. The risk of self-extubation is too high to justify SAT after failed SBT.

B. SAT performed daily decreases overall sedation requirements for ventilated patients.

C. SAT alone is associated with lower rates of delirium.

D. Daily paired SAT and SBT are associated with fewer ventilator days and decreased ICU length of stay.

E. When part of an ICU protocol, daily SAT is associated with decreased overall mortality.

A

ANSWER: D

COMMENTS: The concept of daily interruption in sedation has been widely adopted by ICUs despite little convincing evidence of its independent efficacy.

A number of studies have demonstrated improved time-to-extubation with combined SAT and SBT versus SBT alone.

However, SAT has been shown to be equivalent to no SAT in many more recent trials.

In these studies, there was no decrease in time-to-extubation, sedation requirement, and ICU stay or mortality.

In line with no change in sedation requirements, there also has been no proven improvement in the incidence of ICU delirium with routine SAT.

SAT can be associated with adverse events such as patient removal of the endotracheal tube or other lines; however, when performed in an appropriate setting, there was also no significant increase in these adverse events.

How well did you know this?
1
Not at all
2
3
4
5
Perfectly
50
Q

With regard to ventilatory mechanics, which of the following statements is true?

A. The work of breathing at rest consumes 10% of total-body oxygen consumption (Vo2).

B. COPD is associated with an increase in the work of breathing as a result of increased expiratory work.

C. The work of breathing may increase to 75% of the total-body Vo2 in postoperative patients.

D. Airway pressure reflects the compliance of only the lungs.

E. Compliance is measured as the change in pressure divided by the change in volume.

A

ANSWER: B

COMMENTS: For patients with COPD, the work of breathing is increased because of increased expiratory work, not inspiratory work.

It can be assessed by preoperative pulmonary function testing and optimized by preoperative chest physical therapy, bronchodilators, and antibiotics if infection is present.

The work of breathing at rest consumes 2% of the total-body Vo2 and can be markedly increased, up to 50% of total Vo2, in postoperative patients because of increased airway resistance and decreased compliance of the lung, chest wall, and diaphragm.

The proper use of volume-cycled ventilators and pressure support (PS) ventilation can take over most of the work of breathing during the postoperative period.

Compliance is defined as the change in pressure associated with each milliliter increase in the lung volume.

Measuring airway pressure reflects the compliance of the chest wall and diaphragm, as well as that of the lungs.

In relaxed patients this is of little importance, but in restless patients, intraesophageal or intrapleural pressure pro- vides a more accurate measure of compliance.

In acute respiratory failure, decreased compliance is usually associated with decreased functional residual capacity.

Less compliant lungs need ventilatory management that maintains inflation of alveoli by the use of PEEP and recruits closed alveoli by elevating peak inspiratory pressure.

However, because positive airway pressure may overdistend already-ventilated alveoli, the peak inspiratory pressure should be kept below 40 cm H2O.

How well did you know this?
1
Not at all
2
3
4
5
Perfectly
51
Q

A 53-year-old man with chronic kidney disease, severe COPD, and systolic heart failure after MI is in your ICU. He is unable to tolerate the Trendelenburg position during the placement of a central line in the right internal jugular vein. Immediately after placement, he becomes diaphoretic and complains of difficulty in breathing.
He then becomes obtunded, tachycardic, and progressively tachypneic. His BP is 80/50 mmHg, and he has bilateral breath sounds by auscultation. Which of the following should be performed immediately?

A. Left lateral decubitus and Trendelenburg position, then aspiration from the central line

B. Heparinization

C. Removal of the central line

D. Fluid bolus

E. Dobutamine

A

ANSWER: A

COMMENTS: Though uncommon, air embolism is a potentially lethal complication of the central line placement.

Providers must be familiar with its presentation and immediate, lifesaving treatment maneuvers.

Presentation depends upon the exact volume of air that enters the venous system.

The lethal dose for humans has been estimated at 3 to 5 mL/kg/min; however, cardiovascular compromise can be seen with rates less than 1.5 mL/kg per min.

The incidence of air embolism with central line placement is reported as between 0.2% and 1%.

Physical examination for a significant air embolism will demonstrate hemodynamic instability and, often, hypoxia, along with a “mill-wheel” murmur in the precordium.

Immediate intervention is Durant’s maneuver—left lateral decubitus and Trendelenburg position in order to entrap air in the apex of the ventricle.

Aspiration via central venous catheter may then allow removal of some amount of the air in the system.

How well did you know this?
1
Not at all
2
3
4
5
Perfectly
52
Q

A family meeting is called for a 69-year-old man who was intubated 6 days earlier for pneumonia and respiratory distress. He is now awake, alert, and asking for removal of the tube. His family wants to know when and whether he will be extubated. Which of the following characteristics of this patient DOES NOT meet conventional weaning criteria?

A. Negative inspiratory force of −10 cm H2O

B. A respiratory frequency/tidal volume (RF/Vt) ratio of 105 or less

C. Correction of underlying pulmonary and nonpulmonary complications

D. Pulse oximetry reading of 92%

E. Vital capacity of 12 to 15 mL/kg and peak inspiratory pressure of less than 25 cm H2O

A

ANSWER: A

COMMENTS: Many indices have been proposed to predict weaning outcome and success or failure of extubation.

Most surgical patients (90%) are weaned from mechanical ventilation in less than 1 week.

Conventional weaning criteria include:

(1) measurements of oxygenation with a pulse oximeter (best determined by arterial blood gas analysis, with an Sao2 > 90% and any FiO2 usually being adequate for weaning) and
(2) measurements of ventilation, such as an RR less than 24 breaths/min, Paco2 less than 50 mmHg, peak inspiratory pressure below 30 cm H2O, Vt of at least 5 to 8 mL/kg, and a vital capacity double the Vt value.

Failure to satisfy these conventional criteria is associated with unsuccessful weaning in as many as 63% of patients.

The rapid, shallow breathing test (RF/Vt) is performed by having the patient breathe room air for 1 min.

When RF/Vt is 105 or less, successful weaning occurs in 78% of patients, and when RF/ Vt is less than 80, the success rate is 95%.

Conversely, an RF/Vt value of 105 or higher is accompanied by a failure rate of 95%.

Another method often described is the Simple Object Access Protocol (SOAP) assessment:

(1) ability to clear secretions,
(2) adequate oxygenation (Pao2/FiO2 ratio > 200 mmHg, which requires an FiO2 of 0.4 to 0.5 and PEEP < 8 cm H2O),
(3) ability to protect the airway, and
(4) adequate pulmonary function.

Clinical judgment and correction of underlying pulmonary and nonpulmonary complications continue to be the best guide to successful weaning.

In addition, helpful ventilation scores include an FiO2 of less than 40%, continuous positive airway pressure (CPAP) of 3 cm H2O, effective static compliance greater than 50 mL/cm H O, dynamic compliance greater than 40 mL/cm H2O, ventilator minute ventilation of less than 10 L/min, and a triggered ventilatory rate of less than 20 breaths/min.

The duration of ventilatory support is not correlated with survival rates at discharge. Forty-one percent of long-term ventilated patients survive.

Because muscle atrophy is often present, a progressive ventilatory withdrawal plan designed to restore muscle function should be used.

Intermittent mandatory ventilation, PS ventilation, and weaning by T-piece have been used effectively.

How well did you know this?
1
Not at all
2
3
4
5
Perfectly
53
Q

A 29-year-old firefighter is intubated in the ICU after being exposed to smoke. She has thick yellow secretions that require frequent suctioning along with the administration of bronchodilators. On hospital day 5, she has a percutaneous central venous catheter placed through the right internal jugular vein. Several hours later, she undergoes respiratory arrest. Her peak inspiratory pressure has risen from 24 to 41 cm H2O, and her plateau pressure has stayed at 16 cm H2O. Choose which of the following is the most likely reason for the respiratory arrest:

A. Tension pneumothorax

B. Flash pulmonary edema

C. Pulmonary embolism (PE)

D. Endotracheal tube obstruction

E. Auto-PEEP with breath stacking

A

ANSWER: D

COMMENTS: This patient has an obstruction of the endotracheal tube.

The key to identifying this problem is recognizing the components of the patient’s respiratory pressure as shown in Table 10.2, most importantly the peak inspiratory and plateau pressures.

The peak inspiratory pressure is the pressure required to overcome the resistance in the endotracheal tube and airways, as well as the compliance of the airways.

The inspiratory plateau pressure is the pres- sure generated to overcome the elastance of the lung parenchyma, pleural space, and chest wall.

This patient had increasing peak inspiratory pressure, so her problem was related to the tube, not the lung itself.

Tension pneumothorax and flash pulmonary edema are associated with increases in both peak inspiratory pressure and inspiratory plateau pressure.

PE also does not change the inspiratory pressure.

How well did you know this?
1
Not at all
2
3
4
5
Perfectly
54
Q

A 73-year-old woman weighing 60 kg is admitted to the hospital with acute pancreatitis.

She is aggressively resuscitated with fluid but becomes hypotensive and has increasing work of breathing and O2 requirements within the next 12 h.

The patient is transferred to the ICU and intubated. A PA catheter is placed, and the wedge pressure is 8 cm H2O. Arterial blood gas analysis shows values of a pH of 7.36, a Pao2 of 62, a Pco2 of 42, a serum bicarbonate of 21, and a base deficit of −2 with Sao2 of 90%.

Which of the following ventilation strategies is most appropriate for this patient?

A. Pressure-control ventilation (PCV) with a pressure of 40 cm H2O and an inverse ratio ventilation of 3:1

B. SIMV with a Vt of 720mL and an RR set to keep the pH at 7.4

C. AC ventilation with a Vt of 600 mL and prone positioning

D. AC ventilation with a Vt of 360 mL and an RR to keep the pH above 7.2

E. SIMV with a Vt of 600 and an FiO2 of 100%

A

D

How well did you know this?
1
Not at all
2
3
4
5
Perfectly
55
Q

Which one of the following criteria is included in the
definition of ARDS?

A. Onset within 1 week of a clinical insult

B. Chest radiograph showing pulmonary infiltrate (unilateral or bilateral)

C. Hypoxemia with Pao2 to FiO2 ratio < 200 mmHg

D. Normal PA wedge pressure

E. Infectious origin

A

ANSWER: A

COMMENTS: Because the principal physiologic problem in ARDS is hypoxemia refractory to increasing FiO2, therapy is centered on the provision of mechanical ventilation to maximize oxygen delivery while minimizing lung injury.

PEEP is used to improve oxygenation and lung compliance and should be optimized with the help of pressure–volume curves to facilitate the maintenance of open alveoli and diffusion of oxygen into the pulmonary capillaries.

For a given FiO2, Pao2 usually increases on the administration of PEEP in patients with ARDS.

However, excessive PEEP (>15 cm H2O) can be hazardous and lead to pneumothorax from barotrauma and decreased venous return to the heart.

Overdistention of alveoli can be prevented by keeping the peak inspiratory pressure below 35 cm H2O.

Newer ventilatory methods attempt to enhance alveolar recruitment, maintain alveolar patency throughout the respiratory cycle, maintain an Sao2 of greater than 90%, avoid dynamic hyperinflation (volu-trauma), and reduce the risk for oxygen toxicity.

Spontaneous, augmented low-volume ventilation, with PS ventilation being used as a primary ventilatory support mode, directs flow to regions of low ventilation/perfusion.

Diuretics in cases of obvious fluid overload and cardiac decompensation and broad-spectrum antibiotics in cases of established pulmonary infection or other sources of sepsis may be useful for patients with ARDS.

The consensus conference on ARDS (ARDSnet.org) showed that a volume-restricted ventilation strategy reduced mortality.

In this well-accepted study, 861 patients were randomly assigned to either a traditional-volume ventilation strategy (12 mL/kg of ideal body weight with plateau pressures < 50 cm H2O) or low-Vt ventilation (6 mL/kg with plateau pressures < 30 cm H2O).

The study was halted early because of significantly reduced overall mortality (31.0% vs. 39.1%, P < .0007).

Permissive hypercapnia was allowed, and sodium bicarbonate was given to maintain pH higher than 7.2.
The patient developed ARDS, probably because of acute pancreatitis and systemic inflammatory response syndrome (SIRS).

Criteria used to define ARDS have been recently revised. The Berlin definition developed in 2013 now stratifies ARDS based upon the degree of hypoxemia.

All types of ARDS include an acute onset within 1 week of clinical insult, bilateral pulmonary infiltrates on chest radiographs, and some degree of hypoxemia not fully explained by cardiac failure.

There does not have to be an infectious process for a patient to have ARDS, and no longer are patients with cardiac failure excluded from having coexisting ARDS.

ARDS and acute lung injury have now been combined and categorized based on the severity of the Pao2/FiO2 ratio: mild (200 to 300), moderate (100 to 200), and severe (<100).

The lung response can be divided into an exudative phase (24 to 96 h), with leakage of proteinaceous fluid into the pulmonary interstitium and corresponding damage to the alveolar–capillary interface; an early proliferative phase (3 to 10 days), with proliferation of alveolar type II cells, cellular infiltration of the septum, and organization of hyaline membranes; and a late proliferative phase (7 to 10 days), with fibrosis of the alveolar septum, ducts, and hyaline membranes.

Frequently, the radiographic changes can lag behind the clinical picture in ARDS considerably.

How well did you know this?
1
Not at all
2
3
4
5
Perfectly
56
Q

A 59-year-old woman with a long-standing history of gastroesophageal reflux disease (GERD) underwent a Nissen fundoplication that was complicated by 2 L of blood loss and hypotension in the OR. Her vital signs are an HR of 103 beats/min, a BP of 100/70 mmHg, an RR of 16 breaths/min, and an Sao2 of 96%. Her urine output was 15 mL of urine/h over the last 4 h. Laboratory results include a urine osmolality of 600 mOsm/kg, urine sodium concentration of 15 mEq/L, plasma sodium concentration of 140 mEq/L, urine creatinine concentration of 20 mg/dL, and plasma creatinine concentration of 1.5 mg/dL.

What is the next step in management?

A. Flushing the Foley catheter with 60 mL of normal saline

B. Hemodialysis

C. Nephrology consultation

D. Decompressive laparotomy for abdominal compartment syndrome (ACS)

E. Administration of a 1000-mL fluid bolus of normal saline as a fluid challenge

A

ANSWER: E

COMMENTS: Acute renal failure is a serious morbidity for post- surgical patients, with mortality rates greater than 50%.

Renal failure can be prerenal, renal, or postrenal.

The most common cause in surgical patients is hypovolemia, as is the case in this patient from blood loss in the OR.

Some indicators for prerenal causes include urine osmolality greater than 500 mOsm/kg, fractional excretion of sodium (FENa) of less than 1%, and urine sodium concentration of less than 20 mEq/L, whereas an FENa greater than 3% and urinary sodium concentration greater than 40 mEq/L are indicative of parenchymal or postrenal causes.

Medications, IV contrast material–induced nephropathy, rhabdomyolysis, and transfusion reactions are all options to consider.

This patient’s FENa is 0.8%.

How well did you know this?
1
Not at all
2
3
4
5
Perfectly
57
Q

A 46-year-old brittle diabetic and hypertensive woman is brought to the ICU after being found unresponsive in her bed. After undergoing a CT scan of her head, abdomen, and pelvis with IV contrast media, she is transferred to the ICU. The ICU team places a central line, orders an echocardio- gram, and places a bladder catheter. Her urine output has been approximately 10 mL/h for the last 4 h. Her FENa is calculated to be 2.4%. Which of the following is not consistent with acute tubular necrosis (ATN)?

A. Oliguria

B. FENa greater than 2%

C. Urine osmolality of 200 mOsm/kg

D. Creatinine clearance greater than 125 mL/min

E. Sodium wasting

A

ANSWER: D

COMMENTS: A creatinine clearance of 125 mL/min represents a normal renal function.

ATN is characterized by a rise in plasma creatinine concentration [decrease in creatinine clearance or glomerular filtration rate (GFR)], a urine volume that is reduced (oliguric) or normal, changes in the findings on urinalysis, and a FENa greater than 1%–2%.

Oliguria, or urine output less than 500 mL/24 h, is a frequent but not an absolute feature of ATN.

Whether oliguria occurs may depend on the severity of the renal injury or the relative reabsorption of filtrate at the tubular level.

Even if a patient’s GFR falls to 10 L/day (normal, 180 L/day), a urine output of 1 to 2 L/ day would still be normal as long as 8 to 9 L of filtrate was reabsorbed.

In cases of well-preserved tubular function, as in prerenal forms of acute kidney injury (ARF), FENa is low, consistent with the sodium-avid state.

As tubular dysfunction progresses, the ability of nephrons to reabsorb sodium is disrupted, and a greater percentage of the filtered sodium is excreted in urine.

As a result, FENa will be greater than 1%–2% because of inappropriate sodium wasting by altered tubular function.

Loss of urinary concentrating ability is an early feature of ATN.

A urine osmolality of less than 350 mOsm/L is consistent with ATN, whereas an osmolality greater than 500 mOsm/L suggests a prerenal cause of ARF.

However, lower values can be seen during prerenal ARF thus limiting the value of this test as a sole indicator of tubular function.

How well did you know this?
1
Not at all
2
3
4
5
Perfectly
58
Q

A 62-year-old man with peripheral vascular disease, diabetes, and bilateral tissue loss in the lower extremities is admitted for angiography of his lower extremities. He has chronic renal failure and his serum creatinine level is 5.0 mg/dL, which has been his baseline for the last 3 years. Which of the following agents is indicated to reduce the risk for IV contrast–induced nephropathy?

A. Calcium channel blocker

B. Aggressive diuresis

C. Saline volume expansion before and after the procedure

D. Acetylcysteine given only after exposure to contrast material

E. Mannitol and saline hydration

A

ANSWER: C

COMMENTS: In most cases, radiocontrast agents can lead to a reversible form of ARF.

The pathogenesis is not well established, but two proposed mechanisms of injury are renal vasoconstriction and direct tubular toxic effects.

The risk is minimal in patients with normal renal function, including those with diabetes, and the renal failure is nonoliguric and transient in most cases.

Severe renal failure requiring short- or long-term dialysis is rare and most likely to occur in patients whose baseline creatinine level is greater than 4 mg/dL. Risk factors for the development of contrast-induced nephropathy include underlying chronic renal failure with a plasma creatinine level greater than 1.5 mg/dL, diabetic nephropathy with renal insufficiency, CHF, multiple myeloma, and a large volume of contrast material.

Saline volume expansion in the precontrast and postcontrast periods is the only preventive measure consistently shown to be of benefit.

Hydration with furosemide may increase the risk for contrast-induced nephropathy when compared with saline alone.

Furthermore, the use of saline solution and mannitol does not have any benefit over the use of saline alone.

Calcium channel blockers given to minimize renal vasoconstriction after exposure to contrast media have not been conclusively shown to prevent renal failure.

The role of nonionic contrast agents is not clearly defined. Studies seem to support the use of isosmolar non-ionic agents in high-risk patients, especially those with diabetes.

There are conflicting data on the role of acetylcysteine and sodium bicarbonate infusions in the prevention of contrast-induced nephropathy, but given its relatively safe side-effect profile and the few series supporting its use, use of both can be justified, particularly in high-risk patients.

A rational approach to preventing con- trast-induced nephropathy in high-risk patients, such as the patient in question, would include acetylcysteine (600 mg orally twice daily the day before and on the day of exposure to contrast mate- rial), saline volume expansion before and after the procedure, and an isosmolar nonionic contrast agent. Several recent meta-analyses have shown that sodium bicarbonate infusion can decrease the damage associated with contrast-induced nephropathy if given both before and after the procedure as well.

The former may be more important for patients with renal dysfunction and diabetes.

How well did you know this?
1
Not at all
2
3
4
5
Perfectly
59
Q

Choose the situation that does not require immediate renal replacement therapy.

A. A 27-year-old bipolar patient, after running a half marathon, taking a prescribed lithium dose and found to have ataxia, confusion, and inverted T waves

B. A 68-year-old man after sigmoid colectomy with new-onset seizures and blood urea nitrogen (BUN) of 150 mg/dL

C. A 58-year-old man after a motor vehicle collision with multiple long-bone fractures, BUN of 120 mg/dL, creatinine of 2.8 mg/dL, and diffuse bleeding

D. A 71-year-old woman with diabetes maintained on an insulin drip after total abdominal hysterectomy and bilateral salpingo-oophorectomy with an FENa of 0.7% and urine output less than 20 mL/h for last 7 h

E. A 45-year-old man with respiratory distress after massive resuscitation for a septic episode, bilateral lung haziness on chest radiography, and coarse crackles who is unresponsive to diuretics

A

ANSWER: D

COMMENTS: Indications for acute dialysis treatment include:

(1) persistent hyperkalemia refractory to medical management;
(2) pulmonary edema unresponsive to conventional therapy;
(3) severe acidemia;
(4) symptoms of uremia such as anorexia, nausea, and vomiting;
(5) uremic encephalopathy, seizures, asterixis, uremic pericarditis, and uremic bleeding; and
(6) overdose with a dialyzable toxin such as lithium or ethylene glycol. Renal replacement therapy is needed in 1%–2% of patients with ARF, and as many as 15% of patients may ultimately require dialysis at some point in their life.

It may be indicated for symptomatic fluid overload, sepsis, uremic complications, and severe electrolyte or acid-base disorders.

Frequently in the ICU, continuous renal replacement therapy is superior to intermittent hemodialysis or peritoneal dialysis, but in the United States, it is only used in 10%–20% of ICU patients.

Proponents of this method over others argue that it allows better hemodynamic stability and prevention of shifts in intracerebral water, minimizes the risk for infection, and provides continuous control of fluid status and acid-base abnormalities.

Complications include the need for anticoagulation and a high level of nursing care.

How well did you know this?
1
Not at all
2
3
4
5
Perfectly
60
Q

A 68-year-old woman with a history of GERD, cholelithiasis, and coronary artery disease is seen in the ED with nausea, vomiting, and epigastric pain. Laboratory tests showed amylase and lipase values of 259 and 1782 units/L, leukocy- tosis of 18,300/mm3, and a prothrombin time (PT) and international normalized ratio (INR) of 47 s and 1.9, respectively. The patient received 6 L of crystalloid solution because of hypotension and required intubation. After 48 h, the hemoglobin has dropped by 2 g. What are the factors that have the strongest correlation with stress-related bleeding in critically ill patients?

A. Mechanical ventilation and hypotension

B. Coagulopathy and renal failure

C. Steroids and sepsis

D. Mechanical ventilation and steroids

E. Mechanical ventilation and coagulopathy

A

ANSWER: E

COMMENTS: Risk factors for stress-related mucosal lesions are mechanical ventilation longer than 48 h, coagulopathy, significant burns, and head injury.

These lesions have been found in 25%– 100% of ICU patients within 48 h of admission, but clinically significant bleeding occurs in only 5%–10%.

Patients with risk factors should receive prophylaxis until consuming an enteral diet of at least 50% of their caloric intake.

How well did you know this?
1
Not at all
2
3
4
5
Perfectly
61
Q

You are caring for a 55-year-old male who is in the ICU 5 days after multiple blunt traumatic injuries. He was intubated in the OR for the repair of a femur fracture on hospital day 1 and has not yet been extubated. He continues to fail his SBT with RSBI higher than 105. What is the most effective strategy to liberate this patient from the ventilator?

A. PS wean

B. AC ventilation with daily SBT

C. SIMV wean

D. Early tracheostomy before day 10

E. PCV with daily SBT

A

ANSWER: A

COMMENTS: The most efficient strategy to wean patients from the ventilator has been studied many times and remains somewhat controversial.

It is clear that the most effective way to identify those patients who are ready to have a trial of extubation is the daily SBT.

However, those who have failed repeated SBTs may require a more targeted and purposeful weaning technique in order to avoid the complications associated with the long-term mechanical ventilation in the ICU setting.

Early tracheostomy, when compared to late tracheostomy, does improve ICU length of stay and number of ventilator days; however, it has not shown direct benefit over other modes of weaning mechanical ventilation and will lead to a higher rate of tracheostomy.

When compared directly, SIMV weaning is not as effective as PS weaning, and there is some evidence that PS weaning improves diaphragmatic dysfunction over both AC and pressure-control modes.

Regardless of strategy, daily SBT and reassessment of readiness for extubation remain appropriate.

How well did you know this?
1
Not at all
2
3
4
5
Perfectly
62
Q

A 37-year-old woman comes to the ED complaining of a severe headache. She undergoes an emergency head CT scan, which shows subarachnoid hemorrhage; an angiogram identifies an arteriovenous malformation, which is subsequently embolized. Four days later, her serum sodium concentration is 122 mEq/L. Which is the most correct statement regarding the syndrome of inappropriate secretion of antidiuretic hormone (SIADH) and cerebral salt wasting (CSW)?

A. SIADH and CSW share the same underlying pathophysiology and cannot be reliably distinguished.

B. SIADH and CSW can be differentiated by measuring urine sodium and serum uric acid concentrations.

C. SIADH and CSW can be differentiated by measuring urine osmolality and sodium concentration.

D. Assessment of extracellular fluid volume will best differentiate between SIADH and CSW.

E. Regardless of the diagnosis, treatment of the hyponatremia is the same.

A

ANSWER: D

COMMENTS: Hyponatremia is common in the setting of central nervous system disease. Most often it results from inappropriate SIADH.

With SIADH, the hyponatremia initially results from ADH induced water retention. This volume expansion activates natriuretic mechanisms that induce the loss of sodium and water, with the patient typically being restored to a nearly euvolemic state.

With chronic SIADH, the loss of sodium (and often potassium) is much more significant than the water retention.

CSW is characterized by hyponatremia and loss of extracellular volume from inappropriate sodium wasting in urine. Patients with CSW meet the laboratory criteria for SIADH: hyponatremia, elevated urine osmolality (>100 mOsm/kg), elevated urine sodium concentration (>40 mEq/L), and low serum uric acid concentration.

However, they also have clinical evidence of hypovolemia (decreased skin turgor, elevated hematocrit, decreased weight, and hypotension) rather than the nearly euvolemic state seen with SIADH.

Furthermore, volume repletion with isotonic saline in patients with CSW will lead to a dilute urine (and eventual correction of the hyponatremia), whereas isotonic saline administration may worsen the hyponatremia of SIADH because the sodium is retained while the water is excreted.

SIADH is usually treated by fluid restriction; however, this must be done with caution in patients with SIADH because of the risk of hypotension and cerebral infarction. Iso- tonic saline may be used but requires careful monitoring of the serum sodium concentration; if a further fall in serum sodium occurs, a switch to hypertonic saline may be necessary.

CSW generally responds well to volume repletion with isotonic saline. Salt tablets and mineralocorticoids (such as fludrocortisones) may also be useful as adjunctive measures.

How well did you know this?
1
Not at all
2
3
4
5
Perfectly
63
Q

A patient is admitted to the surgical ICU after a prolonged laparoscopic cholecystectomy, which required conversion to an open procedure. The patient was reintubated for respira- tory distress in the recovery area. Upon arrival to the unit, you obtain an arterial blood gas: pH 7.46, Paco2 23, and Pao2 85. Which of the following is true?

A. The primary problem is metabolic with respiratory compensation.

B. No changes should be made to the ventilation as the kidneys will compensate for this abnormality.

C. This acid-base disturbance is consistent with septic shock.

D. Minute ventilation should be decreased.

E. PEEP should be increased.

A

ANSWER: D

COMMENTS: Arterial blood gas is commonly used to adjust ventilation parameters in order to optimize a patient’s respiratory function and maintain homeostasis.

This patient has a respiratory alkalosis with low Paco2 contributing to increased pH.

Pao2 is in the normal range and will maintain saturation of hemoglobin with oxygen.

Adding PEEP will increase the Pao2 and is unnecessary for this patient.

Decreasing the minute ventilation, however, either by changing the RR or Vt, will increase CO2 retention and thereby decrease the pH.

This patient’s primary disturbance is respiratory distress, and while the renal system does compensate for prolonged acid-base disturbance, it requires a much longer duration than do our respiratory compensatory mechanisms.

Sepsis is generally associated with metabolic or lactic acidosis.

How well did you know this?
1
Not at all
2
3
4
5
Perfectly
64
Q

An 80-kg, 65-year-old woman with severe lupus is admitted to the ICU after exploratory laparotomy for sigmoid diverticulitis (Hinchey type IV). She is given a stoma and brought to the ICU intubated. Her vital signs are a tempera- ture of 97.5°F, HR of 105 beats/min, BP of 70/50 mmHg, and Sao2 of 96%. In the first hour her urine output is 20 mL; she has received 4 L of crystalloid and one unit of packed red blood cells (PRBCs), and her antibiotics have been redosed. Her CVP is 10 mmHg, but she remains hypotensive.

Choose the next intervention that will be most beneficial?

A. Additional 2 L of a normal saline bolus

B. Hydrocortisone, 100 mg IV

C. Administration of furosemide for low urine output

D. Initiation of vasopressor therapy with norepinephrine or dopamine

E. Aggressive rewarming

A

ANSWER: D

COMMENTS: In a patient in septic shock who is adequately volume resuscitated (shown by a CVP of 10 mmHg) and is unresponsive to fluid challenges, vasopressor therapy should be started.

Administration of vasopressors can quickly restore BP; in the Sur- viving Sepsis Guidelines, norepinephrine administered centrally is the initial vasopressor of choice.

Epinephrine, phenylephrine, and vasopressin should not be administered as the initial vasopressor to patients in septic shock.

Epinephrine can be used as the first alternative agent in septic shock when BP is poorly responsive to norepinephrine.

Dobutamine should be used in patients with myocardial dysfunction as evidenced by elevated cardiac filling pressures and low CO.

In general, the rate of fluid administration should be reduced if cardiac filling pressures increase without concurrent hemodynamic improvement.

How well did you know this?
1
Not at all
2
3
4
5
Perfectly
65
Q

Norepinephrine therapy is started in septic patient. Later, vasopressin, 0.03 units/min, is added, but the patient remains hypotensive with a MAP below 55 mmHg. Her hemoglobin concentration is 9.0 g/dL. After performing an echocardiogram, dobutamine infusion was started at a maximum of 20 mcg/kg/min; pH is 7.21 with a Pco2 of 34 mmHg. The patient remains hypotensive. What is the next step?

A. Increase the cardiac index to predetermined supranormal levels.

B. Administer hydrocortisone, 100 mg IV.

C. Have a family discussion about withdrawing care.

D. Perform an adrenocorticotropic hormone (ACTH) stimulation test.

E. Switch the ventilatory mode to AC.

A

ANSWER: B

COMMENTS: Although there is still much debate in the critical care literature about steroids, IV corticosteroids are recommended in patients with septic shock who despite adequate fluid replace- ment require vasopressor therapy to maintain adequate BP.

Random cortisol levels may be helpful in determining a patient’s benefit from steroid therapy, although it is not required.

Consideration can be made to discontinue corticosteroid therapy in patients with a random cortisol level of greater than 25 mcg/dL.

An ACTH stimulation test is not recommended to identify the subset of patients with septic shock who should receive hydrocortisone.

How well did you know this?
1
Not at all
2
3
4
5
Perfectly
66
Q

A 70-kg, 33-year-old woman who had not seen a physician in 10 years arrives at the ED with symptoms of dyspnea, fatigue, weight gain, diplopia, and dysphagia following an urgent laparoscopic cholecystectomy 10 days ago. On examination, she is awake and alert. She is afebrile with an HR of 80 beats/min, BP of 120/70 mmHg, and RR of 29 breaths/min. Her heart sounds are normal and breaths are bilateral and shallow. She has ptosis and significant proximal muscle weakness in all extremities. She is drooling slightly and having difficulty swallowing. Her vital capacity is 500 mL and her laboratory tests are pending. Which of the following treatments is the most appropriate to initiate next?

A. Administration of pyridostigmine

B. Endotracheal intubation

C. Administration of steroids

D. Administration of IV immunoglobulin

E. Administration of levothyroxine

A

ANSWER: B

COMMENTS: This patient is having a myasthenic crisis, which is a consequence of an autoimmune attack on the acetylcholine receptor complex.

There is clinical weakness that is mostly marked after prolonged muscle exertion and should be considered in any patient with respiratory distress and cranial nerve findings.

The myasthenic crisis with respiratory failure develops in approximately 20% of patients and necessitates intubation.

It can be precipitated by bronchopulmonary infections, sepsis, surgical procedures, tapering of steroid medications, pregnancy, and some drugs.

Upper airway muscle weakness can lead to the collapse of the airways and aspiration.

Patients with marginal vital capacity (<15 mL/kg), weak cough or voice, and worsening negative inspiratory force should be considered for intubation.

How well did you know this?
1
Not at all
2
3
4
5
Perfectly
67
Q

A 73-year-old male with stage IV colon cancer is intubated in the ICU 5 days after undergoing palliative resection of his primary tumor. He becomes acutely febrile and tachycardic with increased oxygen requirement. His BP drops to 88/50 mmHg. A bedside echocardiogram demonstrates a hyperdynamic left ventricle and right ventricular strain. Which of the following is the optimal treatment for this patient?

A. Surgical embolectomy

B. Systemic tissue plasminogen activator (tPA)

C. Catheter-directed tPA

D. Initiation of therapeutic heparin drip

E. Placement of inferior vena cava (IVC) filter

A

ANSWER: C

COMMENTS: Therapeutic anticoagulation is the mainstay of treatment for confirmed or suspected PE.

The goal range for partial thromboplastin time (PTT) in the treatment of venous thromboembolism is 60 to 90s, and the drip should be titrated to this value.

Massive PE is defined as hypotension with systolic BP < 90 mmHg, while submassive PEs are those with systolic BP > 90 mmHg but with right heart strain or right ventricular dilation on echocardio- gram.

Thrombolysis is an option for many nonsurgical patients diagnosed with massive or submassive PE.

Contraindications to systemic tPA include recent head trauma, ischemic stroke (within 3 weeks), any prior intracranial hemorrhage, or neoplasm.

Relative contraindications include age > 75, recent surgery (within 3 to 4 weeks), pregnancy, or remote ischemic stroke.

For patients with massive or submassive PE who do not meet criteria for systemic tPA, a more recent consideration is catheter-directed tPA.

While more studies are ongoing, early randomized trials of catheter- directed tPA in postsurgical patients have demonstrated improve- ment in right ventricle (RV) function at 24 h without increase in bleeding complications.

How well did you know this?
1
Not at all
2
3
4
5
Perfectly
68
Q

Which of the following sites for central venous catheter placement is associated with the lowest rate of catheter-asso- ciated blood stream infections?

A. Internal jugular vein

B. Subclavian vein

C. Femoral vein

D. Both A and B are equivalent

E. All sites are equivalent if sterile technique is maintained

A

B

How well did you know this?
1
Not at all
2
3
4
5
Perfectly
69
Q

You are attempting to place a central venous catheter in the right internal jugular vein of a hemodialysis patient who is in the ICU after below-knee amputation for a severe diabetic foot infection. Which of the following is true regarding the central line placement?

A. The widespread use of ultrasound has been shown to reduce complication rates regardless of the site.

B. The risk of complication increases after three attempts by the same proceduralist.

C. If ultrasound is used, a chest radiograph is unnecessary prior to attempts on the contralateral side.

D. Routine exchange of catheters reduces infectious complications.

E. Aspiration of dark blood confirms appropriate venous placement.

A

ANSWER: B

COMMENTS: Nearly 15% of patients who receive central venous catheters will have an associated complication. While there is an obvious risk associated with their placement, many of these com- plications are related to catheter-associated infection or thrombosis.

Experience reduces the likelihood of complications with line placement, and it has been shown that the complication rate after three attempts is nearly six times that of the complication rate after only one attempt by any one proceduralist.

Ultrasound does reduce the risk of complication with internal jugular catheter placement but does not reliably improve outcomes with subclavian catheter placement.

Regardless of the use of ultrasound guidance, pneumothorax is a potential complication and does not obviate the need for a chest x-ray.

A recent meta-analysis demonstrated 4.5 infections per 1000 femoral venous catheter placements versus only 1.2 infections per 1000 subclavian venous catheter placements.

Likewise, subclavian catheters were also demonstrated to be associated with fewer bloodstream infections than internal jugular venous catheters.

How well did you know this?
1
Not at all
2
3
4
5
Perfectly
70
Q

A 66-year-old woman who has been in the ICU for 2 weeks following total hip replacement complicated by massive infection and sepsis is complaining of right calf pain. She gets out of bed to go to physical therapy and develops severe dyspnea, tachycardia, and hypotension. Pulse oximetry reading is 75%. A CT angiogram shows bilateral clots in the pulmonary arteries. With regard to PE, which of the following is true?

A. Early chest radiographic abnormalities are rarely present in patients with PE.

B. A shunt abnormality is present early after the PE and a V/Q abnormality becomes the mechanism for hypoxemia in later stages.

C. Thrombolytic therapy has been shown to reduce mortality rates in comparison to heparin in patients with PE.

D. Heparin should never be given until the diagnosis of PE is absolute.

E. More than 33% of patients with PE have negative lower extremity duplex studies for deep vein thrombosis (DVT).

A

ANSWER: E

COMMENTS: DVT occurs in 30% of ICU patients and is moni- tored by governing bodies in the United States; all ICUs should have a prevention protocol.

High-risk factors are thoracic or general procedures requiring general anesthesia for longer than 30 min, active malignancy, neurosurgical procedures, coronary artery bypass grafting (CABG), CHF, and respiratory failure, along with the traumatic injury.

ICU patients almost always have at least one risk factor and need prophylaxis. Options include pharmacologic treatment with low-molecular-weight heparin (LMWH), unfractionated heparin, or pneumatic compression devices. In regard to diagnosis, duplex ultrasound has a specificity and sensitivity greater than 95%.

Many emboli can be silent, but symptoms of small-to-medium emboli are usually pulmonary (i.e., dyspnea, chest pain, and cough).

Tachypnea and tachycardia are present as well.

Massive PE often produces cardiovascular findings such as elevated PA pressure and right heart strain.

Angiography is the definitive diagnostic technique for this disease, but a helical CT scan of the chest with infusion has shown excellent specificity.

Even without pulmonary infarction, radiographic abnormalities appear as diaphragmatic elevation, atelectasis, and effusion.

For treatment of DVT, heparin therapy over a period of 5 to 7 days with an overlap with warfarin constitutes the treatment of choice.

Warfarin should be continued for 6 to 12 weeks for calf vein and large- vein thrombosis and up to 6 months for PE.

How well did you know this?
1
Not at all
2
3
4
5
Perfectly
71
Q

A 25-year-old male patient with Von Hippel–Lindau (VHL) syndrome is found to have bilateral adrenal masses identified on a CT of the abdomen. His medication list includes both an angiotensin-converting enzyme (ACE) inhibitor and calcium channel blocker for hypertension. As part of his preoperative preparation, what medication(s) should he receive?

A. β-blockade

B. α-blockade

C. α-blockade followed by β-blockade

D. β-blockade followed by α-blockade

E. Hydrocortisone

A

ANSWER: C

COMMENTS: A rare genetic syndrome, VHL is associated with numerous benign tumors that have potential malignant transformation or other physiologic consequences.

Pheochromocytoma is one of these associated lesions and should be suspected in any young patient with the disorder and hypertension.

All patients with pheo- chromocytoma should undergo preoperative α-blockade to negate the activity of the tumor.

Typically this is accomplished with twice- daily dosing of phenoxybenzamine or another α-blocking medication.

The dose is generally started at a low level and increased until orthostatic hypotension develops.

At this point, with the patient sufficiently α-blocked, there is often rebound tachycardia for which a β-blocker is then started.

These medications are continued through the perioperative period until the patient has had the pheochromocytoma surgically excised.

If the patient is also adrenally insufficient, stress-dose steroid at the time of the operation may be indicated.

How well did you know this?
1
Not at all
2
3
4
5
Perfectly
72
Q

A 56-year-old male with hypertension and diabetes is anticoagulated following an emergent right femoral-popliteal arterial bypass. He has been continually oozing from his fasciotomy sites, and his hemoglobin has drifted down in the past 3 days to a level of 7.8 g/dL. On review of his chart you see that in the preoperative clearance note from cardiology he had a hemoglobin level of 13.0 g/dL and no significant cardiac disease. His family is concerned about how pale he has been during this ICU stay. His vital signs are an HR of 86 beats/min, BP of 128/69 mmHg, and Sao2 of 96%. What is the appropriate answer regarding a blood transfusion for this patient at this time?

A. Transfuse five units of PRBCs to reach the preoperative hemoglobin level of 13 g/dL.

B. Check complete blood count (CBC) levels daily and hold transfusion until the hemoglobin level is lower than 9 g/dL.

C. Start erythropoietin at 40,000 units daily.

D. Transfuse PRBCs to a level greater than 10 g/dL.

E. Check daily CBC levels and hold transfusion until the hemoglobin level is lower than 7 g/dL.

A

ANSWER: E

COMMENTS: Anemia is very common in critically ill patients; in the United States, approximately 85% of patients spending more than 1 week in the ICU receive one or more units of PRBCs in their first week.

Blood is a scarce and expensive resource and is associated with morbidity, including transfusion reactions, infections, and worse outcomes.

Historically, patients received transfusions if their hemoglobin level dropped below 10 g/dL.

However, a multicenter prospective randomized clinical trial in 1999 showed that transfu- sion for a hemoglobin level of less than 7 g/dL had the same 30-day mortality rate as transfusion when the hemoglobin level was less than 10 g/dL (except in patients with the significant cardiac disease).

This patient had preoperative cardiac clearance and is not presently showing any signs of hemodynamic instability.

Patients with anemia of critical illness have been shown to have a blunted response to both endogenous and exogenous erythropoietin.

A multicenter trial showed a mild increase in hemoglobin, but it is unclear in the literature whether this improves clinical outcomes.

How well did you know this?
1
Not at all
2
3
4
5
Perfectly
73
Q

The routine use of which of the following is associated with a lower rate of VAP in the ICU setting?

A. Ventilator weaning protocol

B. Chlorhexidine oral rinse

C. Daily sedation vacation

D. H2 blocker

E. Maintaining the head of bed at 30 to 45 degrees

A

ANSWER: A

COMMENTS: Most of the items are routinely included on “VAP bundles” in ICUs across the country, although not all are aimed at specifically preventing VAP.

The use of H2 blockers and DVT prophylaxis are indicated for nearly all mechanically ventilated patients given their increased risk for stress ulcer and thromboembolic disease; however, they play no role in the reduction of pneumonia or liberation from the ventilator.

Chlorhexidine oral rinse and head- of-bed elevation are also standard practice for ventilated patients, and while not often harmful, their efficacy has been unable to be proven or widely replicated in randomized trials.

It is widely accepted, however, that an established ventilator weaning protocol does reduce overall days of mechanical ventilation, which in turn results in a decreased rate of VAP.

Daily SATs do not have a significant effect on ventilator days, ICU length of stay, or rates of VAP.

Other considerations include the role of early tracheostomy as well as newer technologies like endotracheal tubes with the aspiration of subglottic secretions and their potential applications going forward.

How well did you know this?
1
Not at all
2
3
4
5
Perfectly
74
Q

A 47-year-old man with hepatitis C cirrhosis and hepatocellu- lar carcinoma is in the ICU following orthotopic liver transplant. Preoperatively, he developed renal failure secondary to hepatorenal syndrome and remains on dialysis but otherwise is recovering well. On postoperative day 5, his platelet count is noted to be 70,000 per microliter from 150,000 per microliter one day prior. What is the most likely diagnosis?

A. Consumptive coagulopathy

B. Laboratory error

C. Hemodilution

D. Heparin-induced thrombocytopenia

E. Idiopathic thrombocytopenic purpura (ITP)

A

D

How well did you know this?
1
Not at all
2
3
4
5
Perfectly
75
Q

The diagnosis of heparin- induced thrombocytopenia (HIT) is confirmed. What is the next best step?

A. Argatroban drip

B. Oral anticoagulation with Coumadin

C. Hold all anticoagulation

D. Lepirudin

E. Transition from heparin to Lovenox

A

ANSWER: A

COMMENTS: Type II HIT has developed in this patient. Type I HIT occurs in 1%–2% of patients and causes transient sequestration of platelets with a drop in the count to less than the normal range or a 50% fall in the platelet count within the normal range.

In general, this is of little consequence. Platelet levels normalize in a few days after heparin is discontinued.

Type II is more severe, and antiplatelet antibodies develop in 0.1%–0.2% of patients exposed to heparin.

It is associated with thromboticcomplications in more than 30% of cases and should be suspected in a patient in whom resistance to anticoagulation, thromboembolic events, and a fall in the platelet count greater than 30% or a count of less than 100,000/mm3 develop.

Once HIT is suspected, all sources of heparin, including LMWH, should be discontinued. Warfarin can worsen the prothrombotic state and should not be used before complete anticoagulation is achieved with argatroban or lepirudin, both antithrombin agents.

Lepirudin undergoes renal elimination, which should be noted in situations such as this patient with renal insufficiency. Argatroban is metabolized hepatically and is the best choice in this situation.

How well did you know this?
1
Not at all
2
3
4
5
Perfectly
76
Q

A 60-year-old man with renal failure who has been undergoing dialysis for the past 2 years is admitted for cellulitis surrounding the fistula site on his right upper extremity. Antibiotics are started, and the patient is observed. On hospital day 4, his fistula clots and he is taken to the OR for revision. On the following day, he is febrile, coughing up thick green sputum, and dyspneic despite having undergone dialysis that morning. A chest radiograph shows an infiltrate in his right lower lobe, and laboratory tests show a WBC count of 18,000/mm3. Which characteristic of nosocomial pneumonia listed below is not correct?

A. Characterized by onset within 24 h of hospital admission

B. Purulent sputum

C. Isolation of the pathogenic organism from blood or the lung

D. Elevated WBC count

E. Infiltrate on chest radiography

A

ANSWER: A

COMMENTS: Hospital-acquired pneumonia (HAP) is the second most common of all nosocomial infections in the United States.

The Centers for Disease Control and Prevention’s definition of nosocomial pneumonia is a clinical one that requires pneumonia to occur more than 48 h after hospital admission and excludes any infections that are present or incubating at admission. The other two criteria include appropriate findings on physical examination or an infiltrate on chest radiography plus one of the following: purulent sputum, isolation of the pathogenic organism from blood or the lung, identification of a virus from the lower respiratory tract, or serologic or pathologic evidence of recent infection.

Many clinical studies have shown that early, appropriate, and adequate antibiotic therapy can reduce the mortality rate from HAP, currently listed as anywhere from 24% to 76%. T

he American Thoracic Society presumes that early-onset pneumonia is due to Haemophilus influenzae, methicillin- susceptible Staphylococcus aureus, Streptococcus pneumoniae, or anaerobes.

Late-onset HAP occurs more than 4 days after admission and is usually caused by gram-negative organisms, especially Pseudomonas aeruginosa, Acinetobacter, Enterobacteriaceae (Klebsiella, Enterobacter, Serratia), or methicillin-resistant S. aureus (MRSA). Broad-spectrum antibiotics should be started early and deescalated, not escalated, when culture sensitivities are known.

This patient does not require intubation at this time, and low-volume, lung-protective ventilation is best used for ARDS. Patients with HAP do not need bronchoscopy daily. Chest therapy, elevation of the head of the bed, and ambulation are all methods to improve pulmonary toilet.

How well did you know this?
1
Not at all
2
3
4
5
Perfectly
77
Q

A 27-year-old patient presented with a prolonged ICU course for Fournier’s gangrene. Upon admission 5 days ago he had arterial and central lines placed along with a Foley catheter, which has since been removed. He now complains of diffuse body ache, anorexia, and cough with thin, white sputum. He has not had flatus or a bowel movement for 24 h. Chest radiography shows bilateral haziness at the costophrenic angles. Physical examination shows no acute distress, crackles in the lung bases bilaterally, and a swollen right arm. He has some redness around the right side of his neck and chest while his abdomen is soft, distended, and tympanitic but nontender. Vital signs include a temperature of 101.6°F, HR of 100 beats/ min, BP of 128/75 mmHg, and Sao2 of 96%. Lab findings include WBC of 18,500/mm3, sodium concentration of 140 mEq/L, potassium of 4.3 mEq/L, BUN of 21 mg/dL, creatinine level of 0.8 mg/dL, and liver profile within normal limits. What is the most likely diagnosis?

A. Acalculous cholecystitis

B. HAP

C. Catheter-related bloodstream infection

D. Perforated peptic ulcer

E. Viral respiratory infection

A

ANSWER: C

COMMENTS: The most likely diagnosis is a catheter-related bloodstream infection.

He had a catheter placed on an emergency basis and is now experiencing fevers and malaise, with cellulitis evident in the right side of his neck.

His infection can explain the anorexia, ileus, and elevated WBC count.

Catheter-related blood- stream infection is seen in approximately 5% of patients with indwelling catheters and should be suspected if any erythema or purulence is identified at the catheter site.

The subclavian vein is the preferred site for the reduction of infection, over internal jugular or femoral locations. Once an infection is suspected, blood should be drawn through the line and peripherally for culture, and immediate removal of the catheter is suggested and the tip sent for culture.

A catheter–peripheral colony-forming unit (CFU) ratio of 8 signifies line sepsis, and a catheter tip culture with 25 CFUs confirms a catheter-related infection.

How well did you know this?
1
Not at all
2
3
4
5
Perfectly
78
Q

A 45-year-old male is recovering from multiorgan failure after laparotomy for a perforated gastric ulcer. He has been afebrile for 48 h and is not taking any antibiotics. His WBC count is normal and renal failure has resolved. Encephalopathy is improving, and oxygenation is ade- quate, although attempts to wean him from the ventilator have been unsuccessful. Neurologic examination shows symmetrical quadriparesis with sparing of the face and depressed deep tendon reflexes. Spinal tap is normal. Which of the following statements is true concerning his condition?

A. Nerve biopsy often shows demyelination or inflammation.

B. Failure to wean from the ventilator is due to phrenic nerve involvement.

C. Corticosteroids are the treatment of choice.

D. Serum antibodies against acetylcholine receptors are always present.

E. Plasmapheresis is the initial treatment of choice.

A

ANSWER: B

COMMENTS: Critical illness polyneuropathy (CPU) is an axonal motor sensory neuropathy that accompanies sepsis with encephalopathy.

It is due to primary axonal degeneration and affects motor fibers more than sensory fibers. Frequently, it is manifested as a failure to wean a patient from the ventilator because of phrenic nerve involvement despite clinical improvement.

Symmetrical quadriparesis with facial sparing and depressed deep tendon reflexes is characteristic, and electromyography confirms the diagnosis.

Spinal fluid is normal, unlike the case in patients with Guillain-Barré syndrome. Facial involvement and detection of antibodies against acetylcholine are characteristic of myasthenia gravis.

Nerve biopsy shows axonal degeneration without demyelination or inflammation.

Treatment is supportive, and corticosteroids are contraindicated.

How well did you know this?
1
Not at all
2
3
4
5
Perfectly
79
Q

A 63-year-old man is admitted to the ICU following a Hartmann procedure for Hinchey type IV diverticulitis 5 days earlier. The patient is intubated and maintained on AC ventilation, is tachycardic, and is febrile to 101°F. The nurse has noticed an increase in tracheobronchial secretions that are purulent in character. A chest radiograph shows a new infiltrate in the right lung. Which of the following statements is false regarding this patient’s condition?

A. The most likely organism involved is methicillin-sensitive S. aureus.

B. The frequency of ventilator circuit changes does not influence the incidence of this complication.

C. Kinetic beds and elevation of the head of the patient to 45 degrees decrease its incidence.

D. The risk for development of this complication is highest in the second week.

E. Qualitative cultures or secretions are preferred over quantitative culture techniques.

A

ANSWER: D

COMMENTS: VAP has significant costs and a mortality of about 25%. The risk of acquiring VAP is highest in the first week (3% per day), thereafter decreasing to 2% per day in the second week and to 1% per day in the third week.

VAP is generally categorized as early (<48 h after intubation) or late (occurring after 5 to 7 days of intubation).

Early-onset VAP is associated with bacteria that are normally sensitive to antibiotics (S. aureus, H. influenzae, and S. pneumoniae), whereas late-onset VAP is typically associated with antibiotic-resistant bacteria (MRSA, P. aeruginosa, Acinetobacter, and Enterobacter species).

The major risk factors for VAP include trauma, burns, and stay in neurosurgical units as opposed to medical ICUs.

Known risk factors include patients older than 60 years who require prolonged (>48 h) mechanical ventilatory support, aspiration, a nasogastric tube, failure to elevate the head of the bed, and endotracheal cuff pressures of less than 20 cm H2O.

Orotracheal intubation carries a lower incidence of VAP than nasotracheal intubation.

Because contamination of ventilator circuits is universal, the ventilator circuit change interval does not affect the incidence of VAP.

Heat and moisture exchangers may be associated with a slightly lower incidence of VAP than heated humidifiers.

Drainage of subglottic secretions is associated with a decreased incidence of VAP, especially early-onset VAP.

Kinetic beds and positioning of patients at 45 degrees from the horizontal are also associated with a decreased incidence.

Previous exposure to antibiotics in a pro- longed preoperative hospitalization exposes patients to health care– related infections.

Selective digestive decontamination has been reported to be associated with a decreased incidence of VAP, yet these therapies should be time limited to prevent the growth of resistant organisms.

The suspicion for VAP in this patient with a prolonged period of ventilation and new onset of fever, leukocytosis, and purulent sputum should be high, particularly if the chest radiograph shows a new infiltrate.

The diagnosis is best established by quantitative culture of secretions obtained from the lower respiratory tract. The two techniques used include protected specimen brush (PSB) sampling and bronchoalveolar lavage (BAL).

A threshold of 1000 CFU/mL for PSB and 10,000 CFU/mL for BAL is currently recommended. The presence of less than 50% neutrophils in BAL fluid has also been used to exclude pneumonia.

Even though the effect of these techniques on patient outcome is unclear, they have resulted in a significant reduction in the use of antibiotics.

How well did you know this?
1
Not at all
2
3
4
5
Perfectly
80
Q

A 22-year-old man involved in a motor vehicle accident is found to have a thoracic spine fracture (T6) and paraplegia. The patient is hypotensive with a systolic BP of 70 mmHg, is bradycardic with a pulse of 48 beats/min, and is breathing comfortably. Which of the following would be the most appropriate initial treatment?

A. Isotonic fluid administration

B. Steroid administration within 24 h of the injury

C. Immediate intubation

D. α-Agonist administration

E. Immediate magnetic resonance imaging

A

ANSWER: A

COMMENTS: Neurogenic shock refers to a condition characterized by hypotension and bradycardia that results from the interruption of the sympathetic nervous system pathways within the spinal cord.

Common causes include sensory stimulation, such as severe pain, exposure to unpleasant events or sights, high spinal anesthesia, and traumatic spinal cord injury. Clinical characteristics include a BP that is often low, as in other forms of shock.

However, the pulse rate is usually slower than normal, and the skin is flushed, warm, and dry.

CO is reduced secondary to decreased blood return to the heart because of the increased capacitance of the arterioles and venules.

Since the heart receives sympathetic input, there is a difference between injuries above and below T4.

The former depresses cardiac function and decreases venous return. The bradycardia is caused by sympathectomy of the spinal injury above the level of T4 with no capacity for compensatory tachycardia.

Treatment of neurogenic shock secondary to spinal cord injury is usually more complicated not only because of more prolonged hypotension but also because of the presence of coin- cident hypovolemic shock resulting from associated injuries.

Such patients often require ventilatory support as a result of decreased spontaneous respiration and loss of the accessory muscles for breathing. Aggressive fluid therapy should be instituted early under continuous cardiovascular monitoring.

Persistent hypotension necessitates recognition of possible hemorrhagic shock, and a vasopressor such as ephedrine or phenylephrine may be needed. If the injury is below T4, a pure α-agonist may aggravate the reflex bradycardia. Thus a drug with mixed chronotropic and inotropic effects (e.g., norepinephrine or dopamine) is preferred.

A nasogastric tube should be inserted because gastric atony, dilation, and hypersecretion develop in these patients.

Treatment of milder forms of neurogenic shock consists of removing the nociceptive stimulus.

Neurogenic shock resulting from high spinal anesthesia can usually be treated with a vasopressor such as ephedrine or phenylephrine, each of which increases CO by direct effects on the heart and by increasing peripheral vasoconstriction.

Although the administration of steroids remains controversial, their useful- ness for blunt spinal cord injury has been suggested when they are given within 8 h of injury and their administration is extended for 48 h.

How well did you know this?
1
Not at all
2
3
4
5
Perfectly
81
Q

Which of the following is associated with the greatest in-hospital mortality?

A. An ICU patient with an increase in the Sequential Organ Failure Assessment (SOFA) score by 2 points in 24 h

B. A patient in the ED with altered mental status and an RR of 25 breaths/min but no hypotension

C. An ICU patient requiring one vasopressor with a serum lactate value of 2 mmol/L

D. A patient on the medical ward who is alert and oriented but hypotensive and tachypneic

E. All represent sepsis with equal in-hospital mortality

A

ANSWER: C

COMMENTS: In the newest definitions of sepsis and septic shock, the Society of Critical Care Medicine has identified the utility of the SOFA score and its association with in-hospital mortality.

The variables include the Pao2:FiO2 ratio, platelet count, serum creatinine, and bilirubin.

Sepsis may now be indicated by an increase in an individual patient’s SOFA score by 2 points, which is associated with a 10% increase in in-hospital mortality.

The quick SOFA, or qSOFA, has been advocated for the rapid assessment of patients and consists of three variables: mental status, tachypnea (RR > 22 breaths/min), and hypotension (systolic BP < 100 mmHg).

If at least two of these criteria are met, there should be a high index of suspicion for sepsis in these patients and admission to an intensive care setting is appropriate.

Lastly, they identified that the combination of hypotension requiring at least one vasopressor along with a serum lactate value of 2 mmol/L or greater in the absence of hypovolemia is associated with a 40% in-hospital mortality rate.

These two criteria are now recognized and utilized as the definition of septic shock.

How well did you know this?
1
Not at all
2
3
4
5
Perfectly
82
Q

A 28-year-old male with a history of depression is brought into the ED by a family member who believes the patient is overdosed on acetaminophen.

On further questioning the patient admits to ingesting “a few handfuls” of acetaminophen tablets approximately 18 h prior to the arrival to the hospital. On assessment, the patient is awake; however, he is confused. His vital signs are an HR of 105 beats/min, BP 102/77 mmHg, RR 18 breaths/min, and Sao2 of 99%. Lab values are significant for arterial pH 7.32, INR 3.1, and creatinine 4.2. What is the best first step for management of this patient?

A. Administration of IV N-acetylcysteine

B. Stress-dose steroids

C. Aggressive fluid resuscitation with normal saline

D. Activated charcoal

E. Urgent evaluation for liver transplantation

A

ANSWER: A

COMMENTS: Fulminant hepatic failure is classically defined as the development of severe liver injury with impaired synthetic function (INR ≥ 1.5) and new encephalopathy in patients without preexisting liver disease.

In the United States, the most common causes of acute liver failure are acetaminophen overdose and viral hepatitis.

Acetaminophen toxicity can be difficult to predict, as there are often no early symptoms and the modified Rumack- Matthew nomogram is widely used to predict poisoning severity following acetaminophen overdose.

The use of activated charcoal for gastrointestinal decontamination is useful for patients who present within 4 h of acetaminophen ingestion; however, activated charcoal should be used with caution in patients who may not be able to protect their airway.

N-acetylcysteine is the most effective antidote for acetaminophen toxicity, and initiation of therapy within 8 h of ingestion is associated with greatly decreased morbidity.

The King’s College Criteria is used to identify patient’s with a poor prognosis with acetaminophen-induced fulminant hepatic failure.

An arterial pH less than 7.3 or a combination comprising INR greater than 6.5, creatinine greater than 3.4 mg/ dL, and grade III/IV encephalopathy are strong predictors of poor prognosis.

Patients who continue to deteriorate despite appropriate medical therapy should be considered for liver transplantation.

How well did you know this?
1
Not at all
2
3
4
5
Perfectly
83
Q

A 30-year-old female with a history of nonalcoholic steatohepatitis underwent a spontaneous vaginal delivery complicated by uterine atony and a large-volume bleeding, refractory to fundal massage, medical management, and uterine artery embolization. Hemostasis eventually is achieved after the patient underwent exploratory laparotomy and hysterectomy; however, the patient required rapid transfusion of 15 units of PRBCs during her resuscitation. Postoperatively, the patient is extubated and transferred to the surgical ICU for monitoring. Shortly after arrival, the patient complains of perioral numbness and paresthesias in her feet. What is the most likely cause of the patient’s symptoms?

A. Metabolic alkalosis due to alkaline transfusions

B. Overadministration of IV crystalloids

C. Low serum calcium levels in the transfused blood

D. Citrate toxicity

E. Hyperkalemia due to hemolysis of transfused blood

A

ANSWER: D

COMMENTS: Massive blood transfusion is classically defined as transfusion of 10 units of red blood cells within 24 h; however, many other definitions have been proposed.

There are many physiologic alterations associated with massive blood transfusion that require careful monitoring.

Large-volume transfusion of red cells can dilute serum coagulation proteins and platelets leading to a prolongation in PT and activated partial thrombo- plastin time (aPTT).

Additionally, transfused blood is anticoagu- lated with sodium citrate and citric acid.

Rapid transfusion of red blood cells can overwhelm a patient’s ability to metabolize the excess citrate, which can bind ionized calcium and can lead to clinically significant hypocalcemia.

Generally, very rapid transfusion is required to cause significant hypocalcemia; however, those with preexisting liver disease are at a greatly increased risk.

Administration of either calcium gluconate or calcium chloride is an acceptable treatment of hypocalcemia following massive transfusion.

How well did you know this?
1
Not at all
2
3
4
5
Perfectly
84
Q

A 67-year-old-male is brought emergently to the OR for the management of freely perforated diverticulitis. During the procedure the patient is unstable, requiring multiple vasopressors as well as large-volume fluid resuscitation. At the completion of the case, the patient was stabilized and transferred to the SICU intubated with nasogastric decom- pression. The patient is noted to have worsening abdominal distension throughout the night and decreasing urine output despite multiple crystalloid boluses. On examination, the patient has a Richmond Agitation-Sedation Scale score of −1. He has a tense abdomen with a healthy appearing ostomy and an abdominal surgical drain with minimal serosanguinous drainage. His vital signs are a temperature of 37.2°C, HR 98 beats/min, and BP 144/92 mmHg. He is mechanically ventilated, with peak airway pressures of 38 cm H2O and plateau pressures of 28 cm H20. Intravesicular pressure is measured to be 22 mmHg. What is the next best step in managing this patient?

A. Increase the patient’s sedation

B. Decrease the patient’s Vt

C. Administration of a paralytic agent

D. Broadening of antibiotic therapy

E. Emergent return to the OR for reexploration

A

ANSWER: A

COMMENTS: ACS is defined as an end-organ dysfunction in the setting of intraabdominal hypertension.

Intraabdominal hyper- tension refers to sustained intraabdominal pressures ≥ 12 mmHg.

Clinically, there is no defined intraabdominal pressure for ACS as end-organ dysfunction can occur at a variety of pressures between different patients; however, intraabdominal pressures ≥ 25 mmHg are frequently associated with ACS.

Bladder pressures are widely used as a surrogate measurement for intraabdominal pressure.

ACS can occur in a variety of clinical settings, including trauma, burns (>30% of the total body surface area), liver trans- plantation, and postsurgical patients.

Additionally, large-volume resuscitation has been associated with increased occurrence of ACS.

The diagnosis of ACS is generally made with the triad of a tense abdomen, decreased urine output despite fluid resuscitation, and increased airway pressures, all in the setting of abdominal hypertension.

For patients with progressive renal dysfunction or worsening shock, the definitive management of ACS is a decompressive laparotomy.

In patients where there is a concern for the impending development of ACS, supportive care measures can be made to decrease intraabdominal pressures.

Nasogastric and rectal decompression, deep sedation, chemical paralysis, limited fluid administration, and ventilator support to reduce airway pressures may be used.

In the above scenario, the patient is minimally sedated and may have improved intraabdominal pressures with increased sedative dosing.

How well did you know this?
1
Not at all
2
3
4
5
Perfectly
85
Q

The primary survey is best described by which sequence of steps?

A. Airway, Blood pressure, Pulses, Breath sounds, Extremities

B. Airway, Breathing, Circulation, Disrobe, Extremities

C. Airway, Breathing, Circulation, Disability, Exposure

D. Access, Blood pressure, Chest compressions, Disability, Endotracheal intubation

E. Airway intubation, Bilateral chest tube placement, Central line placement

A

ANSWER: C

COMMENTS: The Advanced Trauma Life Support (ATLS) Program of the American College of Surgeons has put forth a general framework on how to approach and manage an injured patient.

To streamline care, the A, B, C, D, E, and F sequence has been endorsed, which represents Airway, Breathing, Circulation, Disability, Exposure, and Focused Assessment with Sonography for Trauma (FAST).

This comprises the primary survey with the goal of identifying life-threatening injuries and supporting oxygenation, ventilation, and perfusion.

A team leader confirms that each system is intact and then moves to the next step with continued reassessment and interventions as needed.

Common interventions include endotracheal intubation or cricothyroidotomy, placement of chest tubes and central venous catheters (CVCs), and, rarely, an emergency department (ED) thoracotomy.

This is followed by a secondary survey that allows for a head-to-toe examination and a brief medical history.

How well did you know this?
1
Not at all
2
3
4
5
Perfectly
86
Q

A middle-aged male is found lying on the ground in an alley and is brought into the trauma bay for additional evaluation. His vital signs are stable. The only sign of trauma is a large scalp laceration. The patient is confused with a Glasgow Coma Scale (GCS) score of 7. The decision is made to intubate the patient. What is the correct order for rapid-sequence intubation in trauma?

A. In-line cervical immobilization, preoxygenation, cricoid pressure, induction, paralysis, intubate, confirm placement with CO2 detector

B. In-line cervical immobilization, preoxygenation, cricoid pressure, paralysis, induction, intubate, confirm placement with CO2 detector

C. In-line cervical immobilization, preoxygenation, cricoid pressure, induction, intubate, confirm placement with CO2 detector, paralysis

D. Tilt head back in sniffing position, preoxygenation, cricoid pressure, induction, paralysis, intubate, confirm placement with CO2 detector

E. Tilt head back in sniffing position, preoxygenation, cricoid pressure, paralysis, induction, intubate, confirm placement with CO2 detector

A

ANSWER: A

COMMENTS: The most commonly used method for securing a trauma patient’s airway is orotracheal intubation.

Rapid-sequence intubation consists of preoxygenating the patient for 3 min with bag-valve-mask ventilation while maintaining in-line cervical stabilization, applying cricoid pressure to limit aspiration, administering an induction agent followed by a paralytic, performing laryngoscopy, and then placing an endotracheal tube.

In-line cervical stabilization should be maintained with the help of an assistant, not a rigid cervical spine collar.

Common medications used include etomidate, ketamine, and succinylcholine.

Etomidate is an induction agent that has a quick onset of action and is indicated for patients with a suspected brain injury or open globe because it does not cause an increase in intracranial pressure (ICP).

However, it can cause adrenal insufficiency in rare cases. Ket- amine is a dissociative anesthetic that can cause tachycardia and hypertension as common side effects.

Succinylcholine should be avoided in patients with burns or a spinal cord injury because it can cause a rise in the serum potassium concentration and can lead to severe hyperkalemia.

Confirmation of tube placement should be completed with a carbon dioxide detector and chest x-ray.

How well did you know this?
1
Not at all
2
3
4
5
Perfectly
87
Q

A 72-year-old female presents after being ejected during a high-speed motor vehicle crash. Her heart rate (HR) is 155 beats/min, and blood pressure is 60/35 mmHg. Emergency medical technicians (EMT) were not able to obtain intravenous (IV) access. What is the best access for this patient so that fluid resuscitation can be initiated?

A. Saphenous vein cut down

B. Ultrasound-guided left internal jugular vein triple-lumen catheter

C. Right subclavian triple-lumen catheter using landmarks

D. Intraosseous (IO) access at the humerus

E. IO access at the tibia

A

ANSWER: D

COMMENTS: When traditional vascular access methods have not been successful, IO access can provide a means to give resuscitative fluids, medications, blood, and contrast.

The IO route has been demonstrated to have a significantly higher first-attempt success rate and can be completed in a much quicker fashion and with fewer complications.

Even experienced providers have dif- ficulty placing a CVC using landmarks. CVC in trauma has about a 50% success on the first pass and requires an average of 8 min to place.

In contrast, IO placement is over 90% successful and takes approximately 1 min to place.

Ultrasound guidance has been shown to improve central line placement. However, it is not always practical in an emergency setting because it may not be readily available, takes time to set up, and space around a patient is limited.

Many sites have been utilized for IO access. Although IO use is limited to the anterior tibia in the pediatric population, cadaver studies in adults showed the best flow rates with minimal placement or dislodgement issues in the sternum followed by the humerus.

The humerus is now the preferred position.

How well did you know this?
1
Not at all
2
3
4
5
Perfectly
88
Q

A 17-year-old female presents after a motor vehicle crash and is in hemorrhagic shock. A massive transfusion protocol is triggered. What is the optimal ratio in damage control resuscitation?

A. 2 plasma:1 platelet:1 red blood cell

B. 1 plasma:1 platelet:1 red blood cell

C. 1 plasma:1 platelet:3 red blood cells

D. 2 plasma:1 cryoprecipitate:1 red blood cell

E. 1 plasma:1 cryoprecipitate:1 red blood cell

A

ANSWER: B

COMMENTS: Approximately 1%–3% of trauma patients require a massive transfusion. The US Department of Defense’s Prospec- tive Observational Multicenter Major Trauma Transfusion (PROM- MITT) study demonstrated that early transfusion with higher plasma-to-platelet ratios was associated with decreased mortality.

The Pragmatic, Randomized Optimal Platelets and Plasma Ratios (PROPPR) trial demonstrated the optimal ratio of 1 plasma:1 plate- let:1 red blood cell versus 1 plasma:1 platelet:2 red blood cells was associated with more patients achieving hemostasis with fewer hemorrhage-related deaths without increased complications.

The overall 24-h and 30-day mortality was equal between groups. Both these studies support the early use of balanced transfusion therapy in an exsanguinating injured patient.

How well did you know this?
1
Not at all
2
3
4
5
Perfectly
89
Q

A 77-year-old male with a history of atrial fibrillation on dabigatran, a direct thrombin inhibitor, presents after a fall from a ladder. His HR is 130 beats/min and irregular, blood pressure is 82/44 mmHg, and FAST is positive, with fluid in the right upper quadrant (RUQ) and pelvis. Prior to proceed- ing to the operating room, what is the best reversal agent for this patient’s anticoagulation?

A. Fresh-frozen plasma

B. Prothrombin complex concentrates

C. Protamine sulfate

D. Idarucizumab

E. Platelets

A

ANSWER: D

COMMENTS: As the population grows older and more novel anticoagulants are prescribed, trauma surgeons need to know how to manage and reverse these medications acutely in a bleeding patient.

Warfarin, a vitamin K antagonist, has traditionally been managed with fresh-frozen plasma and vitamin K injections.

Recombinant factor VII has also been used to treat these patients and cirrhotics with an elevated international normalized ratio (INR).

However, there is an increased prothrombotic risk potential with this therapy.

Newer therapies include either 3- or 4-factor prothrombin complex concentrates (PCCs) that contain variable amounts of factors II, VII, IX, and X and proteins C and S and rapidly reverse the effects of warfarin within 30 min.

Heparin and low-molecular-weight heparin (LMWH) are reversed with variable dosing of protamine sulfate.

The effects of platelet inhibitors including aspirin and clopidogrel can be minimized with desmopressin and platelet transfusions.

Novel agents including direct thrombin or factor Xa inhibitors can be managed with activated charcoal and PCC.

A monoclonal antibody, idarucizumab, specifi- cally adheres to the thrombin-binding site of dabigatran, rendering it inactive within a minute of dosing.

How well did you know this?
1
Not at all
2
3
4
5
Perfectly
90
Q

A 75-year-old morbidly obese male presents after a motor- cycle crash. The primary survey demonstrates an intact airway, diminished breath sounds from the left chest, GCS of 10, and limited movement of upper extremities. His HR is 122 beats/ min, and blood pressure is 80/44 mmHg. A left chest tube is placed with 1 L of bloody output. The patient has 2 L of saline infused. His HR decreases to 116 beats/min, and blood pressure is 86/48 mmHg. FAST was indeterminate. What is the next best step to evaluate for intraabdominal hemorrhage?

A. Serial abdominal examinations

B. Diagnostic peritoneal tap

C. Computed tomography (CT)

D. Exploratory laparoscopy

E. Exploratory laparotomy

A

ANSWER: B

COMMENTS: This patient has several sources of shock including hemorrhage from his thorax and likely intracranial and spinal injuries.

He is too unstable to leave the resuscitation area to go for CT imaging, and an abdominal source of bleeding must be ruled out prior to the definitive management of his injuries.

Diagnostic peritoneal tap with lavage is a very sensitive, but nonspecific, test that can be used for either blunt or penetrating trauma.

In this situation, diagnos- tic peritoneal lavage (DPL) is used to determine an intraperitoneal injury.

However, it will miss retroperitoneal bleeding. DPL is performed by introducing a catheter into the abdomen via the Seldinger technique and then infusing 1 L of 0.9% normal saline.

The fluid is then returned by gravity and sent to the laboratory for analysis. A positive DPL is defined as one with greater than 100,000 red blood cells/mm3, 500 white blood cells (WBCs)/mm3, or the presence of bilious/particulate material.

A limited version of this procedure is a diagnostic peritoneal tap in which a needle is passed into the peritoneal cavity.

If blood or succus is aspirated by a 10-ml syringe, it is deemed positive. If this patient has a positive DPL, the next most appropriate step in management is exploratory laparotomy.

Serial abdominal examinations in an unstable patient are not appropriate.

How well did you know this?
1
Not at all
2
3
4
5
Perfectly
91
Q

A 20-year-old female presents after a motor vehicle crash. She complains of diffuse abdominal pain. Her HR is 140 beats/min, and blood pressure is 78/42 mmHg. FAST is positive. Which statement best describes the FAST examination?

A. A 3.5-MHz convex-array transducer should be used.

B. The hepatorenal space, known as the Morison pouch, is viewed between the 11th and 12th ribs in the right midaxillary line.

C. The splenorenal space is evaluated between the 9th and 11th ribs in the left posterior axillary line.

D. The bladder should
preferentially be full before the examination to allow better visualization of fluid in the pelvis.

E. All of the above.

A

ANSWER: E

COMMENTS: Focused assessment for the sonographic examination of trauma patients is performed as part of the ATLS secondary survey.

A 3.5-MHz convex-array transducer is used to evaluate for the presence of fluid in the abdomen.

Four areas are to be examined.

The first is the pericardial window, which is viewed with the transducer placed subxiphoid. The hepatorenal space is evaluated in the right midaxillary line, between the 11th and 12th ribs.

The splenorenal space is evaluated in the right posterior axillary line, between the 9th and 11th ribs.

The last area examined is the pouch of Douglas in the pelvis. This rectouterine/rectovesical space is evaluated with the transducer placed approximately 3 cm above the pubic symphysis.

A full bladder aids in detecting the presence of blood in this space; therefore Foley catheters should be placed after FAST has been performed.

How well did you know this?
1
Not at all
2
3
4
5
Perfectly
92
Q

A 23-year-old male is brought by emergency medical services (EMS) with a stab wound to the chest 2 cm medial from the right nipple. He was intubated en route; carotid pulse was present but faint, and pupils were responsive to light. After transferring the patient onto the gurney, no palpable pulses are appreciated. What is the next step in management?

A. Right tube thoracostomy

B. Right anterolateral thoracotomy

C. Median sternotomy

D. Left anterolateral thoracotomy

E. Left posterolateral thoracotomy

A

ANSWER: D

COMMENTS: An ED thoracotomy is performed for a select group of patients, with the overall survival rate being dependent on the mechanism and ranging from 1.5% to 19%.

It is least successful in patients with injuries caused by a blunt mechanism and is therefore usually reserved for those who initially have vital signs present but then lose these signs in the ED.

In patients with a penetrating mechanism, a thoracotomy is indicated for those who have lost their pulse and blood pressure either in the ED after initial evalu- ation or during transport to the ED.

Thoracotomy is best used when cardiac tamponade or severe thoracic hemorrhage is suspected.

The thoracotomy is performed on the left side via an anterolateral approach, regardless of the supposed side of the injury, and allows the release of pericardial tamponade, open cardiopulmonary resus- citation, and aortic cross-clamping.

Resuscitative endovascular balloon occlusion of the aorta (REBOA) is a newer, less invasive technique that is being utilized to control intraabdominal and pelvic hemorrhage.

Outcomes are still inconclusive, and studies are ongoing to establish its routine use as an adjunct or alternative to current therapy.

How well did you know this?
1
Not at all
2
3
4
5
Perfectly
93
Q

A 29-year-old female is the driver of an automobile involved in a high-speed motor vehicle crash. She is 30 weeks pregnant. She complains of abdominal pain but does not have peritoneal signs. Her HR is 105 beats/min, and blood pressure is 108/66 mmHg. Which of the following statements is true regarding trauma in a pregnant patient?

A. Less than 5% of all pregnancies are affected by trauma.

B. The uterus is protected by the bony pelvis until the beginning of the second trimester.

C. A woman of 25 weeks’ gestation will have a palpable fundal height at approximately the level of the umbilicus.

D. Blood volume during pregnancy increases by approximately 30%.

E. Hypotensive patients should be placed in the right lateral position.

A

ANSWER: B

COMMENTS: Trauma is the leading cause of death in women of childbearing age and thus understanding the physiologic changes throughout the progression of pregnancy is imperative.

However, since approximately 10% of pregnant patients are unaware of their pregnancy, a pregnancy test is recommended for all women of childbearing age early in their resuscitation.

The most common cause of fetal death is maternal death. Therefore the focus of all initial resuscitative effort is directed toward the mother.

Although the primary and secondary surveys for a pregnant patient are virtu- ally identical to those of a nonpregnant patient, it is important to perform a focused abdominal examination.

Normal physiologic changes in pregnancy include an increase in HR by 10 to 15 beats/ min, a 25% increase in cardiac output, mild hypotension with average systolic blood pressures of 100, increased minute ventilation with resulting respiratory alkalosis, relative anemia, and a hypercoagulable state.

Blood volume may increase by as much as 50% during pregnancy, which means that a patient may not have the tachycardia and hypotension usually associated with acute blood loss until almost 30% of total blood volume is lost.

The pelvis typically protects the uterus until about 12 weeks. At about 20 weeks’ gestation, the fundal height of the uterus approximates the umbilicus, and for every week of gestation past this stage, the height increases roughly by 1 cm.

During the advanced stages of pregnancy, the uterus causes compression on the inferior vena cava (IVC), thereby leading to a decreased central venous return.

Hypotensive patients should be placed in the left lateral position, which even in patients with a suspected spinal injury can be accomplished by securing the patient firmly to the backboard, which can then be tilted to the left.

Evaluation of the fetus is accomplished by fetal heart tone monitoring and pelvic ultrasound.

Tachycardia, bradycardia, and decelerations with contractions are all signs of potential fetal distress.

How well did you know this?
1
Not at all
2
3
4
5
Perfectly
94
Q

A 32-year-old male presents after being assaulted by a bat to his head, face, and torso. He opens his eyes to sternal rub, localizes to pain, and is only making groaning sounds. What is this patient’s GCS?

A. 7

B. 8

C. 9

D. 10

E. 11

The patient in the previous scenario now only has extensor posturing. What is the next best step in management?

A. CT scan of the head

B. Continue to perform primary and secondary surveys

C. Bolus of hypertonic saline

D. Elevation of the head of the bed 30 degrees

E. Endotracheal intubation

A CT of the head is obtained demonstrating a 3-cm right- sided subdural hematoma with a 1-cm midline shift. The patient is taken to the operating room, and the hematoma is evacuated. Postoperatively, he is admitted to the intensive care unit (ICU), intubated, and sedated with an ICP monitor in place. Which of the following treatment measures can be used to maintain adequate cerebral perfusion pressure (CPP)?

A. Hyperventilation to a PaCO2 of 25 mmHg

B. Mannitol, 1 g/kg IV

C. Hydrocortisone, 100 mg IV every 8 h for 3 days

D. Strict blood pressure control to a systolic range of 90 to 100 mmHg

E. Reverse Trendelenburg positioning of the bed at all times

A

A N S W E R S : C, E, B

COMMENTS: The GCS score allows multiple health professionals to follow a patient’s neurologic status. It is based on the evaluation of three examinations—eyes, verbal, and motor—that are independently recorded and added together to obtain an overall score.

The best examination from each category is utilized.

The motor component is the most important regarding prognosis.

As with all trauma patients, repeated examinations with a focus on the fundamentals of airway, breathing, and circulation are key.

This patient demonstrates a decline in GCS score, which should prompt an evaluation of his airway and intubation.

A GCS score of 8 is often used as a marker for the need for intubation to secure an airway.

Any patient with a GCS score ≤ 14 or loss of consciousness should get a CT of the brain without contrast to evaluate for an intracranial injury; however, this is secondary in this patient.

The overall goal in treating patients with a traumatic head injury is maintaining adequate cerebral blood flow.

An estimate of this flow is obtained by calculating CPP (CPP = mean blood pressure − ICP).

The goal CPP in an adult is 60 to 70 mmHg. Therefore patients should be aggressively volume resuscitated or started on vasopressors to maintain adequate mean blood pressure.

ICP monitors are indicated in patients with a GCS score of 3 to 8 and either (1) an abnormal finding on CT of the head or (2) any two of the following: (a) age older than 40 years, (b) posturing response to pain, or (c) systolic blood pressure less than 90 mmHg.

Overall, there is a 5% infection and 1% hematoma formation rate associated with ICP-monitoring catheters.

Treatment measures used to decrease ICP include elevation of the head of the bed to 30 degrees or more, hyperventilation of patients to a PaCO2 of 35 mmHg, barbiturates, and mannitol at a dose of 1 g/kg IV.

PaCO2 should not be kept below 30 mmHg to avoid worsening the cerebral ischemia.

Steroids do not have a role in the treatment of acute traumatic head injury.

How well did you know this?
1
Not at all
2
3
4
5
Perfectly
95
Q

A 21-year-old male is brought in by EMTs on a backboard with a cervical collar in place after a bar fight that included falling over a stool. The patient is belligerent, and urine drug screen is positive for methamphetamines and alcohol. Primary survey is normal, and secondary survey reveals several facial fractures and multiple right hand fractures. What is the best way to evaluate this patient’s c-spine and remove his collar?

A. Clinical examination

B. Radiographic series of the cervical spine—cross-table lateral, anteroposterior, and open-mouth view of the dens

C. Flexion–extension x-rays

D. Magnetic resonance imaging (MRI) cervical spine followed by clinical examination when appropriate

E. CT cervical spine followed by clinical examination when appropriate

A

ANSWER: E

COMMENTS: Clearance of cervical spine precautions relies on the patient’s ability to participate in the examination.

In an awake patient with minimal risk factors, clinical examination alone is appropriate.

In an obtunded patient, many accepted algorithms are present to evaluate for injury.

According to the Eastern Association for the Surgery of Trauma, obtunded patients should have a CT of the cervical spine.

If no focal neurologic deficits are present and the patient is likely to be obtunded for greater than 72 h, the collar can be removed with a negative CT scan only.

Alternatively, MRI evaluation can be added to the workup.

If both studies are negative, the collar can be removed. If a person is likely to become appropriate within 24 h as determined by an alert mental state without distracting injuries, CT should be completed, and if negative, it should be followed by a clinical examination when the patient is appropriate.

Prolonged use of a cervical collar has been linked to pressure ulcers, increased ICP, prolonged ICU stay, and pneumonia and should be avoided.

CT allows for evaluation of the bony elements of the spine while MRI is best for soft tissue, the spinal cord, intervertebral disks, and ligaments.

Patients with spinal cord injuries, especially in the cervical area, may require early intubation for airway protection and aggressive fluid resuscitation and vasopressors to counter the neurogenic shock due to the loss of autonomic innervation.

These patients typically present with hypotension and relative bradycardia.

Regardless of suspicion for the neurogenic cause of hypotension, full primary and secondary surveys should be completed to rule out the hemorrhagic shock.

How well did you know this?
1
Not at all
2
3
4
5
Perfectly
96
Q

A 35-year-old male presents after being stabbed in the neck. A 3-cm wound is present at the base of the neck, just right of midline above the sternum. An expanding hematoma is noted. The patient is intubated for airway protection and taken to the operating room for exploration. What is the best initial incision to expose and control this injury?

A. Anterior border of the sternocleidomastoid muscle

B. Median sternotomy

C. Right anterolateral thoracotomy

D. Collar incision

E. Right infraclavicular incision

A

ANSWER: A

COMMENTS: The neck is divided into three zones, and management of penetrating injuries is based on the affected area.

Zone 1 is most inferior from the base of the neck to the cricoid cartilage and is exposed with an incision along the anterior border of the sternocleidomastoid muscle with possible extension into a median sternotomy.

Zone 2 extends from the cricoid cartilage to the angle of the mandible and is best approached with an incision along the anterior border of the sternocleidomastoid muscle.

Zone 3 is most superior from the angle of the mandible to the skull base. It is very difficult to approach and may require nasotracheal intubation, division of the digastric muscle, and subluxation of the mandible. Endovascular techniques may be also utilized in this area.

The aerodigestive tract, vascular system, and the spine all run through the neck and must be evaluated for injury.

This can include a combination of a barium swallow followed by an esophagogastroduodenoscopy (EGD), bronchoscopy, a CT angiogram, or an MRI based on clinical symptoms.

How well did you know this?
1
Not at all
2
3
4
5
Perfectly
97
Q

A 19-year-old male presents after a high-speed head-on motor vehicle crash into a tree. He was the unrestrained driver and hit the steering wheel with his chest. He has a sternal fracture, broken left clavicle, and a left pneumothorax. A left chest tube is placed with a resolution of the pneumothorax. What is the best initial evaluation for a blunt cardiac injury (BCI)?

A. Serial electrocardiogram (ECG) only

B. Serial troponins only

C. Echocardiogram only

D. Admission ECG and troponin

E. Admission echocardiogram and serial ECG

A

ANSWER: D

COMMENTS: The best way to evaluate for a significant BCI is an initial ECG and troponin. If these are both normal, BCI can be ruled out.

Patients with significant blunt trauma to the anterior chest with associated injuries, including, but not limited to, rib and sternal fractures, pulmonary contusions, hemothorax, pneumothorax, and polytrauma, should be screened.

Within this population, the incidence of BCI is 13%.

Patients who have abnormal findings on EKG, an elevated troponin, or hemodynamic instability require telemetry monitoring for 24 h.

These patients are at risk for a lethal arrhythmia, which may require immediate defibrillation.

A transthoracic or transesophageal echocardiogram is also indicated. Most common findings include myocardial contusion and right ventricle and right atrial injuries.

Left-sided, valvular, and coronary artery injuries are rare and may require cardiopulmonary bypass for repair.

A subset of patients that will have abnormal ECG and elevated troponin are those who had an acute myocardial infarction immediately prior to or during their trauma.

Cardiac CT or MRI can be used to distinguish these patients who may benefit from cardiac catheterization and anticoagulation.

How well did you know this?
1
Not at all
2
3
4
5
Perfectly
98
Q

A 26-year-old male presents with a stab wound to the right chest about 1 cm lateral to the sternum between the second and third ribs. The patient is awake but confused, his HR is 100 beats/min, and his blood pressure is 102/54 mmHg. He is taken to the operating room after initial resuscitation. What is the best approach to expose this injury?

A. Median sternotomy

B. Right anterolateral thoracotomy

C. Right posterolateral thoracotomy

D. Right supraclavicular incision

E. Incision over the stab wound

A

ANSWER: A

COMMENTS: Penetrating thoracic trauma is common, and the choice of incision can be critical in exposing and repairing the injury.

For an unstable patient in the emergency room (ER), a left anterolateral thoracotomy that can be extended across the sternum onto the right chest can provide maximal exposure to the heart and pericardium, root of the ascending aorta, descending aorta to cross-clamp, and pulmonary hilum.

A median sternotomy allows access to the ascending aorta, the innominate artery including the proximal right carotid and subclavian arteries, the left common carotid artery, and the heart.

The distal right subclavian artery can be approached via a right supraclavicular incision with resection of the middle part of the clavicle.

The proximal descending aorta and left subclavian artery are best exposed via a left posterolateral thoracotomy; however, this is not a favorable position in emergency surgery.

Alternatively, a left third intercostal space antero- lateral thoracotomy and a supraclavicular incision with resection of the middle part of the clavicle can be used.

A median sternotomy can join these incisions, completing a trapdoor incision; however, this adds significant morbidity to the procedure.

Placing a patient on cardiopulmonary bypass can assist with the repair of proximal thoracic vessels; however, this requires additional time and expertise.

How well did you know this?
1
Not at all
2
3
4
5
Perfectly
99
Q

A 27-year-old female is brought in after a high-speed motor vehicle collision (MVC). The patient was intubated on scene for GCS 8. HR is 98 beats/min, blood pressure is 110/58 mmHg, and chest x-ray demonstrates a widened mediastinum. CT imaging demonstrates a 2-cm intracranial epidural hematoma with midline shift, a moderate amount of fluid in the abdomen, and a pseudoaneurysm of the descending thoracic aorta. What is correct regarding the management of a blunt aortic injury?

A. Repair of the aorta needs to be completed emergently to reduce the risk of rupture.

B. Mean arterial pressure should be maintained greater than 85 mmHg.

C. Open repair of the injury has less morbidity and mortality.

D. Most patients with a blunt aortic injury die on the scene.

E. The most common site for an aortic injury is at the root of the aorta.

A

ANSWER: D

COMMENTS: The reported incidence of a blunt thoracic aortic injury is approximately 0.5%; however, the real incidence is likely much higher due to the vast majority of patients dying prior to arrival to a hospital.

The most common aortic injuries include pseudoaneurysms, dissections, intimal flaps, and transections.

This occurs most often just distal to the takeoff of the left subclavian artery on the medial wall due to shear forces on a relatively fixed part of the aorta.

Characteristic x-ray findings of a widened mediastinum, left apical capping, depression of left mainstem bronchus, and loss of aortic knob may initially be seen, raising suspicion for injury, and diagnosis can be confirmed with a CT angiogram of the chest.

Treatment includes strict blood pressure control with restrictive fluid resuscitation and short-acting β-blockers such as esmolol titrated to maintain systolic blood pressures between 90 and 100 mmHg.

This has reduced the incidence of rupture to less than 5% of patients who present to the hospital. Repair is now routinely delayed until other life-threatening injuries are definitively managed with minimum risk.

Both open and endovascular techniques can be utilized, with recent literature demonstrating reduced morbidity and mortality with endovascular approaches.

However, patients may suffer from complications such as endoleaks and access vessel injuries that may require additional procedures.

How well did you know this?
1
Not at all
2
3
4
5
Perfectly
100
Q

A 41-year-old male falls from a 12-foot ladder and lands on his left side. He fractures left ribs 2 to 9 with a flail segment, resulting in pneumothorax. After the chest tube is placed, what is the next best step in management?

A. Pain control with an epidural

B. Incentive spirometer

C. Early ambulation

D. Conservative fluid management

E. All of the above

A

ANSWER: E

COMMENTS: Rib fractures are one of the most common injuries in trauma.

Chest x-rays diagnose less than 50% of rib fractures, while CT scan detects nearly 100% of significant fractures.

Fractures of ribs 1 to 3 can be associated with major thoracic vascular injuries, while fractures of ribs 9 to 12 can puncture intraabdominal organs such as the liver and spleen.

The ideal treatment of rib fractures is centered on pain control and aggressive pulmonary toilet.

Pain regimens include scheduled IV and per os (PO) medications, patient-controlled analgesia (PCA), intercostal blocks, and epidurals.

Pulmonary toilet includes aggressive suctioning, deep breathing and coughing, incentive spirometer use, and early ambulation to decrease atelectasis.

Conservative fluid management is encouraged to decrease pulmonary edema, which can worsen pulmonary contusions that are associated with rib fractures.

Rib plating is making a resurgence with many proponents stating that early rib plating within 72 h can decrease many of the complications associated with severely displaced rib fractures, including prolonged need for ventilator support and pneumonia.

How well did you know this?
1
Not at all
2
3
4
5
Perfectly
101
Q

A 37-year-old female comes in with a stab wound to the right chest just lateral to the sternal border. HR is 130 beats/min, blood pressure is 74/58 mmHg, and jugular venous distention is noted on examination with muffled heart sounds. Which option provides the appropriate diagnosis and treatment?

A. Tension pneumothorax: right thoracotomy

B. Tension pneumothorax: right tube thoracostomy

C. Tension pneumothorax: median sternotomy

D. Cardiac tamponade: pericardial window

E. Cardiac tamponade: left tube thoracostomy

A

ANSWER: D

COMMENTS: Many life-threatening injuries to the chest, such as a massive hemothorax, tension pneumothorax, open chest wound, and cardiac tamponade, must be identified on clinical examination and require prompt intervention.

The clinical examination requires visual inspection of the chest rise for symmetry, open wound with air bubbles, paradoxical motion, and jugular venous distention.

Palpation can detect crepitus and instability of ribs and sternum, while auscultation identifies absent or diminished breath sounds or dullness of heart tones.

Extended FAST can also be used as part of the primary survey to detect a pneumothorax or pericardial effusion.

Once the diagnosis is determined, immediate intervention must take place.

A tension pneumothorax is classically described with absent breath sounds, hypotension, distended neck veins, and tracheal deviation and requires urgent chest tube placement.

Many start with needle decompression; however, many times this intervention is done incorrectly, provides a false sense of security, and delays definitive treatment with tube thoracostomy.

Cardiac tamponade presents with distended neck veins, muffled heart sounds, and hypotension due to fluid in the pericardial sac, decreased right atrial filling, and decreased venous return.
This requires a pericardiocentesis or pericardial window.

How well did you know this?
1
Not at all
2
3
4
5
Perfectly
102
Q

A 21-year-old man is taken to the ED with a gunshot wound to the right side of his chest. HR is 126 beats/min and blood pressure is 88/46 mmHg. A right-sided chest tube is placed, with the return of 1200 mL of blood. He is resuscitated with 2 L of lactated Ringer’s solution, and his vital signs return to within normal limits. His chest tube output is rechecked 4 h later, and the total amount in the collection container is 2300 mL. What is the next most appropriate step in management?

A. Chest CT

B. Stat hemoglobin/ hematocrit

C. Right thoracotomy

D. Left anterolateral thoracotomy

E. Admission to the ICU for continuous cardiac monitoring and pulse oximetry

A

ANSWER: C

COMMENTS: This patient has a massive hemothorax, which is defined as greater than 1500 mL of blood loss on initial placement of tube thoracostomy, continuing loss greater than 200 mL/h for 4 h, or a total of 2500 mL in 24 h after the initial placement.

This is an indication that there is ongoing bleeding that requires surgical intervention.

Additional indications for an urgent thoracotomy include tracheal, esophageal, great vessel, or lung injury.

Penetrating injury to the lung parenchyma may present with a large and persistent air leak.

This may be best managed with a tractotomy, which utilizes a stapler passed into the tract through the lung parenchyma to control the air leak and bleeding.

How well did you know this?
1
Not at all
2
3
4
5
Perfectly
103
Q

A 38-year-old car mechanic is taken to the ED after having been pinned underneath a car. On chest radiography, multiple rib fractures are noted, as well as an air-fluid level consistent with the stomach being above the level of the left diaphragm. Which of the following statements regarding this injury is true?

A. Right-sided diaphragmatic rupture is more common than left-sided rupture.

B. There is a 30% incidence of coexisting pelvic fractures.

C. The best initial radiographic assessment for this type of injury is FAST.

D. In most acute diaphragmatic ruptures, repair can be completed with primary repair.

E. There is a 60% incidence of a coexisting thoracic aortic injury.

A

ANSWER: D

COMMENTS: Diaphragmatic injury/rupture occurs in 3%–5% of patients suffering major blunt abdominal trauma.

Although this injury is uncommon, it is associated with a high incidence of coexistent injuries, including pelvic fractures (15%), hepatic and splenic injuries (40%), and rupture of the thoracic aorta (5%).

The left side is affected three times more often than the right side.

The use of CT imaging of these patients has significantly decreased the incidence of missed injury.

Once a diagnosis of diaphragmatic rupture has been made, treatment is operative.

In this situation, in which large diaphragmatic rupture is strongly suspected, laparoscopy is typically avoided because insufflation of the abdomen may cause a tension pneumothorax.

However, laparoscopy may be useful to evaluate for a penetrating diaphragm injury. In the acute setting of a blunt diaphragmatic injury, laparotomy is the preferred operative approach. It allows reduction of any organs back into the abdominal cavity, as well as thorough inspection of all intraperitoneal contents.

In the acute setting, even large defects can be closed primarily with a # 0- or # 1-monofilament or braided nonabsorbable suture.

Diagnosing small diaphragmatic injuries can be difficult in as much as up to almost half of all patients have normal findings on physical examination at initial evaluation.

Missed diaphragmatic injuries tend to enlarge over time, which may lead to herniation and strangulation of abdominal organs.

Primary repair is not usually feasible because of the rapid atrophy of diaphragmatic muscle fibers.

A thoracotomy is generally performed for a chronic diaphragmatic hernia since it provides better access to the adhesions usually found in the chest.

How well did you know this?
1
Not at all
2
3
4
5
Perfectly
104
Q

A 46-year-old female presents after a high-speed head-on motor vehicle crash. The patient was wearing a seatbelt and is noted to have an abrasion across her neck and abdomen. CT of the abdomen demonstrates no solid organ injury and no pneumoperitoneum with trace free fluid in the pelvis. Abdominal examination is benign. What is the next appropriate management step?

A. Nil per os (NPO), admit for serial abdominal examinations

B. NPO, admit for serial CT scans

C. Diet and discharge home

D. Immediate repeat CT with PO contrast

E. Immediate exploratory laparotomy

A

ANSWER: A

COMMENTS: The incidence of a blunt small bowel injury is less than 1% of all traumatic injuries reported; however, it is associated with significant morbidity and mortality.

A high index of suspicion must be used when evaluating for a blunt bowel injury.

CT findings can be nonspecific with bowel wall thickening, fat stranding, mesenteric edema, and free fluid without solid organ injury.

The presence of seatbelt sign or a lumbar vertebral body fracture significantly increases the risk of a small bowel injury.

A patient without signs of peritonitis who is alert and appropriate can be monitored with clinical examinations.

CT with oral contrast has not been shown to enhance the detection of these injuries and can lead to a delay of treatment and risk aspiration.

If the patient has an unreliable examination, DPL may be attempted to look for amylase, bilirubin, particulates, or WBC > 500.

Alternatively, a diagnostic laparoscopy or laparotomy may be completed.

How well did you know this?
1
Not at all
2
3
4
5
Perfectly
105
Q

The patient from the previous scenario also has a CT angiogram of her neck performed. She is found to have a right carotid artery dissection. The patient has a normal neurologic examination with no deficits. No other injuries are present. What is the next step in management?

A. Operating room for neck exploration

B. Perform endovascular stenting

C. Anticoagulation with heparin and aspirin

D. No intervention, discharge home with follow-up CT angiogram in 1 week

E. Repeat imaging with formal angiogram to confirm known lesion

A

ANSWER: C

COMMENTS: Blunt cerebrovascular injury (BCVI) refers to inju- ries to the carotid and vertebral arteries. Patients at risk include those with a neck hematoma or arterial bleeding, bruit, neurologic deficit, CT brain with stroke, Lefort II or III fractures, basilar skull fracture involving the carotid canal, cervical spine fracture or ligamentous injury, diffuse axonal injury with GCS ≤ 6, and near- hanging with anoxia.

These patients should be screened with a CT angiogram of the neck. Some may require a traditional angiogram if CT images are inadequate.

Injuries are graded from I to V.

Grade I injury is a dissection with <25% luminal narrowing, while grade II is ≥25% narrowing, a thrombus, or intimal flap.

A pseudoaneurysm is a grade III lesion, and total occlusion is grade IV.

These are all treated with systemic anticoagulation with heparin, with a partial thromboplastin time (PTT) goal of 40 to 50 and an antiplatelet agent, aspirin or clopidogrel. Anticoagulation has decreased the incidence of stroke in these patients, which most often occurs during the first week after injury. Grade V injuries are transections with free extravasation and require immediate operative or endovascular management.

Repeat imaging is completed 7 to 10 days after the initial injury. If no lesion is seen, the injury has healed and antithrombotic therapy is stopped.

How well did you know this?
1
Not at all
2
3
4
5
Perfectly
106
Q

An 8-year-old child hits a curb with his bicycle, which causes him to flip over the handlebars. He had no initial symptoms and was monitored at home by his parents. However, 2 days after the incident, he begins having nonbilious emesis. He is brought to the ED and undergoes CT of the abdomen and pelvis, which demonstrates a duodenal hematoma. What is the next step in management?

A. Initiation of NPO status and gastric decompression with a nasogastric tube

B. EGD to assess for luminal compromise

C. Drainage of the hematoma via laparoscopy

D. Drainage of the hematoma via laparotomy

E. Resection of the injured portion of the duodenum with primary anastomoses

A

ANSWER: A
COMMENTS: Blunt injuries to the duodenum can be difficult to diagnose.

Duodenal hematomas typically occur up to 3 days after injury with a gastric outlet obstruction type of clinical picture.

The duodenal lumen is narrowed because of the hematoma itself and the associated edema. CT with oral contrast enhancement and upper gastrointestinal studies are useful in diagnosing this condition.

If no other indication exists for exploration, treatment is conservative and consists of placement of a nasogastric tube for decompression and parenteral nutrition.

Typically, these hematomas and their symptoms resolve within 7 to 15 days after injury.

Operative exploration is reserved for patients in whom the symptoms do not resolve within this period.

How well did you know this?
1
Not at all
2
3
4
5
Perfectly
107
Q

A patient with multiple gunshot wounds to the abdomen is taken to the operating room. On exploration she is found to have two small holes in the first part of the duodenum and a 2-cm lateral defect in the second part of the duodenum. The ampulla is intact and cannulated. Resulting pancreaticochol- angiogram demonstrates normal anatomy. What is the best management for this patient?

A. Primary closure with internal drainage

B. Pancreaticoduodenectomy

C. Gastrojejunostomy

D. Omental patch only

E. Serosal patch only

A

ANSWER: A

COMMENTS: Management options for duodenal injuries include repair, reinforcement, intraluminal decompression, and enteric diversion.

These can all be used in varying degrees depending on the nature of the injury and surgeon’s preference.

Two-layer primary repair may be completed for small injuries; however, with injuries involving the retroperitoneal duodenum or greater than 50% of the lumen, additional steps should be taken to prevent complications.

Primary repairs can be reinforced with additional tissue such as a rotational flap of peritoneum from the abdominal wall, omentum, or a piece of small bowel (Thal patch). Internal drainage to reduce intraluminal pressure can be completed by passing a nasogastric tube postpyloric into the proximal duodenum or retrograde from the jejunum into the proximal duodenum.

An enteric diversion while maintaining enteral nutrition can be completed with a Billroth II gastrojejunostomy or placement of a nasojejunal tube. A temporary alternative is a pyloric exclusion with a stapler or hand-sewn closure of the pylorus and a distal feeding jejunostomy; however, most open up within 4 weeks.

Pancreatico-duodenectomy is only indicated in trauma if the damage to both the duodenum and pancreas is beyond repair and the patient is hemodynamically stable.

Assessment of the pancreas and extrahepatic biliary tree is also mandated with duodenal injuries due to their close proximity to the duodenum.

How well did you know this?
1
Not at all
2
3
4
5
Perfectly
108
Q

A 44-year-old male presents with a single gunshot wound to the abdomen. He is hemodynamically stable and taken to the operating room. On exploration, his injuries are found to be limited to four small bowel injuries 8 cm apart, each with destruction of 20% of the bowel wall, and a through-and- through injury to the ascending colon with destruction of 30% of the bowel wall. How should these injuries be managed?

A. Resection and anastomosis of the small bowel injuries and primary repair of the colon injury

B. Primary repair of both the small bowel and colon injuries

C. Primary repair of the small bowel injuries, primary repair of the colon injury, and creation of a diverting ileostomy

D. Resection of the small bowel injuries and exteriorization of the colon injury as a colostomy

E. Resection and anastomosis of all injuries

A

ANSWER: A

COMMENTS: Treatment of bowel injuries relies on the amount of damage to the viscera and stability of the patient. Patients who are hemodynamically stable and have injuries that involve less than 50% of the circumferential bowel with no vascular disruption can undergo primary repair without the need for diversion.

Resection is indicated for injuries involving greater than 50% of the wall circumference, multiple injuries in a short segment, or both.

Anastomosis can be hand-sewn or stapled. However, there may be a slight decrease in the number of complications with a hand-sewn technique.

This can be completed with a running absorbable full- thickness suture for the inner layer and an interrupted silk suture for the outer seromuscular layer.

Complications occur in approximately 10% of cases and include anastomotic leaks, deep space abscesses, and enterocutaneous fistulas. They are increased in damage control cases and those with other intraabdominal injuries, especially pancreaticoduodenal injuries.

Broad-spectrum antibiotics should be given preoperatively and discontinued 24 h postoperatively.

How well did you know this?
1
Not at all
2
3
4
5
Perfectly
109
Q

A 42-year-old male unrestrained driver struck his steering wheel against his abdomen during a motor vehicle crash. He is hemodynamically stable but complains of abdominal pain. CT of the abdomen demonstrates a moderate amount of free fluid with no solid organ injuries. He undergoes an explor- atory laparotomy, at which time complete transection of the pancreatic neck is found. What is the most appropriate management of this injury?

A. Roux-en-Y pancreaticojejunostomy to the distal end of the pancreas with oversewing of the proximal pancreatic stump

B. Distal pancreatectomy with oversewing of the proximal pancreatic stump

C. Primary repair and drainage of the pancreatic duct

D. Pancreaticoduodenectomy

E. Total pancreatectomy

A

ANSWER: B
COMMENTS: Operative management of pancreatic injuries centers on the location of the injury and whether the duct is involved.

Approximately 50% of the pancreas is located on either side of the superior mesenteric artery. For pancreatic wounds with an intact duct, drainage of the area with soft closed suction drains suffices.

If the main pancreatic duct is injured to the left of the mesenteric vessels, as in this patient, distal pancreatectomy with drainage of the proximal stump is indicated.

The proximal pancreatic duct should be individually ligated with nonabsorbable suture, if possible, and the parenchymal tissue oversewn or stapled across with a stapler.

The spleen should be preserved if the patient’s hemodynamic status allows. Roux-en-Y pancreaticojejunostomy to the distal end of the pancreas with oversewing of the proximal pancreatic stump carries a high rate of leakage.

A pancreaticoduodenectomy and total pancreatectomy would be reserved for injuries that involve extensive devitalization of the duodenum and pancreas.

Primary repair is technically difficult and does not address the transected pancreatic tissue. Intraoperative cholangiography and pancreaticogram, or postoperative endoscopic retrograde cholangiopancreatography (ERCP) or magnetic resonance cholangio- pancreatography (MRCP), can aid in defining critical anatomy.

How well did you know this?
1
Not at all
2
3
4
5
Perfectly
110
Q

During exploratory laparotomy in a patient with multiple gunshot wounds to the abdomen, a through-and-through gunshot wound is noted in the left lobe of the liver. Brisk bleeding is seen from the bullet track. Which is an appropriate operative maneuver for this injury?

A. Pringle maneuver

B. Tractotomy

C. Omental packing

D. Large mattress sutures traversing the bullet track

E. All of the above

A

ANSWER: E

COMMENTS: With regard to hepatic injuries, there are three overall goals of treatment: (1) control of hemorrhage, (2) debridement of nonviable tissue, and (3) adequate drainage.

Multiple operative techniques can be used to establish control of bleeding, and often a combination of these techniques is used.

The Pringle maneuver is direct compression of the portal triad, either manually or with a vascular clamp. This is helpful in identifying whether the bleeding source is from the hepatic artery or portal vein versus hepatic veins or retrohepatic vena cava.

The clamping time should be minimized as much as possible with 5 min of flow for every 15 min clamped.

These patients tend to be hypovolemic and, as a rule, do not tolerate hepatic ischemia well. Omental packing is performed by first creating a pedicle of omentum and then placing it across or in the defect.

This creates a well-vascularized “packing” of the liver that also has its own natural hemostatic properties. Large mattress sutures using a 0-chromic on a blunt tipped needle work by compressing the bullet track with the surrounding liver parenchyma.

A tractotomy is the act of opening the already present wound to fully examine the track and identify the bleeding vessels. This then allows directed individual vessel ligation.

Historically, selective hepatic artery ligation has been used and involves ligation of the hepatic artery branch to the involved lobe. Although this is still a viable option, it is associated with a fairly high rate of abscess formation and hepatic necrosis.

How well did you know this?
1
Not at all
2
3
4
5
Perfectly
111
Q

A 32-year-old female is a restrained passenger in a high- speed motor vehicle crash. Initial workup shows an HR of 95 beats/min, blood pressure of 110/82 mmHg, and hemoglobin of 12.2 g/dL. On FAST, a moderate amount of fluid is seen in the RUQ, between the liver, kidney, and diaphragm. A grade IV liver laceration is seen on CT with no active extravasation. Her vital signs 6 h after admission are an HR of 100 beats/min and blood pressure of 105/80 mmHg. What is the next most appropriate step?

A. Exploratory laparotomy

B. Angiography

C. Repeated FAST

D. Diagnostic laparoscopy

E. Repeated hemoglobin determination

A

ANSWER: E

COMMENTS: The overall success rate for nonoperative management of blunt hepatic injuries is about 90%. Patients with grade IV and V injuries are able to be treated without surgery between 75% and 80% of the time.

Requirements for nonoperative therapy include hemodynamic stability, no signs or symptoms of peritonitis, and a transfusion requirement of no more than two to four units of packed red blood cells.

This patient is hemodynamically stable, and therefore further imaging with CT is warranted. The initial CT can not only localize the injury but, in the case of solid organ injuries, also provide information regarding active hemorrhage.

The patient should be closely monitored in an intensive care setting with serial abdominal examinations and hemoglobin determinations.

Angiography is a helpful adjunct to nonoperative treatment, but it is usually reserved for situations in which active extravasation or a “blush” is seen on CT.

Repeated CT is advised for patients who do experience a decrease in their hemoglobin to reevaluate the liver damage and to look for any active extravasation that would be amenable to angiographic embolization.

How well did you know this?
1
Not at all
2
3
4
5
Perfectly
112
Q

The patient in the previous scenario is discharged home on hospital day 4 without needing operative intervention. She returns to the clinic 2 months after discharge with persistent dull continuous RUQ pain. She denies any fevers or chills, and all laboratory studies, including a hepatic function panel, are within normal limits. CT of the abdomen and pelvis is performed and reveals a localized homogeneous fluid collection directly adjacent to the liver. What is the correct diagnosis and treatment?

A. Hemobilia: exploratory laparotomy and ligation of bleeding vessel

B. Hemobilia: ERCP with stent placement

C. Biloma: image-guided percutaneous drainage

D. Biloma: exploratory laparotomy with external drainage

E. Hepatic abscess: angiogram

A

ANSWER: C
COMMENTS: Because of the increasing number of patients with significant liver lacerations being treated nonoperatively, posttreatment complications are being encountered more often.

Such complications include hemobilia, biloma, hepatic necrosis, and abscess.

Hemobilia occurs when a connection exists between the biliary and arterial systems, and it is typically manifested as RUQ pain, melena, and jaundice. Hemobilia can be diagnosed by CT with IV contrast enhancement or upper endoscopy and is usually treated by angiography with embolization.

Patients with hepatic necrosis or abscess (or both) typically have RUQ pain, fever, leukocytosis, and, at times, localized peritonitis. It can be appreciated on CT with IV contrast enhancement as nonperfused liver parenchyma sometimes associated with a heterogeneous fluid collection. This condition warrants laparotomy with debridement.

Bilomas occur as a result of leakage of bile and typically close spontaneously over time. The fluid collections themselves are best treated with image-guided percutaneous drainage when localized, as in this patient. If the fluid collection is not amenable to percutaneous drainage, ERCP is recommended because biliary stents and sphincterotomy can reduce intrahepatic biliary pressure and increase healing.

How well did you know this?
1
Not at all
2
3
4
5
Perfectly
113
Q

A 22-year-old construction worker falls off of a ladder and lands on his left side. He fractures left ribs 6 to 12 and has a grade II splenic laceration. HR is 88 beats/min, and blood pressure is 110/76 mmHg. Hemoglobin is 12.3 g/dL. The patient is admitted to the ICU for serial hemoglobins and monitoring. Six hours later, the HR is 104 beats/min, blood pressure is 100/68 mmHg, and hemoglobin is 9.1 g/dL. What is the next step in management?

A. Exploratory laparoscopy

B. Exploratory laparotomy

C. Angiography

D. Continue serial examinations

E. Transfuse two units, and continue serial examinations

A

ANSWER: C

COMMENTS: The annual incidence of a blunt splenic injury is 40,000 patients, of which over 90% are managed without surgery.

Nonoperative management, including observation, serial hemoglobins and abdominal examinations, limited blood transfusions, and angiography with possible embolization, is utilized for the remaining majority of patients.

Although controversy exists regarding who should undergo angiography, the literature suggests that those with contrast blush or extravasation on initial CT or American Association for the Surgery of Trauma (AAST) grade III to V splenic injuries may benefit the most from intervention, reducing the need for delayed splenectomy.

This strategy is limited to hemodynamically stable patients with reliable examinations.

Complications associated with angiography and embolization include splenic abscesses, acute kidney injury, hematomas, and coil migration.

Patients should be observed in the hospital or can be discharged home after 24 to 72 h of monitoring, with strict instructions to return to the hospital for any signs of bleeding. Most patients who fail nonoperative management require a splenectomy within 5 days of the injury.

Transfusions of greater than two to four units of packed red blood cells, hypotension, tachycardia, or peritonitis are triggers to convert to operative management.

How well did you know this?
1
Not at all
2
3
4
5
Perfectly
114
Q

The patient in the previous scenario is being monitored in the ICU. HR is now 122 beats/min, blood pressure is 88/56 mmHg, and hemoglobin is 8.2 g/dL. What is the next step in management?

A. Exploratory laparoscopy

B. Exploratory laparotomy

C. Angiography

D. Continue serial examinations

E. Transfuse two units, and continue serial examinations

A

ANSWER: B

COMMENTS: Patients with hypotension, tachycardia, or other injuries that limit reliable serial examinations should undergo urgent splenectomy.

In this case scenario, the patient initially was appropriate for embolization but deteriorated despite intervention and became unstable, necessitating a trip to the operating room for a splenectomy.

Although giving a transfusion is warranted, due to the significant drop in hemoglobin, tachycardia, and hypotension, surgical intervention is indicated.

Patients who respond to resuscitation and become relatively stable in the operating room with a limited splenic injury may benefit from splenorrhaphy or partial splenectomy.

This can be completed by ligating the blood supply to the damaged part of the spleen, letting the tissue demarcate, and debriding nonviable tissue.

The remaining spleen can be wrapped in an absorbable mesh or primarily repaired by approximating the splenic capsule with a pledgeted mattress suture.

Patients who undergo splenectomy should be vaccinated against encapsulated organisms, specifically pneumococcus, meningococcus, and Haemophilus influenzae, prior to discharge.

Immune function remains preserved after embolization.

How well did you know this?
1
Not at all
2
3
4
5
Perfectly
115
Q

Which option provides the most appropriate management for the injury described?

A. Blunt mechanism: zone 3 nonexpanding retroperitoneal hematoma: exploration

B. Blunt mechanism: zone 2 expanding retroperitoneal hematoma: angiogram

C. Penetrating mechanism: zone 1 expanding hematoma: observation

D. Penetrating mechanism: zone 1 nonexpanding hematoma: exploration

E. Penetrating mechanism: zone 2 nonexpanding hematoma: angiogram

A

ANSWER: D

COMMENTS: The major abdominal vasculature is located in the retroperitoneum, which is divided into three zones.

Management of injuries is based on the mechanism, blunt versus penetrating, if there is a concern for ongoing bleeding, and which zone is involved.

Zone I is the central retroperitoneum and contains the aorta and its branches, the vena cava, and the proximal renal vessels.

The lateral areas are zone II and contain the kidney, proximal collecting system, and distal renal vessels.

Zone III is the pelvis and its con- tents including the iliac vessels.

Most hematomas resulting from blunt trauma should not be explored other than zone 1 injuries or pulsatile expanding hematomas.

All penetrating injuries must be explored due to the risk of injury to other retroperitoneal organs such as the duodenum, pancreas, colon, rectum, and bladder.

The only exception is a stable retrohepatic hematoma.

Obtaining proximal and distal control prior to exploration of a hematoma is ideal but not always possible.

With blunt trauma, angiography with embolization may be used as an adjunct in nonexpanding hematomas.

How well did you know this?
1
Not at all
2
3
4
5
Perfectly
116
Q

A patient presents with multiple gunshot wounds to the extremity and right lower quadrant. In the operating room, upon entry into the abdomen, a large amount of hemoperitoneum is encountered and all quadrants are packed immediately.

Upon reinspection, the patient is noted to have an injury to the infrahepatic vena cava. How can you best expose this injury?

A. Transect the right renal artery

B. Right medial visceral rotation

C. Left medial visceral rotation

D. Transect the pancreas

E. All of the above

A

ANSWER: B

COMMENTS: Rapid exposure and control of abdominal vessels are critical in patients with life-threatening hemorrhage.

The principles of proximal and distal control should always be attempted prior to entering a contained hematoma.

However, once bleeding has started and the injury is identified, direct pressure with a finger may allow for temporary control.

Left medial visceral rotation, which requires the division of the lateral attachments of the colon and spleen allowing for mobilization to the right toward midline, exposes the aorta and the left renal hilum.

The distal aorta and iliac arteries can be exposed by lifting up the transverse colon and dissecting at the base of the mesocolon.

The IVC and right renal hilum are best approached with right medial visceral rotation, which divides the colon and duodenum’s lateral peritoneal attachments allowing for mobilization of the ascending and transverse colon, duodenum, and pancreas cephalad and to the left toward the midline.

Suprahepatic IVC may require the division of the diaphragm or even a sternotomy for control within the pericardial sac. Supraceliac control of the aorta at the diaphragm can temporarily curb the bleeding when the patient is in extremis and can be used while attempting definitive vascular control.

The vena cava can be controlled with sponge sticks placed proximal and distal to the injury. There are a few special maneuvers that can be utilized for repair of intraabdominal vascular injuries. Exposure of the back wall of the vena cava can be done through an anterior venotomy, allowing for repair.

Superior mesenteric artery injuries may require transection of the pancreas for exposure, and the right iliac vein may require transection and subsequent repair of the right iliac artery for exposure.

How well did you know this?
1
Not at all
2
3
4
5
Perfectly
117
Q

A 23-year-old male presents with multiple gunshot wounds to the abdomen and right leg. During exploration, the patient is found to have a grade IV liver laceration and multiple enterotomies. He also has a destructive injury to his proximal superficial femoral artery with no distal pulses. Anesthesia tells you that the patient’s temperature is 36°C; he has received 10 units of product and is on vasopressors. Most recent arterial blood gas (ABG) shows a pH of 7.15 and a lactic acid that is twice the normal value. Which of these is a trigger to transition to damage control surgery in this patient?

A. Body temperature less than 37°C

B. Arterial pH less than 7.2

C. Base deficit greater than 6

D. Hemoglobin less than 7

E. Oxygen saturation less than 90%

A

ANSWER: B

COMMENTS: Damage control surgery is a staged resuscitation strategy with three phases that include the initial operation, resuscitation in the ICU, and return to the operating room for definitive treatment.

Goals for the initial procedure include control of hemorrhage with sutures and packing, limiting fecal contamination and restoring critical blood flow. In the ICU, the patient is aggressively resuscitated with blood products, fluids, and vasopressors with the goal of obtaining a normal physiologic state.

Once the patient is stable, a plan can be made to return to the operating room for definitive surgical management and evaluation for missed injuries. The trigger to employ a damage control strategy is metabolic failure defined by hypothermia, metabolic acidosis, and coagulopathy despite hemorrhage control.

Published guidelines include an arterial pH of ≤7.2, lactic acid > 5 mmol/L, base deficit < −15 in a patient younger than 55 years or 50% normal.

How well did you know this?
1
Not at all
2
3
4
5
Perfectly
118
Q

In the previous scenario, what is the best treatment option for the superficial femoral artery injury?

A. Ligation of the artery

B. Placement of temporary intravascular shunt

C. Definitive repair with a reversed saphenous vein
interposition graft from the contralateral leg

D. Definitive repair with an in situ saphenous vein interposition graft

E. Definitive repair with a polytetrafluoroethylene (PTFE) graft

A

ANSWER: B

COMMENTS: The patient in the above scenario is in extremis and requires a quick establishment of distal perfusion.

Although ligation is an option, it puts the lower extremity at risk of ischemia and amputation.

Placement of vascular shunts require proximal and distal control, debridement to clean edges, thrombectomy with a Fogarty catheter, regional heparinization, proper shunt selection, securing the distal end first with a 0 silk tie to allow for back bleed- ing, and then securing the proximal end to allow reestablishment of flow.

Fasciotomy should be considered based on ischemia time. This will allow perfusion and an increased chance for limb salvage while other life-threating injuries are being managed.

Dwell times have ranged from 3 to 36 h and do not require systemic anticoagulation. They are routinely used for peripheral arterial injuries but have also been selectively used for extremity venous and truncal vascular injuries.

Definitive repair can be completed during the next trip to the operating room with the best choice being a reversed saphenous vein interposition graft from the contralateral leg.

How well did you know this?
1
Not at all
2
3
4
5
Perfectly
119
Q

The patient is a 52-year-old male involved in a high-speed head-on motor vehicle crash. He arrives to the ED unstable with BP 80/60 mmHg and HR 125 beats/min. Initial x-ray workup reveals an open-book pelvis fracture. Despite 2 L of crystalloid, the patient remains tachycardic and hypotensive. What techniques can be used to control bleeding in this patient?

A. Interventional radiology (IR) angiogram

B. Preperitoneal packing

C. External fixator

D. Pelvic binder

E. All of the above

A

ANSWER: E

COMMENTS: Pelvic injuries are present in approximately 10% of blunt trauma patients. Pelvic fractures can be classified accord- ing to the force vector that caused the injury: anteroposterior compression, lateral compression, or vertical shear fractures.

The overall mortality in these patients is 6%. However, it increases to greater than 30% when patients present with hemorrhagic shock.

Bleeding is usually from branches of the internal iliac artery or the extensive venous plexus surrounding the sacrum.

Early control of pelvic hemorrhage is critical. Placement of a pelvic binder or external fixator works by decreasing the volume of the pelvis and temporarily stabilizes the fractured bones, preventing additional injury.

Preperitoneal packing completed through a low midline or supra- pubic incision carried through the fascia allows for three rolled lap pads to be placed into the preperitoneal space on each side of the bladder, resulting in volume reduction and tamponade of bleeding.

This can be done in the operating room in conjunction with a laparotomy being completed for other injuries through a separate incision.

Bilateral internal iliac artery embolization with a temporary agent, such as gelatin material, is the preferred angiographic treatment in unstable patients.

This provides quick control by limiting flow through the large number of collaterals feeding the pelvis and allowing time for a clot to form.

REBOA is now being used in a limited number of centers as an adjunct for early control of pelvic bleeding and increasing mean arterial pressure in patients presenting in hemorrhagic shock with isolated pelvic trauma.

How well did you know this?
1
Not at all
2
3
4
5
Perfectly
120
Q

For the patient in the previous scenario, a pelvic binder is placed and blood pressure and HR improves. A negative urethrogram is completed, and a Foley catheter is inserted. The urine output is bloody, and a cystogram demonstrates a bladder injury. Which combination of injury and management is correct?

A. Extraperitoneal bladder injury = operating room for primary repair

B. Extraperitoneal bladder injury = suprapubic catheter placement

C. Intraperitoneal bladder injury = operating room for
primary repair

D. Intraperitoneal bladder injury = maintain Foley catheter

E. Intraperitoneal bladder injury = cystoscopy

A

ANSWER: C

COMMENTS: Approximately 5% of pelvic fractures will have an associated bladder injury. The majority of these will present with hematuria and can be diagnosed with a CT or traditional cystogram.

Contrast extravasation into the retroperitoneum is seen as flame- shaped densities surrounding the bladder, which is characteristic of an extraperitoneal injury.

With an intraperitoneal injury, contrast leaks into the abdomen, outlining loops of bowel.

Approximately 80% of injuries are extraperitoneal and can be treated with a Foley catheter for 10 to 14 days.

An intraperitoneal bladder injury requires operative exploration of the bladder, identification of ureteral ori- fices and bladder neck, and a two-layer repair with absorbable suture.

A closed suction drain is left next to the repair, and the Foley catheter remains in place for 10 to 14 days.

A repeat cystogram is completed at 10 days to evaluate the healing process. The Foley catheter can be removed at that time if no extravasation is noted.

How well did you know this?
1
Not at all
2
3
4
5
Perfectly
121
Q

A 22-year-old man undergoes proctoscopy and exploratory laparotomy for a transpelvic gunshot wound. A 2-cm, partial-thickness laceration is found in the distal portion of the extraperitoneal rectum. What is the appropriate surgical management of this injury?

A. Resection of the injured area and anastomosis with a diverting colostomy

B. Primary repair with a diverting colostomy

C. Diverting colostomy only

D. Presacral drainage only

E. Primary repair only

A

ANSWER: C
COMMENTS: Repair of rectal injury largely depends on location, intraperitoneal versus extraperitoneal.

The posterior rectum and distal third of the anterior rectum are not serosalized, and an injury in these regions is considered extraperitoneal. Extraperitoneal rectal injuries should be treated chiefly by fecal diversion.

Nondestructive intraperitoneal lacerations that are less than 50% of the circumference of the rectal wall should be repaired primarily after debridement of any devitalized tissue in the absence of peritonitis.

Presacral drainage, which historically had been used rather rou-tinely, has been decreasing in use and has not been shown to decrease the complication rate.

How well did you know this?
1
Not at all
2
3
4
5
Perfectly
122
Q

A 27-year-old female presents with multiple gunshot wounds to the abdomen. She is immediately taken to the operating room for an exploratory laparotomy. She is found to have a mid-ureter injury with a 2-cm segment loss. During the surgery, the patient becomes hypotensive, requiring vasopressors. What is the best damage control treatment option for an unstable patient with a ureteral injury?

A. Ligation of the injured ureter

B. Percutaneous nephrostomy

C. Ureteral drainage via a single-J stent, which is externalized to the skin

D. Placement of a bridging stent

E. All of the above

A

ANSWER: E

COMMENTS: When a trauma patient is unstable on the operating room table and damage control has been initiated, time should not be spent on primary repair of a ureteral injury.

Surgical options for this type of situation consist of simple ligation of the ureter, placing a percutaneous nephrostomy through the renal parenchyma into the renal pelvis, inserting a catheter into the proximal end of the damaged ureter and bringing it out through the wound, or placing a catheter or stent in the proximal and distal ends of a small-segment ureteral injury.

These methods will allow for urinary diversion until the patient is stable.

How well did you know this?
1
Not at all
2
3
4
5
Perfectly
123
Q

A 30-year-old male is brought to the ED after being involved in a motorcycle crash. He is hemodynamically stable. Blood is noted at the urethral meatus. On portable pelvic radio- graphs, he is found to have bilateral pubic rami fractures. He has not yet voided since admission. Which of the following is the best next step?

A. Wait for the patient to void freely before attempting transurethral bladder catheterization.

B. Initially attempt gentle transurethral bladder catheterization but stop if resistance is encountered.

C. Obtain a urethrogram before attempting transurethral bladder catheterization.

D. Insert a suprapubic cystostomy tube.

E. Perform CT of the pelvis with three-dimensional reconstruction.

A

ANSWER: C

COMMENTS: Approximately 10% of all patients with a pelvic fracture have a concomitant urethral injury.

Findings on physical examination, such as blood at the meatus, perineal ecchymosis or hematoma, or inability to void, should raise suspicion for a urethral injury.

If any of these signs or symptoms is present, or there is a significant anterior pelvic fracture, a urethrogram should be obtained to exclude an injury before transurethral catheterization is attempted.

Bladder decompression and drainage are the mainstays of treatment of urethral injuries, either via suprapubic cystostomy for complete disruption or with a bridging transurethral catheter for partial tears.

How well did you know this?
1
Not at all
2
3
4
5
Perfectly
124
Q

A 36-year-old male presents after a prolonged extrication from a motor vehicle crash. The patient has bilateral femur fractures and an open left tibia-fibula fracture with significant tissue loss and no distal pulses. Which state- ment is correct regarding the management of a mangled extremity?

A. This patient requires angiography prior to going to the operating room.

B. A mangled extremity severity score ≥ 5 is predictive of need for amputation.

C. In a hemodynamically unstable patient, the definitive vascular repair should be completed first.

D. Bony injury should be reduced prior to definitive vascular repair.

E. Optimal maximum time to surgery should be less than 24 h from injury.

A

ANSWER: D

COMMENTS: A significant injury to a limb requires assessment of multiple factors including bone, soft tissue, nerve function, vasculature, and overall hemodynamic status to assist with the decision for limb salvage versus amputation.

Although multiple scoring systems, including the mangled extremity severity score (MESS) and the Ganga Hospital Score, have been created to help evaluate and manage mangled extremities, none have been predictive to determine which limb can be salvaged versus amputated.

Despite these findings, experts recommend considering primary amputation with a MESS ≥ 7, delay of greater than 6 h to treatment, and in hemodynamically unstable patients.

Imaging with CT angiogram or formal angiography may assist the surgeon. However, in the setting of “hard signs,” including ongoing hemorrhage, expanding hematoma, a bruit or thrill, signs of ischemia, or absent pulses, patients should be taken directly to the operating room for exploration.

Orthopedic injuries are reduced first and temporarily stabilized, which may unkink vessels and allow for reperfusion and better evaluation of a soft tissue injury.

How well did you know this?
1
Not at all
2
3
4
5
Perfectly
125
Q

The patient in the previous scenario is taken to the ICU for postoperative monitoring. It is noted that his urine output has decreased in the past 2 h and is very dark. Urinalysis is positive for blood, but there are no red blood cells on microscopic analysis. What is true regarding his diagnosis?

A. Best treatment includes aggressive IV fluid resuscitation.

B. All patients with rhabdomyolysis require dialysis.

C. The renal failure from rhabdomyolysis typically resolves within 3 to 5 days.

D. Severe hyponatremia is a frequent complication.

E. Alkalinization to a pH between 8 and 9 is an important treatment goal.

A

ANSWER: A

COMMENTS: Rhabdomyolysis is the breakdown of muscles that causes a release of myoglobin.

Myoglobin causes tubular obstruction and vasoconstriction, which leads to acute tubular necrosis and acute kidney injury.

There are a variety of causes of rhabdomyolysis including crush injury, multiple trauma, seizures, illicit drugs, and several medications, including many antibiotics and antipsychotics.

Creatine kinase is the most specific marker for diagnosis, and higher levels correlate with a greater injury.

Additional tests show the described characteristic urinalysis, hyperkalemia, hypocalcemia, and elevated creatinine.

Treatment includes preventing additional muscle breakdown, aggressive IV fluid resuscitation, and managing complications including electrolyte imbalances and renal failure, which may require renal replacement therapy.

Indications for dialysis include hyperkalemia, metabolic acidosis, and hypervolemia.

Treatment with bicarbonate infusion has been used to alkalinize urine to a pH of 6 to theoretically prevent myoglobin precipitation in the tubules; however, there are minimal data to support this practice.

Overall prognosis with early and aggressive volume resuscitation is good, with most patients recovering to baseline renal function within 2 weeks to 3 months.

How well did you know this?
1
Not at all
2
3
4
5
Perfectly
126
Q

A 34-year-old male presents with a gunshot wound to his left thigh with no pulses in his feet. The patient is taken immediately to the operating room, and a superficial femoral artery injury is repaired with a saphenous vein interposition graft. At the end of the surgery, the patient had equal pulses bilaterally and was neurologically intact. Five hours postoperatively, the left distal pulses are diminished and he begins to experience pain with passive dorsiflexion and extension. Which of the following statements is true regarding compart- ment syndrome in an extremity?

A. Fractures are the cause of approximately 30% of all compartment syndromes.

B. The lateral compartment of the lower part of the leg is the most commonly affected.

C. A compartment pressure of 25 mmHg negates a need for fasciotomy.

D. Loss of pulses is an early clinical development.

E. A four-compartment fasciotomy should be performed.

A

ANSWER: E

COMMENTS: Compartment syndrome of the extremity is an extremely important diagnosis to make as early as possible because of the significant risk for permanent limb dysfunction and potential loss.

Causes of compartment syndrome include crush injury, reperfusion after a time of ischemia, and fractures, which account for 50% of cases.

The anterior compartment of the lower part of the leg contains mostly type I (slow twitch) muscle fibers and is encased by dense fascia, thus making it most vulnerable to the development of ischemia.

The diagnosis is largely clinical, with pain out of proportion to the findings on examination, pallor, paresthesias, paralysis, poikilothermia, pulselessness, and tense compartments being the hallmark symp- toms.

Because of the fact that even after an hour of ischemia, impulses can still be conducted through peripheral nerves, par- esthesias are a late sign of compartment syndrome.

In addition, even though compartment pressures of 30 mmHg or higher are classically quoted, lesser pressures do not prove that there is adequate tissue perfusion, and if other signs or symptoms of compartment syndrome are present, fasciotomies should still be performed.

How well did you know this?
1
Not at all
2
3
4
5
Perfectly
127
Q

A 24-year-old male presents after a motorcycle crash. He is found to have multiple left-sided rib fractures, a grade II splenic laceration, and a small 2-mm subdural hematoma. His GCS score is 15. The patient is admitted to the ICU for serial hemoglobin and neurologic checks. Clinical examination, hemoglobin, and repeat CT of the head at 48 h remain stable. What is the most appropriate deep vein thrombosis (DVT) prophylaxis to initiate at this time?

A. Early ambulation only

B. Sequential compression devices only

C. Retrievable IVC filter

D. Subcutaneous heparin 5000 units daily

E. Subcutaneous Lovenox 30 mg twice a day

A

ANSWER: E

COMMENTS: Trauma patients, especially those with multiple injuries, including intracranial hemorrhage, pelvic and extremity fractures, and spinal injuries, are at significant risk of having a DVT.

This is also a population that has an increased risk of bleeding, and standard prophylaxis protocols have been modified to account for this.

All patients should have sequential compression devices upon admission, and chemical prophylaxis with either heparin 5000 U three times per day or enoxaparin 30 U twice a day should be started as early as possible.

LMWH has been shown to be superior in this patient population.

Patients with truncal hemorrhage with stable hemoglobin values for 24 h are at minimal risk of bleeding with the initiation of chemical prophylaxis.

Controversy has been placed around patients with traumatic brain injuries. Many recent studies have shown that it is safe to start chemical prophylaxis as early as 24 h after a stable brain CT without increased risk of progression of hematoma.

Few patients will have contraindications to pharmacologic prophylaxis and should be considered for weekly extremity surveillance duplex ultrasound or placement of a retrievable IVC filter.

Risks of placing an IVC filter include dislodgement, perforation, and thrombosis. These should be removed at the earliest occasion when pharmacologic prophylaxis becomes appropriate and the risk of bleeding is minimized.

How well did you know this?
1
Not at all
2
3
4
5
Perfectly
128
Q

A 79-year-old female falls at home. In the ER, she is found to have a pelvic fracture with a large retroperitoneal hematoma. While waiting for angioembolization by Interventional Radiology (IR), the patient becomes hypotensive and is transfused with two units of packed red blood cells and two units of fresh-frozen plasma. After IR, the patient starts to complain of shortness of breath, has desaturations, and requires intubation. Which statement best describes a transfusion-related acute lung injury (TRALI)?

A. Onset occurs 24 h after a transfusion.

B. Onset occurs most often after transfusion with fresh- frozen plasma.

C. Clinical findings include dyspnea, hypertension, and bradycardia.

D. Treatment includes plasmapheresis.

E. Affected patients cannot receive blood transfusions in the
future.

A

ANSWER: B

COMMENTS: The reported incidence of TRALI varies from 1% to 15% of patients receiving transfusions with a mortality rate of 5%–10%. It occurs 50 to 100 times more often in critically ill patients.

It can occur after transfusion of any component of blood but is most associated with fresh-frozen plasma and red blood cells.

Several hypotheses exist regarding its pathogenesis, including human leukocyte antigen–specific antibody in the donor directed against an antigen in the recipient, resulting in an inflammatory response causing damage to the endothelial lining of the lungs.

Criteria for diagnosis include acute onset, usually within 6 h of transfusion, hypoxemia with a PaO2/FiO2 ratio less than 300, and bilateral infiltrates on chest x-ray in a patient with no alternative risk for lung injury.

Symptoms include dyspnea, tachypnea, tachycardia, hypotension, and fever. Treatment is supportive, with approximately 70%–90% of patients requiring ventilator support.

Being diagnosed with TRALI does not affect the ability to receive blood transfusions in the future. However, risk reduction maneuvers including restrictive transfusion policies and washing of stored blood products should be employed with future transfusions.

How well did you know this?
1
Not at all
2
3
4
5
Perfectly
129
Q

A coal mine explodes in a rural part of the country. There is only one small local hospital, and critical patients will need to be airlifted to the closest Level 1 Trauma Center. Which patient should be triaged to the expectant category?

A. A 22-year-old male with a broken femur

B. An 18-year-old female with 90% total body surface area (TBSA) burns with agonal breathing

C. A 38-year-old male with multiple left rib fractures and decreased breath sounds

D. A 29-year-old female with a traumatic amputation of her right forearm

E. A 34-year-old male with a fractured thoracic spine and traumatic paralysis

A

ANSWER: B

COMMENTS: A mass casualty incident involves an influx of hundreds of casualties that overwhelms the capacity and resources of a hospital, resulting in suboptimal and delayed care.

Field triage is defined as sorting casualties according to their needs, considering their chance of survival, and is key in providing the greatest benefit to the maximum number of patients in these emergencies.

Sort, Assess, Lifesaving Interventions, Treatment/Transport (SALT) triage allows for quick determination of which patients will benefit the most from immediate hospital transfer.

Approximately 15% of patients will be critically injured in any event, and limited resources such as ICU beds, ventilators, and operating rooms need to be directed toward these critical patients with a high likelihood of survival with intervention.

Causalities defined as expectant will vary depending on casualty numbers, type and severity of injuries, and resource availability.

To minimize a large resource expenditure on a patient with a low likelihood of survival, these patients are kept out of the hospital until the immediate surge subsides.

How well did you know this?
1
Not at all
2
3
4
5
Perfectly
130
Q

During the annual Memorial Day parade of your hometown, a large explosion occurs along the parade route, with multiply injured persons and fatalities at the scene.

Prehospital EMS providers, first responders, and bystanders begin to administer first aid to those injured and start transfer of patients to the local hospital. As an on-call general surgeon, you and your colleagues stand ready in the emergency department while preparing for a potential mass casualty event. Hospital administrators have activated the disaster/mass casualty plan, triage officers are designated, and the incident command plan is in effect.

  1. A pre-hospital provider encounters an injured female with a below knee traumatic amputation and multiple truncal fragment wounds with labored breathing (Figure 67-1). The first priority in the initial care of this patient should be?

A. Airway

B. Breathing

C. Circulation/control of life-threatening hemorrhage

D. Disability

E. Evacuation

A

C.

Traditional teaching of advanced cardiac life support (ACLS) and advanced trauma life support (ALLS) concepts stress the “ABCDE” mantra (airway, breathing, circulation, disability, and exposure), in which airway control is paramount.

However, in the setting of combat injuries and civilian catastrophes such as blast injuries, immediate control of life-threatening hemorrhage takes precedence as fatal hemorrhage can rapidly occur while addressing other elements of the primary survey.

Hemorrhage remains the leading cause of potentially preventable morbidity and mortality in both military and civilian trauma, reinforcing the importance of pre-hospital efforts to control bleeding and prevent exsanguination after severe injury.

The experiences gained during combat operations in Iraq and Afghanistan have
led to a change in the priorities of treatment that are taught to combat medics and first-responders.

As the most likely cause of potentially preventable death in these scenarios is hemorrhage, the standard ATLS approach of focusing first on the airway has been changed to a primary focus on C, or the control of hemorrhage, as the first priority. T e next priorities would be on assessing the airway and breathing for life-threatening issues such as airway obstruction, tension pneumothorax, or an open pneumothorax (“sucking chest wound”).

How well did you know this?
1
Not at all
2
3
4
5
Perfectly
131
Q

Which of the following pairs of hemorrhage source and preferred management is correct?

A. Junctional hemorrhage—Tourniquet placed directly over the injury

B. Extremity hemorrhage—Hemostatic dressing and direct pressure

C. Truncal hemorrhage—Permissive hypotension and hemostatic resuscitation

D. Pelvic hemorrhage—Early use of rFactor Vila

E. Intra-cranial hemorrhage—Reversal of hypercoaguable state and neurosurgical intervention

A

C

Major extremity hemorrhage can be effectively controlled in both the pre-hospital and hospital setting using an effective proximal tourniquet. Application of tourniquets to control major hemorrhage should be placed as distal as possible yet proximal to the site of injury, to control hemorrhage and limit the extent of tissue ischemia.

Hasty tourniquets applied in combat under lire or prior to moving a patient to
a safer location may be placed expediently anywhere proximal on the injured extremity and later moved distally.

Approximately one-third of bleeding deaths after explosive events arc due to extremity hemorrhage, with the remaining two-thirds attributed to junctional bleeding or non-compressible truncal hemorrhage (Figure 67-2).

“Junctional” injuries are defined as injuries to the groin, axilla, neck, or
perineum. These injuries present major challenges to hemorrhage control as damaged deep vascular structures transition from the extremities to major
cavities beyond the effective reach of a proximal tourniquet.

Direct pressure and topical hemostatic adjuncts are often the only currently available treatments of potential benefit before proximal surgical control can be achieved. Non-compressible truncal hemorrhage is the most feared and fatal type of bleeding as it is not amenable to pre-hospital hemorrhage control measures, and requires higher-level interventions such as surgery or angioembolization.

Significant pelvic hemorrhage may result from complex bony injuries and damage to the pelvic vasculature. Treatment of pelvic fractures may include
pelvic sheeting, commercial or improvised binders, and forms of external fixation, that aim to stabilize the pelvic ring. Surgical control of pelvic hemorrhage includes open and catheter based therapies as well as pre-peritoneal pelvic packing to tamponade hemorrhage.

Permissive hypotension is the concept that avoiding normal or elevated blood pressure in a bleeding patient prior to surgical hemorrhage control limits blood loss.

The goal is to maintain an adequate arterial pressure for critical organ perfusion while limiting overpressurization from vigorous fuid resuscitation that
may exacerbate hemorrhage.

Although there was initial interest in the use of activated recombinant factor VII (rFactor Vila) as an adjunctive therapy for bleeding, subsequent studies have found little to no benefit and it also would not be used as a prehospital therapy for pelvic bleeding.

The treatment of traumatic intracranial hemorrhage begins with reversal of any identified coagulopathy, not hypercoagulable state, treatment of associated cerebral edema, and possible neurosurgical interventions.

Distribution of preventable bleeding deaths from battlefield wounds by site of hemorrhage:

Torso 48%
Extremity 31%
Axilla/Groin 21%

How well did you know this?
1
Not at all
2
3
4
5
Perfectly
132
Q

The principles of damage control surgery include which of the following?

A. Primary focus upon control of hemorrhage only

B. Avoiding the use of temporary abdominal closures

C. Appropriate triage based upon injuries and available resources

D. Colostomy for all colon injuries

E. Definitive repair of all abdominal injuries

A

C.

Damage control surgery (DCS) is a concept that includes the principles of limiting further physiologic insult to a severely injured patient through therapid surgical control of life-threatening hemorrhage and enteric spillage, to minimize further bleeding, restore tissue perfusion, and prevent sepsis. The goal is to avoid prolonged, initial operative procedures that may further exacerbate hypothermia, coagulopathy, and/or acidosis.

Temporary closures of the abdomen and thorax may be employed to shorten
the procedure length and allow for urgent or planned re-operation in this population. Once bleeding and gastrointestinal spillage arc controlled, there are no further attempts to address other non-life-threatening abdominal injuries at the initial surgery.

The operation is terminated and the patient is returned to the ICU for resuscitation and restoration of normal physiology and coagulation. Following
physiologic restoration these patients arc taken back to the operating room for definitive treatments and closure.

In mass casualty situations and resource-constrained environments, DCS allows limited surgical personnel to provide critical surgical procedures to severely injured patients rapidly, and then either evacuate them to higher levels of care or clear the operating room for additional patients.

How well did you know this?
1
Not at all
2
3
4
5
Perfectly
133
Q

The concept of damage control or hemostatic resuscitation includes which of the following?

A. Limited use of colloid solution resuscitation

B. 3:1:1 PRBC:FFP:Platelet transfusion strategy

C. Empiric transfusion of cryoprecipitate

D. Administration of tranexamic acid (TXA) for treatment of hyper fibrinolysis

A

D. Damage control resuscitation (DCR) or hemostatic resuscitation is the concept of limiting further blood loss before surgical hemorrhage control is obtained, as well as replacing shed-blood volume with a balanced blood product based resuscitation that restores the capacity to carry and deliver oxygen and addresses derangements ol the coagulation system.

DCR research has focused upon improved morbidity and mortality with limitation of crystalloid infusion to prevent hemodilution (colloid solutions
are, in general, not advocated for trauma resuscitations), acidosis and worsening of coagulopathy, in favor of transfusion ratios approaching 1:1:1 of
packed red blood to fresh frozen plasma and platelets.

Additional targeted treatment of specific coagulation abnormalities based upon traditional studies (PT/INR, PTT, fibrinogen) and point of care testiing such as thromboelastography is recommended, to avoid unnecessary transfusion and appropriate resource utilization.

Permissive hypotension is the concept that avoiding normal or elevated blood pressure in a bleeding patient prior to surgical hemorrhage control limits
blood loss.

The goal is to maintain an adequate arterial pressure for critical organ perfusion while limiting over-pressurization from vigorous fuid resuscitation
that may exacerbate hemorrhage. Previous studies in civilian trauma patients have confirmed a favorable survival advantage by limiting pre-hospital resuscitation but heterogeneity in clinical trials such as optimal blood pressure target and f uid resuscitation type have yet to yield definitive conclusions.

Current research has identified early derangements of the coagulation system that appear to confer an increased risk of morbidity and mortality after
trauma, termed the acute coagulopathy of trauma.

Hyperfibrinolysis is a key component of this condition. and targeted pharmacologic intervention with the antifibrinolytic agent, tranexamic acid (TXA), has been linked to a survival advantage in several studies of bleeding trauma patients. The use of TXA in the setting of major hemorrhage and massive transfusion situations is now recommended in the early treatment (< 3 hours from injury) in civilian and combat injuries.

How well did you know this?
1
Not at all
2
3
4
5
Perfectly
134
Q

Primary blast injuries include which of the following?

A. Damage to gas filled organs due to extreme pressure changes

B. Truncal injury sustained as victim is thrown through the air by blast

C. Crush injuries from structural collapse around the patient

D. Penetrating injury from debris and fragments propelled by the blast force

A

A.

Blast injuries are traditionally classified based upon the discrete mechanism of injury involved. Primary blast injuries involve damage to gas-filled organs such as the intestine, lungs, and middle ear as well as the brain.

These injuries result from overpressurization due to the blast force. Rupture of the tympanic membrane is a frequently encountered primary blast injury, but the absence does not rule out other types of blast injury.

Secondary blast injuries result from injury due to flying debris and fragments
that are propelled by the blast. These are the most frequently sustained type of blast injuries.

Tertiary injuries occur when a person is thrown through the air by the blast, while Quaternary injuries include other blast effects such as burn injury, inhalation of toxic gases, or injury due to collapse of structures around the person.

Lastly, quinary injuries are the result of contamination by chemical, radiologic, or biologic materials from the blast itself or other injured persons.

In addition to the injury classification described above, the severity of injury sustained after a blast mechanism depends upon the patient’s proximity to
the blast, the energy of the blast itself, and whether or not the blast occurred in a closed space. Persons injured by a blast in an enclosed space such as a
vehicle or building frequently sustain a higher rate of lethal primary blast injury as well as increased frequency ot secondary penetrating injuries.

How well did you know this?
1
Not at all
2
3
4
5
Perfectly
135
Q

A 35-year-old man is brought to the emergency department after an altercation at a bar. He has a 3 cm stabwound to his right chest, approximately 5 cm lateral to and just superior to his nipple. He is able to tell you that the wound is from a knife. He is oriented and appropriate, although intoxicated, and complains loudly of pain in his right chest. His initial vital signs are as follows: HR 96, BP 110/63, RR 20,98% saturation on room air.

What is your first step in the management of this patient?

A. Perform a tube thoracostomy.

B. Perform the primary survey and examine for any other wounds.

C. Perform a FAST (focused assessment with sonography for trauma) scan to identify intraabdominal injury.

D. Obtain a chest radiograph to identify presence of pneumothorax or hemothorax.

A

B.

In any trauma patient, the first priority is to ensure stable A,B,C’s, airway, breathing, and circulation, which may include emergent interventions (e.g.,tube thoracostomy).

In this patient with stable vital signs, the initial step is to perform a primary and secondary survey—a brief head to toe physical exam. A common mistake in the setting of penetrating trauma is failure to identity multiple injuries.

Common locations for missed penetrating injuries can be in the back, axilla, or perineum, so a complete head to toe survey is critical.

Most patients can be stabilized enough to undergo a secondary survey, but any problems identified on the primary survey should be immediately addressed. Once all injuries arc identified, they can be prioritized for diagnostic imaging or operative repair.

Both chest X-ray and FAST scan can be used as adjuncts to the secondary survey, but they do not replace the need lor a head-to-toe assessment of the
patient. In the initial evaluation of a trauma patient, priorities are:

  1. identifying all wounds,
  2. determining if urgent lifesaving intervention is indicated, and
  3. determining if additional testing is needed.
How well did you know this?
1
Not at all
2
3
4
5
Perfectly
136
Q

Despite fluid resuscitation, the patient’s blood pressure suddenly becomes 60/palpable. His trachea is midline and his breath sounds are reduced over the right hemithorax, with dullness
to percussion. What is the best next step in management?

A. Place a 36F chest tube into the right pleural cavity at the 2nd interspace in the midaxillary line.

B. Place a 36F chest tube into the right pleural cavity at the 5th interspace in the mid- or anterior axillary line.

C. Place an 18 g angiocath into the 2nd intercostal space at the midclavicular line.

D. Take the patient immediately to the operating room for thoracotomy.

E. Initiate a massive transfusion protocol to stabi¬
lize the patient for computerized tomography
(CT) scan to identify the source ol the hypoten¬
sion.

A

B.

In the setting of penetrating trauma to the chest, the differential diagnosis for hypotension includes tension pneumothorax, cardiac tamponade, and hemothorax.

Classically, a tension pneumothorax will present with deviation of the trachea away from the injury, increased percussion and decreased breath sounds with distended neck veins. However, these can be late findings and may be difficult to appreciate in a busy and noisy trauma bay.

If a tension pneumothorax is suspected, the first step would be placing
an large bore (14 or 18 gauge) angiocath to needle decompress the thoracic cavity. The diagnosis would be confirmed by a rush of air from the needle, and a chest tube should then be placed.

In this patient, the absence of tracheal deviation along with dullness to percussion makes a hemothorax the more likely diagnosis, and a chest tube the appropriate next step.

The preferred method for tube thoracostomy is to enter the “safe triangle” bounded by the anterior border of the latissimus, the lateral border of the pectoralis major, and a horizontal line at the level of the nipple (males) or infra-mammary crease (females). This positioning minimizes the likelihood of entering the abdominal cavity, injuring muscle or breast tissue, or underlying structures such as the internal mammary artery, and avoids the major chest wall musculature that can cause significant pain with chest tube insertion.

How well did you know this?
1
Not at all
2
3
4
5
Perfectly
137
Q

Which of the following is a correct indicator for operative intervention paired with an appropriate operative approach for this patient?

A. Greater than 500 cc blood from the chest tube upon initial placement—right posterolateral thoracotomy in the operating room.

B. Cardiac arrest in the trauma bay—left anterolateral thoracotomy in the emergency room.

C. Persistent transfusion requirements to maintain stable blood pressure—median sternotomy in the emergency room.

D. Second entry wound identified below the costal margin—transverse anterior thoracotomy (clamshell incision) in the operating room.

E. An entry wound media) to the midclavicular
line—median sternotomy in the emergency room.

A

B.

Accepted indications for emergency department thoracotomy in the setting of penetrating thoracic trauma are loss of pulses with previously witnessed cardiac activity and unresponsive, persistent, hypo¬
tension.

In blunt trauma, indications are rapid exsanguination from the chest tube (> 1500 cc on initial placement with unresponsive hypotension.

Survival rates after emergency thoracotomy are up to 30% for penetrating trauma, but closer to 1% for blunt trauma patients.

The goals of an emergency department thoracotomy are to control hemorrhage (which may require cross-clamping the descending thoracic
aorta), allow access for internal cardiac massage, and treat potential cardiac tamponade. Access to the heart, descending aorta, and mediastinum requires a left thoracotomy incision.

Regardless of the side of injury, traumatic arrest warrants a left thoracotomy for access to the mediastinal structures. If necessary, the incision can be extended to the right chest (clamshell thoracotomy) for access to the right thoracic cavity.

Stable patients with penetrating trauma may stillrequire exploration in the operating room. Traditionally, anterior abdominal stab wounds warranted
mandatory laparotomy to rule out intraabdominal injury, although recent evidence has raised the possibility of expectant management for the stable patient with penetrating injuries.

Thoracic injury to “the box,” the area defined by nipples, sternal notch, and
xiphoid process generally warrants operative exploration as well.

In a stable patient, however, it is reasonable to proceed with additional diagnostic studies. Although some have advocated an extensive workup including esophagoscopy, bronchoscopy, and a pericardial window, more recent data indicates that a high-quality CT scan provides excellent imaging of the thorax and mediastinum, and additional studies can be performed based on the clinical exam and CT findings.

In a patient who undergoes tube thoracostomy, indications to proceed to the operating room are:
> 1500 cc on initial placement, > 150-200 cc/hr for
> 3 hours, or hemodynamic instability requiring persistent blood transfusions.

How well did you know this?
1
Not at all
2
3
4
5
Perfectly
138
Q

If this patients wound were located in the neck above the cricoid cartilage (but below the angle of the mandible), which of the following diagnostic tests would be definitely indicated?

A. None; manage expectantly with admission for serial exams.

B. Computed tomography (CT) of the neck with CT angiography (CTA) of the cervical vessels.

C. CTA or angiography plus bronchoscopy.

D. CTA or angiography, bronchoscopy, and esophagogastroduodenoscopy (EGD).

A

B.

Zone I of the neck extends from the clavicles to the cricoid cartilage.

Zone II is from the cricoid cartilage to the angle of the mandible, and Zone III is from the angle of the mandible to the skull base.

The majority of penetrating injuries, as with the patient in this case, occur in Zone II.

For unstable patients with signs of major vascular or airway injury, management of penetrating neck trauma involves securing an airway and proceeding directly to the operating room for neck exploration.

These signs include stridor, massive subcutaneous air, gurgling or bubbling
through the wound, hemoptysis/hematemesis, and pulsatile bleeding or rapidly expanding hematoma.

Historically, all Zone II injuries that breached the platysma mandated operative exploration. This is no longer widely practiced, and stable patients with
no hard signs of vascular or airway imaging can be managed expectantly with appropriate diagnostic tests.

CTA has similar sensitivity to operative exploration for identifying vascular and soft tissue injuries. CT imaging can also provide valuable information about potential injuries to the esophagus or trachea that can guide the selective use of additional studies to evaluate these structures.

Esophageal injuries are frequently asymptomatic, and morbidity significantly increases if repair is delayed beyond 24 hours.

Contrast esophagography or esophagoscopy is recommended for all Zone II injuries that breach the platysma and have cither physical exam or CT scan
findings concerning for an esophageal injury. Similarly, flexible bronchoscopy can be used selectively based on any exam or CT imaging findings concerning for airway injury.

How well did you know this?
1
Not at all
2
3
4
5
Perfectly
139
Q

The patient has 800 cc initial drainage from the right-sided chest tube, which then slows down over the next few hours. He is admitted to the floor on telemetry for monitoring, and remains stable. A follow up chest radiograph the next morning demonstrates significant residual basilar fluid. What is the most appropriate next step?

A. Go to the operating room tor a video-assisted thoracoscopic surgery (VATS) to evacuate the retained hemothorax.

B. Go to the operating room for a right anterolateral thoracotomy.

C. Place a pigtail catheter to drain the residual hemothorax.

D. Observation and serial chest X-rays.

A

A.

Retained hemothorax is a relatively commonproblem among patients who present with a moderate or large volume hemothorax with either blunt or penetrating trauma.

Although observation only is an option, this carries the risk of developing an infected hematoma (empyema) or fibrothorax. If there is still a significant amount of clotted blood in the thoracic cavity that is not adequately drained by the initial chest tube, then there are several options for management.

These include placement of a second chest tube (or removal and replacement of the initial chest tube) in a better position to evacuate the fluid collection or
administration of local lytic therapy (TPA or urokinase administered through the chest tube) to attempt to break up the clot and allow drainage through the
chest tube.

Although these can be effective in select cases, they have been associated with relatively high failure rates. Since the retained hemothorax likely consists of clotted blood, a small-bore pigtail catheter is unlikely to achieve adequate drainage. There has been a trend toward the increased use of early
VATS to evacuate the hematoma and place a well-positioned chest tube under direct visualization.

This approach can also be useful if there is suspicion for an associated diaphragm injury that can be repaired simultaneously. Median sternotomy would not be indicated for evacuation of a retained hemothorax.

How well did you know this?
1
Not at all
2
3
4
5
Perfectly
140
Q

You wake to the sound of your pager and the familiar refrain overhead, ‘Trauma team to the trauma room. Trauma team to the trauma room’

Your pager reads: 23YOM GSW TO CHEST/ABD/RT THIGH. GCS 15 HR 100 BP 90/P RR30 98%’NRB. ETA 3MIN.

As you reach the trauma bay, your patient arrives in extremis. He is unresponsive, his breathing is agonal with oxygen saturation in the low 80s despite ventilation with a bag valve mask,and his radial pulse is weakly
palpable with a rate of 105 beats per minute. Paramedics have established two large bore peripheral IVs and have begun infusing two liters of normal saline.

They report that the patient was shot three times at close range with an unknown firearm. The first wound in the right upper chest was characterized as “sucking” on the scene and has been dressed with an occlusive dressing, taped on three sides. They have already performed a needle decompression. The second wound is in the left lower quadrant of the abdomen and is hemostatic. The final wound is located in the right mid-thigh and is presently
hemostatic. Paramedics tell you that the bleeding was “pulsatile” prior to the application of a tourniquet.

  1. Your first priority for this patient should be:

A. Establishing a definitive airway.

B. Performing a chest X-ray.

C. Removing the tourniquet on the patient’s right lower extremity.

D. Placing a left-sided tube thoracostomy.

E. Placing a central venous catheter.

  1. Your patient undergoes a rapid sequence intubation, has a left-sided tube thoracostomy placed with prompt return of air and 1800 mL blood, and has the remainder of the two liters of crystalloid infused via EMS. You remove the tourniquet and immediately appreciate pulsatile bleeding. Distal right lower extremity pulses are not palpable prior to the re-application of this device and
    with resultant hemostasis. The patients heart rate has increased to 120 beats per minute wit a blood pressure of 86/44. With your primary survey completed, which of the following injuries necessitates urgent operative intervention?

A. Gunshot wound to the chest only.

B. Gunshot wound to the abdomen only.

C. Right lower extremity injury only.

D. Right lower extremity injury and the chest injury.

E. Right lower extremity injury, abdominal injury,
and chest injury.

A

Gunshot wounds fall under the broad classification of penetrating trauma and comprise up to 10% of all major trauma in the United States.

The energy imparted to tissue can be calculated using the kinetic energy equation:
Ke= ½m(∆v)2,

where m is the mass of the projectile and ∆v, is the change in velocity before and after contact.

Simplistically, it can be inferred that higher caliber firearms and those projecting higher velocities will cause more grievous injury. This assumes, of course, that the entirety of the projectile’s kinetic energy is imparted to the tissue. Those bullets that enter and exit do not expend all of their kinetic energy on the body.

Furthermore, the projectile’s behavior in the body is a function of its relative density. Lower density projectiles (e.g., lead or so-called “hollow-point” bullets) will tend to expand, creating a progressively enlarging wound tract
with a comparatively small entrance wound and a large exit wound. High density and/or high velocity projectiles tend to pass directly through tissue but can cause significant indirect injury via cavitation. Even further, high density tissue like bone can dramatically alter the initial trajectory, deflecting the projectile in nearly any direction. It is imperative that patients who have sustained gunshot wounds undergo a systematic evaluation with complete anatomic exposure and physical examination.

  1. A.

The primary survey on trauma patients follows an algorithmic approach to prevent overlooking potentially life-threatening pathology. Patients with gunshot wounds arc no exception. Evaluation and stabilization of the patient^ airway to ensure adequate oxygenation and ventilation represents the first
step in resuscitation.

This sort of vignette is ubiquitous on board examinations and the admonition is to “Keep it Simple,” by following the algorithm delineated by Advanced Trauma Life Support.

Remember the acronym ‘ABCDE’ for the primary survey. This involves assessing the patient^ airway and maintaining in-line cervical spine immobilization, ensuring adequacy of the patient^ breathing by assessing
both oxygenation (via pulse oximetry) and ventilation (evaluating respiratory rate and effort), circulation (pulse examination, addressing life-threatening
bleeding), disability (neurologic examination), and exposure (strip patient and perform rapid scan for injuries). It is important to be mindful that with a team approach in a working trauma, much of the above can be performed simultaneously.

For our patient, his unresponsiveness, agonal breathing, and oxygen desaturation are concerning and a definitive airway is indicated.

While a chest X-ray will be performed and the left-sided needle decompression will need to be supplanted by a tube thoracostomy, these are of subsidiary importance to securing the patientk airway. A central venous catheter is not necessary at this time given the presence of two large bore peripheral IVs. While it will be important to take down the patient’s tourniquet to perform
a detailed extremity evaluation, the report of pulsatile bleeding in the field is suggestive of vascular injury and given present hemostasis, this can be deferred until the initial resuscitation has been performed.

E.

Knowing the indications for operative intervention is essential to caring for patients having sustained traumatic injuries. For penetrating thoracic trauma, immediate resuscitative thoracotomy (the so-called “Emergency Department T oracotomy”) is indicated for witnessed pulseless electrical activity (overall survival 4% to 5% for gunshot wounds vs. 18% to 24% for stab wounds).

Urgent thoracotomy (within 1 to 4 hours of admission) is indicated for: initial chest tube output > 1500 mL, evidence of ongoing bleeding at a rate of 200-300 mL/h, massive air leak, or cardiac tamponade.

For abdominal trauma, indications for laparotomy include hemodynamic instability, peritonitis on examination, or evisceration. Strong consideration
should also be given to exploration for those patients with an abdomen that cannot be evaluated clinically (e.g., due to altered mental status, distracting injury, paralytic/sedative administration, etc.).

Hard signs of arterial injury include pulsatile hemorrhage, cxpanding/pulsatile hematoma, bruit or thrill over wound, absent distal pulses, or evidence to suggest extremity ischemia (pallor, poikilothermia, pain, paralysis). So-called ‘soft signs” include non-expanding hematoma, peripheral nerve deficit, history of pulsatile hemorrhage at the time of injury, and unexplained hypotension.

Any patient presenting with hard signs of vascular injury should undergo prompt exploration as the positive predictive value of physical examination for arterial injury approaches 100%.

In the absence of hard signs, an alternate means of performing a bedside evaluation is with the injured extremity index (analogous to an ankle-brachial index, also known as an arterial pressure index) with a normal value of > 0.9 having a reported sensitivity/specificity of 95/97% respectively for major vascular injury. A normal physical examination and injured extremity index virtually exclude major arterial injury.

How well did you know this?
1
Not at all
2
3
4
5
Perfectly
141
Q

Regarding penetrating chest trauma, which of the following is correct?

A. Tamponade physiology requires accumulation of at least 150cc of blood in the pericardial space.

B. 85% of injuries can be managed with tube thoracostomy alone.

C. Great vessel injuries arc common in penetrating chest trauma.

D. Lung injury that requires operative intervention is more common following blunt injury.

E. Prophylactic antibiotic use reduces the incidence of post-traumatic empyema in the setting of retained hemothorax.

A

B. Most thoracic injuries can be managed with tube thoracostomy; only 10% to 15% of thoracic trauma requires operative intervention. Tamponade physiology classically only requires 50cc of blood in the pericardial space and is characterized clinically by Beck’s Triad (jugular venous distention, muffled heart sounds, hypotension), occasionally with a narrowed pulse pressure, and the presence of an effusion on FAST examination.

Patients with tamponade physiology should receive aggressive volume administration because they will be dependent on their preload to generate
cardiac output.

Decompressive pericardiocentesis may be performed though in the setting of trauma, but this is often a temporizing measure. Definitive surgical management in this setting of trauma involves the creation of a pericardial window.

The reported incidence of great vessel injury in penetrating chest trauma is only 4%, as the majority of these patients exsanguinate prior to presentation. Gunshot wounds to the mediastinum should raise suspicion and the diagnosis can be confirmed with CT angiography. If present, urgent exploration is warranted.

Lung injury that requires surgical intervention
is more commonly encountered with penetrating trauma. T c majority oi these injuries can be man¬
aged with pulmonary tractotomy (for penetrating
non-hilar injuries) or nonanatomic stapled resec¬
tions. Inadequately evacuated hemothorax can result
in secondary in lection (post-traumatic empyema)
or fibrothorax (entrapped lung). Unfortunately,
prophylactic antibiotics do not appear to prevent
the secondary development of infection in this set¬
ting. Chest tubes arc often unsuccessful in remov¬
ing clotted blood and consideration should be given
to operative exploration for patients with retained
hemothorax.

How well did you know this?
1
Not at all
2
3
4
5
Perfectly
142
Q

Regarding this patient’s penetrating abdominal wound, which of the following is correct?

A. Injuries to the bowel may be primarily repaired if less than 75% of the bowel’s circumference being involved.

B. The most common organ injured is the large bowel.

C. FAST (focused assessment with sonography for trauma) examination is poor at detecting hollow viscous injuries.

D. All patients with penetrating abdominal injuries that violates the posterior fascia must undergo an exploration but a laparoscopic one may suffice.

E. The operation of choice in an unstable patient having sustained penetrating abdominal injury is a limited laparotomy with extension only if
injuries are suspected.

A

C.

While a FAST examination is a useful bedside adjunct to detect the presence of free intra-abdominal or pericardial fluid, it is operator dependent and has a poor sensitivity with respect to hollow viscous, retroperitoneal, and diaphragmatic injuries.

A negative FAST examination docs not rule out intra-peritoneal injury. There is evidence to suggest that hemodynamically stable patients with abdominal gunshot wounds and no evidence of peritonitis on exam may undergo evaluation via computerized tomography of the abdomen to determine whether further operative intervention is required. Even if the posterior fascia is violated, elective non-operative management with observation and serial examinations is a safe alternative to reflex operative exploration in a major trauma center with in-hospital surgical support.

In general, primary repairs can be done on both small and large bowel if 50% or less of the circumference is damaged. The most commonly injured abdominal organ with penetrating trauma is the small bowel. For unstable patients with penetrating abdominal injuries, a full laparotomy incision should be made so that all of the injuries can be identified quickly.

How well did you know this?
1
Not at all
2
3
4
5
Perfectly
143
Q

A 60-year-old male restrained driver is involved in a high-speed head-on motor vehicle collision. He is heavily intoxicated and uncooperative, therefore he is intubated at the scene and brought to the nearest Level I trauma center. Upon arrival he is able to shrug his
shoulders to questions. Initial vita) signs indicate a pulse of 48 bpm and a blood pressure of 78/39 mm Hg. His respiratory status appears stable on the current ventilator settings and initial primary trauma evaluation fails to reveal any major external signs oi injury.

Subsequent secondary trauma survey reveals the patient is able to shrug his shoulders but demonstrates no motor or sensory function below his deltoids. Questionable rectal tone is present. A cervical collar is in place, but the patient demonstrates tenderness to palpation in the midiinc of the cervical spine.

  1. What is the most likely cause of this patient’s hypotension?

A. Splenic rupture

B. Acute blood loss

C. Flail chest

D. Spinal shock

E. Beta-blockade

A

D.

Spinal cord injury following a traumatic cervical spine injury typically results from blunt compression injury to the cord itself. Typically the central grey matter is affected first while the peripherally located white matter fiber tracts may be relatively spared.

If the spinal cord injury is severe enough, neurogenic or spinal shock may occur. The most basic definition of this is inadequate tissue perfusion due to paralysis of vasomotor input, most commonly due to loss of sympathetic tone and significant disruption of the vasodilator and vasoconstrictor balance. It is commonly characterized by bradycardia, hypotension, decreased peripheral vascular resistance, and decreased cardiac output.

Common physical examination findings demonstrate flaccid paralysis of the extremities with no sensory function, lack of rectal tone, lack of foley
catheter sensation, and priapism in males. If the spinal cord injury occurs at C5 or above then respiratory depression may occur.

As this patient demonstrates some deltoid function, this indicates a lower cervical cord injury. Presence of a shoulder shrug should not fool the examiner. T is motor function is provided by the 11th cranial nerve.

The most common classification of spinal cord injuries occurs via the ASIA system:

A = Complete (No motor or sensory function is preserved in
the sacral segments S4-S5)

B = Incomplete (Sensory but no motor function is preserved below the neurological level and includes the sacral segments S4-S5)

C = Incomplete (Motor function is preserved below the neurological level, and more than half of key muscles below the neurological level have a muscle grade less than 3)

D = Incomplete (Motor function is preserved below the neurological level, and at least half of key muscles below the neurological level have a muscle grade of 3 or more
E = Normal
How well did you know this?
1
Not at all
2
3
4
5
Perfectly
144
Q

Which of the following is not a component evaluated by the NEXUS criteria for clearing a patient’s cervical spine from injury such that the cervical collar can be removed?

A. Pain with passive motion ol the patient’s neck

B. History of Ambien use

C. Any other injury

D. Altered mental status

E. High velocity mechanism ol injury

A

E.

Any patient suspected of a cervical spine injury and therefore potentially unstable cervical spine should be placed in cervical immobilization with a rigid collar. Maintenance of in-line cervical alignment during intubation as well as log-roll precautions must be upheld during transfers.

Per the NEXUS criteria, a patient can be cleared with a 99.8% negative predictive value for cervical spine injury if the following criteria are met:

Meets all low-risk criteria?

  1. No posterior midline cervical-spine tenderness
  2. No evidence of intoxication
  3. Anormal level of alertness
  4. No focal neurologic deficit
  5. No painful distracting injuries
YES =  No Radiography
NO = Radiography
How well did you know this?
1
Not at all
2
3
4
5
Perfectly
145
Q

Which of the following therapeutic measures is considered a current standard treatment option for this patient’s spinal shock?

A. Limited boluses of normal saline or LR to prevent spinal cord edema

B. Bed rest

C. Selective vasopressor treatment to keep MAPs >85

D. Administration of methylprednisolone bolus at 30 mg/kg followed by continuous infusion at 5.4 mg/kg/hr for 23 hours

A

C.

Current treatment protocols for spinal shock involve the mainstays of aggressive fluid resuscitation and vasopressors. Vasopressor choice is left to the discretion of the treating physician based on the patient’s co-morbid factors. A goal MAP of > 85 sustained for 7 days has demonstrated some promise for improving a patient’s outcome.

Usage of high dose steroids in acute spinal cord injury is a controversial topic and has been for quite some time. Various large trials (NASCIS I, II, III) have demonstrated mild benefits with the administration of methylprednisolone bolus of 30 mg/kg followed by 23 hours or continuous infusion at 5.4 mg/kg/hour.

However, these studies have also demonstrated significant deleterious side effects with high dose steroids leading to the Congress of Neurosurgeons to declare that high dose steroids’ risks outweigh their benefits, and are thus
not recommended.

How well did you know this?
1
Not at all
2
3
4
5
Perfectly
146
Q

Imaging reveals a fracture dislocation of the midcervical spine with a resultant spinal cord injury. Computed tomographic angiography (CTA) reveals a traumatic vertebral artery dissection. Which of the following is associated with a markedly increased chance of cervical vascular injury and thus requires an evaluation with CTÄ?

A. Neurological examination out of proportion with computerized tomography (CT) head findings

B. Seat belt sign

C. C5 spinous process fracture

D. Glasgow coma scale < 10

E. Le Fort I fracture

A

A.

Blunt carotid or vertebral artery injuries can be a potentially lethal injury if missed upon initial trauma evaluation. Level II evidence exists neck CTA screening in trauma patients with an exam out of proportion to their cranial imaging (i.e., a comatose patient with minimal traumatic intracranial damage). Level III evidence exists for such screening in trauma patients with GCS 8 or below, petrous bone fractures, diffuse axonal injury, C1-3 fractures, any cervical spine fracture with subluxation, cervical spine fracture through the foramen transversarium, or Le Fort II/III injuries.

Surprisingly, despite its widely believed association with cervical vascular injuries, presence of an isolated “seat belt sign” with no other above listed injuries is only associated with a 1% yield for blunt cervical vascular injuries. Treatment of such blunt injuries attempt to prevent intracranial ischemic injuries.

Previously aggressive treatment with open surgical or endovascular repair, often in the form of stenting or coiling, provided the mainstay of treatment. However, current emerging evidence suggests these lesions, if diagnosed prior to evidence of ischemia, can be quite benign if treated with antithrombotic
medications.

Currently, either anticoagulation via heparin/Coumadin or anti-platelet medications via aspirin 325 mg daily are widely used.

How well did you know this?
1
Not at all
2
3
4
5
Perfectly
147
Q

A 32-year-old male, unrestrained driver, is involved in a high-speed motor vehicle collision. After a prolonged extraction, the patient is found to be somnolent and is intubated on the scene. After arrival in the trauma center, a complete evaluation reveals a non-displaced linear skull fracture and multiple areas of intracranial contusions with diffuse edema but no large mass lesions. No other systemic injuries are discovered. His neurologic exam demonstrates small but reactive pupils, presence of corneal, gag, and cough reflexes, lack of eye opening to voice or painful stimulation, brisk withdrawal of both legs and his right arm to painful stimulation as well as localization of his left arm to painful stimulation.

What is the patients Glasgow Coma Scale (GCS)?

A. 5

B. 6

C. 7

D. 8

E. 9

A

C.

The Glasgow Coma Scale (GCS) is the most basic method of quickly communicating a patient’s neurologic examination. It is comprised of three components.

Thus, a non-responsive comatose patient has a GCS of 3 while an awake, oriented patient would demonstrate a GCS of 15.

Intubated patients automatically receive just 1 point for verbal response. If a
patient’s motor exam is asymmetric, the best response is used for grading. Thus this patient’s exam is E1, V1, M5 for a total GCS of 7. For intubated patients, many providers use the convention of adding a “T” following the GCS score, so this patient would be described as a “GCS 7T”!

How well did you know this?
1
Not at all
2
3
4
5
Perfectly
148
Q

Which of the following are indications for the use of intracranial pressure monitoring in traumatic brain injury?

A. Intracranial injury demonstrated on computerized tomography (CT) scan and GCS of 8 or below.

B. Lack of intracranial injury on CT scan, but age> 40, SBP < 90, and motor posturing on exam.

C. Minor neurologic injury but lack of neurologic examination due to need for systemic paralytics in treatment of other injuries.

D. All of the above.

E. None of the above.

A

D.

Insertion of an intracranial pressure monitor can be used to evaluate and treat elevated ICP or as a surrogate for a neurologic examination in those who require sedation or chemical paralysis for other injuries (such as a trauma patient with an open abdomen on paralytics).

The most common indication for an ICP monitor is a patient with a traumatic intracranial injury on imaging and a GCS of 8 or less. These patients will have an elevated ICP 60% of the time.

Patients with a normal head CT scan will only demonstrate elevated ICP 13% of the time. This occurs most commonly in patients over 40 years of age who demonstrate posturing on examination and hypotension.

How well did you know this?
1
Not at all
2
3
4
5
Perfectly
149
Q

Which of the following should be implemented in this patient following external ventricular drain placement and admission to the ICU?

A. Elevation of the head of bed to up to 10 degrees so measurement of the intracranial pressure (ICP) remains calibrated

B. Maintain hemoglobin levels > 10 g/dL

C. Maintenance of a cerebral perfusion pressure of 60 or above

D. Allow hypercapnia

E. Administration of high levels of positive end-expiratory pressure (PEEP) to aid in oxygenation

A

C

The Monro-Kellie doctrine states that the skull is a fixed compartment with 3 basic components: brain (80%), cerebrospinal fluid (10%), and blood (10%).

An increase in any one of these components requires an equal decrease in one or both of the other two to prevent an increase in intracranial pressure. Typical cerebral blood flow compromises 15-20% of the cardiac output; a decrease in this may result in unmet cerebral metabolic demands.

The primary goal of treatment of severe brain injury is to maintain adequate cerebral blood flow and oxygen delivery, but it is dif cult to readily measure these parameters. However, cerebral blood flow can be roughly estimated by
cerebral perfusion pressure (CPP = MAP - ICP), and is commonly used to guide therapy.

In uninjured patients, the brain will maintain cerebral perfusion via autoregulation over estimated CPP ranges of 50 to 150.

However, patients with severe brain injury will commonly exhibit loss of autoregulation, which means that their cerebral blood flow will be directly
dependent on an adequate mean arterial pressure (MAP) and a normal intracranial pressure (ICP).

This is why there is so much attention paid to maintaining a normal to slightly elevated blood pressure and aggressive measures to avoid ICP elevations in
patients with severe brain injury.

All attempts at treating elevated intracranial pressure keep this Monro-Kellie hypothesis and its relationship with cerebral perfusion pressure in mind.

The first steps toward the treatment of a patient with elevated intracranial pressure begin with many simple bedside maneuvers. Initial steps should include elevation of the head of bed to aid cerebral venous outflow, seizure prophylaxis, maintenance of hemoglobin of at least 7 g/dL or above, avoidance of elevated intra-adominal pressure, and adequate pulmonary support.

Pulmonary support should focus on avoidance of hypoxia, hypercapnia, and excessive PEEP. Elevated levels of PaCO2 lead to intracranial vasodilation and cerebral blood volume, thus elevated ICP. Excessive PEEP may lead to increased intra-thoracic pressure and thus impaired cerebral venous outflow.

However, patients with severe brain injury will commonly exhibit loss o autoregulation, which means that their cerebral blood ow will be directly dependent on an adequate mean arterial pressure (MAP) and a normal intracranial pressure (ICP). T is is why there is so much attention paid to maintaining a normal to slightly elevated blood pressure and aggressive measures to avoid ICP elevations in patients with severe brain injury.

How well did you know this?
1
Not at all
2
3
4
5
Perfectly
150
Q

On post-admission day 2, the patient’s intracranial pressures rise to the low 30s. Which of the following maneuvers can be used to reduce his intracranial pressure?

A. Increased sedation with propofol only because other sedatives can increase cerebral metabolic demand

B. 3% hypertonic saline

C. Permissive hypotension

D. Transient periods of hypoventilation

E. Administration of methylprednisolone

A

B.

If elevated ICP occurs, more aggressive methods of treatment include increased sedation (via propofol or barbituates to decreased cerebral metabolic demands), paralytics, mannitol or both rheologic and osmotic effects, and hypertonic saline.

Steroids should be avoided in intracranial trauma. Despite the usage of dexamethasone with other causes of cerebral edema, steroids are associated with elevated mortality in trauma patients with severe brain injury due to the myriad of side effects. Hypotension should be avoided to help maintain the CPP.

How well did you know this?
1
Not at all
2
3
4
5
Perfectly
151
Q

A 27-year-old female presents to the emergency department after a motorcycle crash. She was hemodynamically unstable on arrival and is currently receiving 2L of crystalloid via large bore peripheral IVs. Glasgow Coma Scale (GCS) is 12. She is breathing spontaneously and has no obvious extremity trauma. Initial radiographs demonstrate normal cervical alignment without fracture, right side ribs 4 through 8 fractured, a small right side pulmonary contusion, no free air under the diaphragm, and a pelvis film that shows widening ofthe pubic symphysis by 4 cm along with diastasis of the left sacroiliac joint. Abdominal ultrasound is negative for free fluid. The patient is currently on her menses.

  1. With regards to the pelvic injury, the patient’s hemodynamic instability is most likely related to:

A. Disruption of the anastomis of the external iliac/deep epigastric and obturator vessels (the Corona Mortis)

B. Arterial injury of the superior gluteal artery and vein at the level of the greater sciatic notch

C. Post-traumatic closed soft tissue degloving injury in which the skin and subcutaneous tissue separate from the fascia superficial to the underlying musculature (Morel-Lavallee lesion)

D. Disruption o the anterior sacral venous plexus.

E. Rupture o the pelvic oor structures (sacrospinous and sacrotuberrous ligaments) with vaginal laceration by the anterior pubic symphasis ragments

A

D.

Hemodynamic instability after blunt trauma is due to ongoing hemorrhage until proven otherwise, with the common locations being the chest, abdomen, pelvis, or extremity/cxtcrnal bleeding.

The normal chest X-ray and abdominal ultrasound in conjunction with the abnormal pelvis X-ray make the pelvis the most likely source of bleeding in this
patient.

Although arterial injuries in association with major pelvic trauma can occur, the majority of bleeding occurs at the venous plexus located on the anterior aspect of the sacrum and sacroiliac joints.

This historically has responded best to pelvic packing orfracture reduction, rather than immediate angiography. Injuries to the superior gluteal arteries posteriorly and the Corona Mortis anteriorly have been reported and can be devastating, but they are far outnumbered by the venous lacerations of the presacral venous plexus.

Morel-Lavallee lesions are common after blunt force trauma, but do not typically lead to hemodynamic instability seen from the deep pelvic venous lacerations.

How well did you know this?
1
Not at all
2
3
4
5
Perfectly
152
Q

What percentage of vascular injuries from high-energy pelvic trauma are arterial in nature?

A. <5%

B. 10% to 15%

C. 50%

D. 60% to 75%

E. >85%

A

B.

The current management of pelvic fracture patients who are hemodynamically unstable consists of aggressive resuscitation, mechanical stabilization, and angioembolization.

Despite this multidisciplinary approach, mortality rates of these high-risk patients can exceed 40%.

Initial maneuvers to reduce the pelvic fracture and decrease the pelvic volume include placement of a pelvic binder, wrapping the pelvis with a sheet, or application of an external fixation device.

Preperitoneal pelvic packing (PPP) via laparotomy can directly address the venous bleeding that compromises 85% of pelvic fracture hemorrhage and docs not respond to the initial maneuvers listed above.

The remaining 15% of vascular injuries are arterial and may best be addressed via angiography in the event of continued hemodynamic instability following pelvic packing.

153
Q

Regarding the hemodynamic instability of pelvic fractures, what is the fracture pattern associated with the highest severity of hemorrhage?

A. Anteroposterior compression (APC III) fracture patterns

B. Lateral compression (LC III) fracture patterns

C. Vertical sheer (VS) fracture patterns

D. Combined APC/VS patterns

E. Open pelvic fractures

A

A.

In an antero-posterior compression fracture (APC), the fracture propagates from anterior to posterior. This widens the symphasis pubis and depending on the level of force, can widen the anterior sacroiliac joint or disrupt it entirely.

In an APC III injury, the pelvic floor ligaments (sacrospinous and sacrotuberous) are disrupted and can leading to instability and increased volume in the pelvic cavity. Disruption and displacement of the sacroiliac joint has the potential
to lead to massive hemorrhage from the anterior sacral venous plexus that is closely approximated to the joint.

Major hemorrhage in patients with pelvic fractures can reliably be predicted based on a pulse greater than 130, a hematocrit of 30 or less, and wide
diastasis of the pubic symphisis.

Although pelvic binders are now routinely applied for suspected or proven pelvic fractures, they may have no benefit or may even cause additional harm in select types of fractures.

The AP pelvic radiograph will identify injuries that may benefit from provisional stabilization with external compression with a sheet or pelvic binder.

In general, LC injuries will not respond to binder placement, whereas APC and VS injuries will.

Compression of an LC injury is potentially damaging as it may induce additional lateral compression and fracture dislocation or bleeding. Occasionally, an
LC injury in an elderly patient may have a hemorrhage associated with vascular or visceral disruption; these patients will not benefit from wrapping or binding, but may be candidates for angiography.

154
Q

Regarding the mortality of displaced pelvic fractures, the highest mortality rates arc seen in:

A. Anteroposterior compression (APC III) fracture patterns

B. Lateral compression (LC III) fracture patterns

C. Vertical sheer (VS) fracture patterns

D. Combined APC/VS patterns

E. Open pelvic fractures

A

E.

Open pelvic fractures are potentially lethal injuries with a reported mortality rate of 30% to 50%. Open fractures of the pelvis by definition communicate with the rectum, the vagina, or the outside environment by disruption of the skin. They are often associated with disruption of the pelvic floor, leading to loss of tamponade and persistent bleeding.

An additional concern is pelvic infection and sepsis due to the open communication with the rectum, vagina, or skin. Clinical suspicion of an open fracture and any rectal or vaginal bleeding mandates a thorough examination of the pelvis, perineum, rectum/anal complex, and vagina in females.

In addition to visual inspection and digital examination, full evaluation should include rigid proctoscopy or flexible sigmoidoscopy, and speculum examination in females.

Management of major open pelvic fractures includes administration of intravenous antibiotics, washout and debridement of open wounds, and consideration for a possible diverting colostomy to prevent fecal contamination of the open fracture.

155
Q

Identification and management of open pelvic fractures depends on a high index of suspicion by the initial treating provider. Although this patient was on her menses and blood was to be expected on examination, a speculum examination of the vaginal mucosa demonstrates a 2 cm laceration on the left wall of the vagina and a proctoscopic exam showed a rectal laceration. To decrease the risk for post-injury infection and sepsis, which of the following should occur early in the treatment of this injury?

A. Urgent gynecological consultation with primary closure of vaginal laceration

B. Exploratory laparotomy or laparoscopy with repair of rectal laceration and diverting colostomy

C. Exploratory laparotomy with repair of rectal laceration without diversion

D. Urgent urological consultation with repair of bladder and urethral injuries

E. Immediate placement of a suprapubic catheter

A

B.

Early diagnosis of an open pelvic fracture is essential and a thorough examination must be done so that no such injuries are missed. While it is not difficult to diagnose an open fracture when massive wounds of the skin and perineum arc present, a small vaginal or rectal tear that communicates with and contaminates the fracture may be missed unless it is sought specifically.

Rectal injuries must be sought, particularly in patients with a sacral fracture, because the fragments of bone frequently traverse the rectal wall.

Classically, an open pelvic fracture prompts recommendations for colostomy to prevent soft-tissue sepsis in an expanded perineum. It has recently been suggested that fecal diversion in an open pelvic fractures can be applied selectively, according to the actual location, nature, and size of the cutaneous
wound.

For open fractures with an associated rectal injury, immediate diverting colostomy and repair of the rectal laceration (if possible) arc indicated. Anterior wounds of the groin, anterior thigh, iliac crest, or pubis do not require diversion.

156
Q

A 44-year-old female patient presented with a history of profuse bleeding from a scalp wound. She was brought to the emergency room by an attendant. She had direct trauma caused by falling and hitting her head on a large rock while hiking. She denied any loss of consciousness or seizures. The bleeding was controlled by a gauze dressing and compression bandage at the site of the injury. She was vitally stable with a Glasgow Coma Scale (GCS) of 15/15.

On neurological examination, she had no motor deficit except for right sided lower motor neuron (LMN) facial palsy. A large, parietal lacerated scalp wound was visible 3 cm away from the midiinc on the left, measuring about 2 cm in length.

A computerized tomography (CT) of her head was done after she complained of having an abnormal salty taste in her mouth. On leaning forward she had drops of clear fluid coming from her nose.

The head CT is shown in Figure 73-1 (McGraw Hill GS Board Review, p. 292)

  1. Which of the following is an indication for surgical intervention?

A. Depression that docs not exceed the calvarial thickness

B. Associated intracranial hematoma without a mass effect

C. An isolated clean fracture overlying a dural venous sinus in an asymptomatic patient

D. Dural injury with persistent cerebrospinal fluid (CSF) leak

E. Because this is an open skull fracture then operative intervention is mandated.

  1. Regarding the management of this type of fracture, which of the following is true?

A. Early surgical intervention may decrease the risk of traumatic brain injury.

B. Antibiotics are recommended in almost all cases.

C. Even if there is evidence of a wound infection, primary bone repair will have a favorable outcome.

D. Elevation of depressed fracture usually lowers the risk of post-traumatic seizures.

E. Severe cosmetic deformity is an indication for urgent surgery.

A
  1. D.

Image 73-1 is an axial cut of CT head (bone window) showing a left parietal compound depressed bone fracture. This is a case of an open skull fracture or compound depressed skull fracture. It is recommended to manage patients with skull fractures overlying a dural venous sinuses conservatively provided that there is no neurological deficit or CSF leak.

The high probability of vessel wall laceration with subsequent profuse bleeding during surgery is the main cause of perioperative mortality.

Meier and his colleagues reported 100% mortality rate of cases with skull fractures lacerating the posterior third of the superior sagittal sinus and a 50% mortality rate in cases in which the middle third was afflicted. The study showed an overall mortality rate of 41% for cases with major dural sinus injury. In a retrospective review of 27 patients with depressed compound skull fractures overlying but not disrupting a venous sinus, a management recommendation was made that wound washout with antibiotics is an adequate treatment as long as there is no mass effect or contamination deep to bone.

A depressed skull fracture greater than the thickness of the calvarium, an intracranial hematoma with mass effect are considered possible indications of surgery.

  1. B.

There is no evidence that bone elevation following open skull fractures necessarily decreases the incidence of post-traumatic seizures which probably
do occur as a direct result of the initial brain injury.

Early surgery has been recommended to reduce the risk of infection but it does not effect the extent of a traumatic brain injury. Antibiotics are part of the management strategics of compound depressed fractures.

Same session cranioplasty or repair of autologous bone is accepted only if there is no evidence of infection. Marked cosmetic deformity may warrant elective but not urgent surgical skull reconstruction.

157
Q

Four hours later, another patient with the same type of head injury came to the ER but he deteriorated immediately af er admission. His head CT (Figure 73-2) showed the following image:
Figure 73-2 (McGraw Hill GS Board Review)

The main cause oi deterioration of this patient is:

A. Tension pneumocephalus

B. Acute obstructive hydrocephalus

C. Hydrocephalus ex vacuo

D. Subdural hygroma

E. Subdural hematoma

A

A.

Image 73-2 is an axial cut of CT head (soft tissue) shows a case of “Tension Pneumocephalus” with ‘”Mount Fuji sign” due to collection of bifrontal subdural air compressing the brain. This image depicts a case of tension pneumoccphalus.

Pneumocephalus is defined as the presence of air within the intracranial cavity and may include one or more of the following compartments: (a) epidural, (b) subdural. (c) subarachnoid, (d) intraparenchymal. and/or (c) intraventricular.

Tension pncumocephalus is a term used when a considerable amount of air or
gas is detected on a CT scan of the head with rapid clinical deterioration. This scenario usually warrants immediate surgical intervention in order to avoid fatal brain herniation.

The CT here shows accumulation ot trapped air in subdural and interhemispheric space with some resulting compressing of the brain. This radiological linding is called “Mount Fuji sign”. The air usually appears black on
CT scans with a density measuring - 1000 Hounsfield units. No hydrocephalus is seen as there is no ventricular enlargement.

The latter usually occurs due to obstruction of the CSF-flow resulting in CSF
accumulation and subsequent ventriculomegaly from obstructive hydrocephalus.

Hydrocephalus ex vacuo is a term that falsely describes relative ventricular enlargement that may occur in elderly patients secondary to brain parenchymal atrophy from the natural aging process.

The presence of a subdural hygroma is a post-traumatic condition that develops due to a separation of the dura-arachnoid interface followed by a passive fluid accumulation secondary to decreased ICP or from excessive dehydration. Such a fluid collection overlying the brain convexity rarely shows any mass effect which makes surgery usually unnecessary.

The dynamics of alternating absorption and expansion is thought to be responsible for the occasional transformation of some SDHG cases into chronic subdural hematomas.

158
Q
  1. Regarding the CSF rhinorrhea/otorrhea, which of the following is true?

A. CSF has a lower concentration of glucose than does mucus.

B. Magnetic resonance imaging (MR1) is the most sensitive procedure to localize the site of CSF fistula.

C. Meningococci are the most common pathogen that may cause meningitis in this condition.

D. Post-traumatic CSF rhinorrhea is more likely to persist compared to spontaneous CSF rhinorrhea.

E. Post-traumatic CSF otorrhea is usually self-limited.

A

E.

Most post-traumatic CSF leaks subside within 72 hours to 1 week without any need tor surgical intervention.

Daele and colleagues recommended surgery tor spontaneous CSF fistula cases since they are more likely to persist. Post-traumatic CSF otorrhea on the other hand usually stops within 5 to 10 days.

Detection of ßj-transferrin in rhinorrhea-fluid corroborates the presence of a CSF fistula. Tahir and colleagues reported that CT-cisternography has the highest sensitivity and specificity when compared to CT- and MR-scanning to detect the site oi a CSF leak. Pneumococci arc the most common causative organisms in eases of meningitis following a CSF fistula.

Glucose urine strips could help in identifying CSR due to its high glucose concentration (> 40 mg/dl) when compared to low glucose levels (< 5 mg/dl) found in mucus secretions.

159
Q

A 30-year-old male African patient presents to the emergency department with altered level of consciousness following direct trauma to the head by a heavy object about 3 hours prior to arrival He was brought in by his friend, who said that he had transient loss of consciousness for about one minute after which he woke up complaining of headache and blurry vision. He vomited once before becoming somnolent and again about 30 minutes later.

On general examination, his vital signs were as follows: BP = 140/85 mm Hg. RR = 18 cycle/min. Pulse = 68 beat/min, and GCS = 10/15. Bruises were seen
over the right temple. On neurological examination, the patient had a left hemiparesis with round, equal, and reactive pupils. The patient was mildly agitated with good localization of painful stimuli. Deep tendon reflexes (DTRs) were mildly exaggerated with a positive Babinskis sign on the left side.

A thorough examination was done to exclude associated injuries of the spine, chest, and abdomen. A rigid cervical collar had been placed in the field by EMS. Brain imaging via CT scan revealed a large, right parietotemporal epidural hematoma. Cervical spine images were unremarkable. Emergent surgical intervention was performed with postoperative ICU admission for strict
monitoring of vital signs and neurological status.

Regarding the etiology of cranial epidural hematoma (EDH), which of the following is correct?

A. The most common source of bleeding in this case of EDH is an injured anterior division of middle meningeal artery running beneath the pterion.

B. EDH cannot be caused by injury of dural venous sinuses.

C. An underlying fracture is not commonly found in cases of post-traumatic EDH.

D. The bridging cortical veins are usually torn in cases of EDH with associated extensive brain lacerations.

E. The source of bleeding in traumatic EDH and acute subdural hematoma (aSDH) is usually arterial.

A

A.

Tearing or avulsion of the bridging cortical veins is one of the causes of aSDH. Hemorrhagic brain contusions or lacerations might be another source of aSDH.

EDH is commonly caused by an arterial injury of the middle meningeal artery which may progressively strip the dura from the inner table of the skull. EDH is sometimes caused by bleeding from dural veins, venous sinus or diploic channel injury following skull fractures.

In a case series of 40 infants with EDH, Leggate and colleagues could identify the bleeding source in 31 cases. In 42.5% of those cases it was from the middle meningeal artery. Skull fractures were seen in 45% of cases in another series of 210 traumatic EDH cases.

160
Q

Regarding the clinical presentation of patients with cranial EDH, which of the following is correct?

A. EDH is more common in older patients with a 1:3 male to female ratio.

B. A lucid interval is reported in more than 80% of traumatic EDH.

C. All cases of EDH present with altered level of consciousness.

D. Anisocoria occurs in cases of rapidly expanding temporal EDH secondary to central transtentorial herniation.

E. Motor deficit (eg., hemiparesis) may occur ipsilateral to the side of hematoma due to compression against Kernohan’s notch.

F. Symptoms of increased intracranial pressure (ICP; e.g., vomiting) are often more profound in supratentorial EDH compared to the posterior fossa EDH.

A

E.

The false localizing sign, or Kernohan’s notch phenomenon, is a compression of the contralateral cerebral peduncle against the edge of tentorial incisura which can lead paradoxically to an ipsilateral hemiparesis.

It is an important motor localizing sign that appears raise in topography, but may occur with any supratentorial mass during uncal herniation. Yoo and colleagues explored this state of the corticospinal tract in a patient with chronic SDH and ipsilateral hemiparesis by using diffusion tensor imaging (DTl) and transcranial magnetic stimulation (TMS).

EDH occurs less frequently in children and elderly patients likely due to adherence of the dura to the inner table of the skull, thus sealing firmly the
epidural space.

In an epidemiological analysis of 210 cases of traumatic EDH 59.2% of cases were in males and the observed presenting GCS was between 13 and 15. Therefore, it is not uncommon for patients with EDH to be fully conscious at the time of clinical assessment.

A classic lucid interval is seen in only 21% of traumatic EDH cases. Ipsilateral pupillary dilation mostly occurs following oculomotor nerve compression by the medially displaced uncus and hippocampal gyrus (called: uncal herniation).

In 27 posterior fossa EDU cases reported in a study by Su and colleagues, headache and vomiting the most frequently seen symptoms.

Rapid brainstem compression with respiratory depression and death may occur in posterior fossa EDU eases if not treated promptly.

161
Q

Regarding the diagnosis of cranial EDH. which of the following is correct?

A. CT scan of the head is the standard imaging modality that reveals cranial EDH without the need for IV contrast.

B. EDH is usually crescentic in shape on CT imaging, while aSDH is more lenticular with biconvex borders.

C. EDH is usually diffuse beneath the cranial boneand appears limited by the falx.

D. The fresh, undiluted blood of an EDH appears as a hypodensc mass on head CT scan.

E. A positive swirl sign on a CT scan of the head indicates a long-standing EDH, which can be treated conservatively.

A

A.

CT scanning of the head usually shows a classic appearance of EDH which is a homogeneous hyperdense extra-axial lenticular mass with convex borders.

IV post-contrast CT is very rarely needed in cases of isodense EDH. EDH is usually localized to a limited part ot the calvarium since it is confined by
the skull sutures (not the falx).

On the other hand, a SDH is characterized on head CT as a crescentic mass with biconcave shape, diffusely spann mg a large area of the brain convexity and is not limited by the sutures. This is illustrated in Figure 73-6.

A “Swirl sign” is a distinct radiographic sign of active bleeding which may occur in different types of intracranial hemorrhage. It presents as an area of
irregular density in the hyperdense clot and is typically of low attenuation (radiolucency). Its occurrence is associated with worse clinical outcomes and
bigger sizes of intracerebral hematomas as reported by Sclariu et al., as it represents an area of active ongoing bleeding.

162
Q

Regarding the management ot cranial EDH. which of the following is correct?

A. Lumbar puncture is recommended to alleviate elevated ICP symptoms in eases of temporal EDH.

B. In general, an epidural hematoma exceeding 30 cm should be surgically evacuated regardless of the conscious level of the patient.

C. EDH maximal thickness > 3 mm on axial CT slices of the head is an absolute indication for surgical intervention.

D. An acute rapidly expanding EDH is usually evacuated in the operating room by two widely spaced bur holes rather than a craniotomy.

E. Dural tenting/tack-up/hitching stitches (that holds the dura to bone) should be avoided because it may increase the risk ol dural stripping and post-operative EDH re-accumulation.

A

B.

An EDH measuring more than 30 cm in volume must be surgically evacuated regardless of the GCS.

A midline shift of more than 5 mm is also a critical radiological sign of brain compression which usually indicates the need tor surgical intervention. A
patient with a GCS of more than 8 and a hematoma less than 30 cm, less than 15 mm in thickness, with less than 0.5 cm midiinc shift, and without local deficit can be managed by close observation with serial scans and placement in a monitored neurosurgical unit.

Acute intracranial hematomas are evacuated by cranitomics or craniectomies as the fresh blood is almost clotted. On the other hand, chronic SDH are usually liquified and can be surgically drained by one or two burr holes.

Dural tenting/tack-up/hitching stitches (that holds the dura to bone) are advocated by many surgeons at the end of the surgical procedure as a routine to close the epidural space in order to prevent EDH reaccumulation.

However, hemostasis is still the most relevant aspect to prevent rebleeding and there is no compelling evidence to support the role of dural tenting for all intracranial operations.

Lumbar puncture may lead to life threatening central or tonsillar herniation in eases in which there is high ICP and a pressure gradient between the intracranial and intraspinal compartments.

163
Q

Regarding the complications and outcome of cranial EDH. which of the following is correct?

A. Patients with aSDH have a better prognosis and outcome compared to those with EDH.

B. Rapidly expanding temporal EDH may cause uncal herniation where the uncus compresses the medulla oblongata medially.

C. Bradycardia and hypotension are both part of Cushings triad in markedly elevated ICP.

D. Preoperative decerebration does not predict morbidity in closed head injuries.

E. Presence of a skull fracture is a risk factor for the development oi a delayed EDH.

A

E.

Uncal herniation usually occurs with middle cranial fossa masses where the uncus herniates medially compressing the occulomotor nerve, cerebral peduncles of mid brain, and posterior cerebral arteries which may cause unilateral ptosis with mydriasis, hemiparesis, and hemianopia, respectively.

In a case series of 161 consecutive patients operated on for post-traumatic EDH, the overall mortality was 12%, which could be further lowered by early
diagnosis and proactive management. The overall mortality in another study of aSDH was 66%. The Cushing triad is a reflex triggered by markedly elevated ICP causing hypertension, bradycardia, and respiratory irregularities.

Delayed EDH is defined as a radiologically evolving EDH during sequential neuroradiologic examination and which was not present initially. Detection of a skull fracture is an indication for hospital admission under neurologic observation for early detection ol possible delayed EDH.

164
Q

A 21-year-old male Caucasian patient was brought into the emergency room by his girlfriend. She described an episode of a left facio-brachial seizure about 1 hour after he had a motorbike accident. The patient had shown a transient loss of consciousness at the scene. Upon regaining consciousness he complained only of a headache before he developed the seizure. On generalexamination, vital signs were found to be within normal limits. On neurological examination, he was found
to still be in post-ictal status but without any motor deficit. Intravenous Fosphenytoin was administered and a non-contrast head CT was requested to exclude intracranial bleeding. The axial head CT can be seen in Figure 73-3 (McGraw Hill, p 294).

What type of intracranial hemorrhage did he have?

A. Epidural hemorrhage

B. Subdural hemorrhage

C. Subarachnoid hemorrhage

D. Hemorrhagic brain contusions

E. Intraventricular hemorrhage

A

D. Image 73-3 is an axial cut of CT head shows multiple hemorrhagic brain contusions (left frontal and bitemporal). This is a case showing a right temporal hemorrhagic brain contusion.

It is also called diffuse post-traumatic intracerebral hemorrhage and usually appear on CT during the post-traumatic phase as an intra-axial hyperdensity indicating Iresh blood within the brain parenchyma.

165
Q

Regarding this type of intracranial hemorrhage, which of the following is correct?

A. It occurs most commonly in the occipital and parietal poles of the brain.

B. It occurs from an impact ol the brain on bony prominences.

C. Surgical evacuation is only indicated in case of progressive neurological deterioration.

D. Temporal involvement has a lower threshold for surgery than the other sites.

E. IV Dexamethasone is part of the medical management for this type of head injury.

A

D.

The degree of temporal involvement often plays a critical role due to its proximity to the brain stem, whereas mass effect is better tolerated in affected supratentorial brain areas.

Therefore, the development of temporal hematomas carries a lower threshold for surgical intervention.

The indications for surgery in cases of traumatic brain contusions include hematoma causing progressive neurological deterioration; signs of significant mass effect on CT; medically refractory intracranial hypertension; patients with GCS scores of 6 to 8 with a frontal or temporal contusion greater than 20 cm in volume and with a midline shift of at least 5 mm and/or cisternal compression on CT scan, and patients with any lesion greater than 50cm in volume.

Patients with such features should be treated operatively with decompression and possible resection of the affected area.

Cerebral contusions most frequently develop after
a mechanical impact of distinct skull areas onto the
rather soft brain substance in moments in which
the brain bounces. Frontal and temporal lobe tips
arc most commonly aficctcd by such local hemor¬
rhagic contusions following translational accel¬
eration. Glucocorticoids are not recommended to improve the outcome or lower the ICP in patients
with severe traumatic brain injury.

166
Q

Regarding elevated intracranial pressure (= ICP) and cerebral hemodynamics, which of the following is true?

A. Normal ICP of adults should always be less than 10 mm Hg

B. Treatment of high ICT should be initiated for patients with sustained ICP ≥15 mm Hg.

C. The goal of ICP management is to keep mean ICP < 20 mm Hg and cerebral perfusion pressure (CPP) > 70 mm Hg.

D. The cerebral blood flow (CBF) to white matter is higher than that of grey matter.

A

C

ICP is one of the most powerful predictors of neurological outcome in patients with traumatic brain injury. Normal values are variable among different age groups and are shown in the following table (p.273):

Age group & Normal Range (mmHg):
Adults <10-15
Children 3-7
Term infants 1.5-6

Variable thresholds have been suggested for the initiation of treatment for raised ICP in patients with head injury. Most neurosurgical centers use a
sustained ICP >20-25 mm Hg. An intraventricular catheter is the most accurate way of monitoring.

The goal of ICP management is to keep it below 20 mm Hg provided that the CPP is not lowered below 50 mm Hg (not 70 mm Hg) to guard against
cerebral ischemia.

CBF = CPP/CVR- (MAP- ICP)/CVR
CBF is the cerebral Blood Flow
CVR is the Cerebrovascular Resistance
MAP is the Mean Arterial Pressure
CPP is the Cerebral Perfusion Pressure
ICP is the Intracranial Pressure

The average cerebral blood flow usually ranges from 50 to 60 mLmin/100 g of brain tissue during resting state.

Normal white matter receives 18-25 mL/min/100 g, while normal gray matter receives 67-80mL/min/100g.

167
Q

Which of the following will increase ICP?

A. Mannitol 20%

B. Saline 3%

C. Saline 23.4%

D. Dextrose 10%

E. Furosemide

A

D.

Hyperglycemia aggravates cerebral edema. Brain cell swelling is increased by hyperglycemia and ketosis especially during hypocapnia.

Therefore, Dextrose 10% is not considered appropriate in cases of head
trauma as it will further increase the ICP impeding both the CPP and CBF.

Osmotic medical therapy includes mannitol and/or hypertonic saline (used
to control the ICP in traumatic brain injury). Furosemide is a loop diuretic that can result in greater reduction of brain water content if administered in
combination with mannitol as it enhances its effect on plasma osmolality.

168
Q

Which of the following is considered to be routine
measures to control ICP?

A. Elevation ot’thc head to 75°

B. Avoidance of hypotension, hypertension, or hypoxia

C. Hyperventilation to keep PCO2 < 25 mm Hg

D. Heavy sedation and paralysis

E. Endotracheal intubation for patients with GCS<12

A

B.

It is not recommended to use hyperventilation vigorously to lower PaCO2 to less than 25 mm Hg as a PaCO; of less than 30 mm Hg may lower the CBF or distort cerebral autoregulation without consistently lowering ICP.

The goal is to keep PaCO2 at the lower end of eucapnia (35 mmHg). The other options that arc considered routine measures to control ICP are head of bed elevation to 30°, light sedation, intubation for a GCS< 8.

169
Q

A 26-year-old Hispanic male was brought to the emergency room after being involved in a motor vehicle accident 2 hours prior. He complained of an intense headache, nausea, and was found to have memory troubles. He was a known cocaine addict. He was brought in by his roommate whom he could not identify during examination.

The patient had unremarkable vital signs and a GCS of 14-15. On neurological examination, he was agitated and confused with post-traumatic amnesia. He had no apparent motor or sensory deficit except tor right sided pupillary dilation and ptosis since the accident.

A thorough examination was done to exclude associated injuries and a non-contrast CT study of the head was done, which revealed the image seen (Figure 73-4).

What is the most common cause of this type of IC hemorrhage (hyperdense blood film in the subarachnoid cisterns)?

A. Trauma

B. Aneurysmal rupture

C. Ruptured arteriovenous malformation (AVM)

D. Coagulopathy

E. Pituitary apoplexy

F. Cocaine abuse

A

A. Image 73-4 is an axial cut of CT head shows diffuse post-traumatic subarachnoid hemorrhage.

This is a case of subarachnoid hemorrhage (SAH) shown in the CT in the form of a hyperdense blood film in the subarachnoid cisterns. Trauma is still the most common cause of SAH.

Aneurysmal rupture is the most common cause of spontaneous SAH.

Ruptured AVM usually presents in the form of intracerebral hemorrhage or intraventricular hemorrhage.

SAH occurs in about 5% of AVM cases. Other causes may include pituitary apoplexy, coagulopathy, and cocaine abuse.

170
Q

Regarding the clinical presentation of this type of intracranial hemorrhage, which of the following is correct?

A. The patient usually presents with high ICP symptoms secondary to hemorrhagic mass effect and not cytotoxic diffuse brain edema.

B. Seizures are not a part of the course of the disease.

C. Nuchal rigidity is mandatory to confirm the diagnosis.

D. Patients do not present with focal neurological deficits.

E. Sudden onset unilateral ptosis with pupillary dilation may raise the suspicion of cerebral aneurysm.

A

E. Nuchal rigidity may occur in some eases of SAH due to meningeal irritation but it is not a must for diagnosis— Patients with SAH may present with increased ICP symptoms, seizures and/or neurological deficit.

Unilateral ptosis with pupillary dilation may indicate oceulomotor nerve compression secondary to a posterior communicating artery (P-comm) aneurysm.

Nine percent of P-comm aneurysms present with unilateral occulomotor nerve palsy, which may indicate impending rupture and prompt intervention by a specialized neurovascular team. Cytotoxic brain edema is usually seen in head trauma patients (vs. vasogenic brain edema in brain tumor cases).

171
Q

Regarding the diagnosis of this type of hemorrhage, which of the following is correct?

A. Lumbar puncture is a safe procedure to confirm the diagnosis by CSF analysis.

B. CT Head requires IV contrast to increase the sensitivity to 95% of eases within the first 24 hours.

C. Contrast enhanced head CT docs not mimic the radiological image of this type of intracranial bleeding.

D. Fluid-attenuated inversion recovery (FLAIR) MRI is the most sensitive MRI sequence to detect bleeding.

E. MRI is a sensitive radiological study to detect the acute stage (within the first 24 hours) of this type of bleed.

A

D. Lumbar puncture always carries the risk of rebleeding in ease of aneurysmal SAH. Therefore, LP is only needed to confirm the diagnosis of clinically suspicious cases of SAH with negative or equivocal CT scans. Sidman et al. reported that CT scans of the
head had 100% sensitivity in detecting SAH within the first 12 hours.

Morgenstern and his colleagues stated that 97.5% of patients with SAH presenting to the emergency room with “worst headache” symptoms were adequately diagnosed by CT imaging alone. MR1 is valuable in demonstrating subacute and chronic hemorrhages due to Methemoglobin formation which may determine the source of bleeding in patients with multiple cerebral aneurysms.

The FLAIR sequence has the highest sensitivity in detecting the cases with low-grade SAH. T2/ Gradient echo sequence or Susceptibility Weighted Image (SWI) may be done as a complementary sequence. Post-contrast CT of the head may be misdiagnosed as SAH especially if the contrast is injected intrathecal.

172
Q

If this patient had no obvious history of head trauma, what would be the gold standard study to identify the underlying cause of hemorrhage?

A. Magnetic resonance angiography (MRA)

B. Magnetic resonance venography (MRV)

C. Catheter cerebral angiography

D. MRI diffusion study (DWI)

E. CT perfusion study

F. Transcranial doppler study

A

C.

The four-vessel cerebral angiogram or catheter cerebral angiography is the gold standard study for the evaluation of cerebral aneurysms.

CT angiography (CTA) with 3 -dimensional reconstruction imaging may be used instead of catheter angiography in some centers as the sole imaging diagnostic study before major surgical intervention.

CTA is a non-invasive tool that may provide prompt, accurate, and versatile diagnostic and anatomical information on which clipping can be based.

However, unnecessary craniotomies based exclusively on CTA findings were also reported.

MRA can also detect cerebral aneurysms, but it has lower sensitivity and specificity compared to the catheter cerebral angiogram especially for aneurysms less than 3 mm diameter.

MRA is also useful in screening high-risk patients.

MRI Diffusion (DW1) is the best imaging study for detection of acute brain
ischemia within the first hours especially when it involves the brain stem or cerebellum. 

CT perfusion identifies the area surrounding the infarction called potentially salvageable penumbra that allows for better outcomes it interventional treatment modalities are available.

Transcranial doppler (TCD) is a non-invasive semi-quantitative technique of ultrasound derived blood velocity measurements within major cerebral vessels (eg., MCA) through thin regions of skull bones providing a window.

TCD is usually used in SAH to detect cerebral vasospasm early prior to the clinical delayed ischemic neurological deficit.

Theoretically, the higher the velocity of blood flow, the narrower the lumen of the cerebral blood vessel.

173
Q

If the patients conscious level deteriorates, what could be the underlying cause of this deterioration?

A. Hydrocephalus

B. Cerebral vasospasm

C. Hyponatremia

D. Rebleeding

E. Seizures

F. Any of the above

A

F. SAH has numerous possible complications that may cause neurological deterioration.

It is believed that proteinaceous blood products may occlude the
arachnoid granulations causing secondary communicating hydrocephalus in 20% of SAH survivors.

Angiographic cerebral vasospasm is a contributing factor of delayed neurological deficit in patients with SAH.

Broderick and his colleagues reported on the causes of morbidity and mortality following aneurysmal SAH.

In their study, re-bleeding was the most important preventable cause of death.

It is also well-known that seizures may occur early following SAH mostly due to the presence of blood in the cisterns which may irritate the cerebral cortex, but the explanation of late onset seizures remains unclear.

The prevalence rate of hyponatremia following SAH is about 30% to 55%.

Hyponatremia may lead to seizures, vasospasm, altered mental status, or death.

It may occur due to syndrome of inappropriate ADH (SIADH) secretion or glucocorticoid deficiency.

However, correction of hyponatremia should always be gradual and monitored, as rapid corrections may cause central pontine myelinolysis (osmotic demyelinating syndrome).

174
Q

A 75-year-old female was brought to the emergency room by her daughter. The patient has been complaining of confusion, forgetfulness, and headaches over the last month and her symptoms got worse over the last week. Her daughter, who lives with her, said that she has not been herself for about a month. She denies vomiting, nausea, speech difficulties or visual changes. The patient reports several episodes of transient orthostatic light-headedness and has had several falls; she fell last about 2 months prior to admission and had a minor head trauma which was cared for without medical attention; the patient denies loss of consciousness before or after the falls. During history taking, the patient reported that she had a stroke 2 years ago which left her with slight weakness in the left upper and lower extremities.

On physical examination her vital signs show BP 135/90 mm Hg; temperature of 98°F (36.7C); RR of 16/minute; HR is 76/minute and regular. Head, Eye, Ear, Nose, and Throat Exam (HEENT) is normocephalic but showing a large bruise on her left temple. PERRLA shows no fundoscopic abnormalities. Her neck is supple, with no carotid bruits and her heart rate and rhythm is normal S1/S2, with no murmurs, rubs, or gallops.

The neurological examination shows mental status is alert and oriented. The patient could spell words backwards, but recalled only 2/3 objects. Her GCS is 11/15 and the cranial nerves are grossly intact. Her motor strength is 5/5 in all muscle groups except 3/5 in the left arm throughout. Her DTRs are asymmetric 3+ in left upper and lower extremities and 1+ on the right. There was no Babinski sign bilaterally. The cerebellar exam was negative, as was Rombergs test. Her gait is normal and her sensation is intact to pinprick and light touch.

A thorough physical examination was done to exclude skull bone fractures and spine injuries. Head CT-scan imaging showed the images provided below, revealing a crescent shaped, hypodense extra-axial collection as seen in Figure 73-5 (p. 296 McGraw Hill).

The patient was brought to the operating room for evacuation of a hematoma. She did well post-operatively and was discharged to rehabilitation at day-2 after surgery.

Which of the following is considered to be the highest risk factor for chronic subdural hematoma after head trauma?

A. Seizure history

B. Anticoagulation/antiplatelet therapy

C. Gender

D. Alcohol abuse

E. Age >65

A

B. Image 73-5 is an Axial cut of CT head shows right frontoparietal chronic subdural hematoma.

Patients with chronic anticoagulation/ antiplatelet therapies are considered to be at high risk for chronic subdural hematoma from trivial traumas.

Those patients are prone to develop subdural bleeding within days or weeks after small head trauma. The mechanism is not yet very well understood, although alteration in the blood hematoma components was found in most anticoagulation treated patients.

For answer A, although patients with uncontrolled seizure episodes are more prone to frequent head traumas from potential recurrent seizure episodes, those patients don’t have higher incidence of subdural hematomas after trivial trauma compared to patients with chronic anticoagulation therapy.

For answer C, there is higher incidence of chronic subdural hematoma in male compared to female, but gender does not seem to influence the development of CSDH after slight head trauma.

In answer D, history of alcohol abuse associated with coordination and motor problems, consequently more falls, that put the patient at risk for recurrent head trauma.

But history of alcohol abuse does not show a strict correlation with CSDH after trauma as coagulation therapies do.

For answer E, although old people (e.g. age >65 years) have a higher incidence of CSDH compared to those with age <65, age is not considered a risk factor for the development CSDH after head trauma.

175
Q

Which of the following statements is correct with regard the options for management of chronic subdural hematoma?

A. Conservative treatment (e.g., corticosteroids and bed rest) is suitable in lesions with less than 1 cm thickness even with the presence of focal neurologic signs.

B. Burrhole drainage with the placement of subdural drain is not as effective as a craniotomy and should be avoided.

C. Craniotomy is a suitable surgical intervention for loculated lesions.

D. Percutaneous twist-drill bedside drainage is not indicated because of the risk of infection.

E. Operating room twist drill craniostomy requires intubation or heavy sedation.

A

C.

Conservative treatment including bed-rest and corticosteroids have been shown to be very efficient in the treatment of patients with hematoma less than 1 cm without focal neurologic deficit or mental status changes.

Corticosteroids have been posited as both an anti-inflammatory and anti-angiogenic agents.

Answer B, Burrhole drainage with the placement of subdural drain, was shown to decrease significantly the recurrence rate of CSDH after treatment. .

Answer C, Craniotomy with excision of subdural membranes, is indicated when the hematoma persistently recurs despite the treatment with twist-drill craniostomy or burrhole drainage. It has been shown to be safe and valid technique.

Answer D. Percutaneous twist-drill bedside drainage, is the least invasive technique over the above mentioned techniques. It requires a 0.5 cm incision in the scalp and has been shown to be a safe technique.

Operating room twist drill craniostomy, answer E, is a valid option for the management of CSDH, although, it does not offer any advantage over the percutaneous twist-drill drainage in terms of recurrence. However, given that CSDH is more frequent among the elderly, the treatments diverge regarding the perioperative risks related to general anesthesia; burrholes are frequently performed under general sedation, whereas a local anesthetic is usually sufficient with percutaneous drainage, so patients’ comorbidities have to be taken into consideration.

176
Q

Which of the following correlates with the outcome
of chronic subdural hematoma?

A. Hematoma density on CT

B. Traumatic etiology

C. The male sex

D. Hematoma location

E. Age

A

A.

Brain atrophy, hydrocephalus, midline shift, and hematoma density on CT (c.g.. high, iso, or low) all influence the outcome of CSDHs.

Traumatic causes, sex, hematoma location and age have never been shown to correlate with CSDHs outcome.

177
Q

Which of the following risk factors is associated with higher recurrence rate of chronic subdural hematoma (CSDH)?

A. Bilateral hematomas with poor re-expansion rate after surgery

B. Etiology

C. Interval from trauma to first operation

D. CT-characteristics (density)

E. Age

A

A.

The presence of bilateral effusion and poor re-expansion is associated with high rate of recurrence.

For answers B, C, D. E, etiology (e.g., alcoholism, head injury, neurosurgery, anticoagulant, coagulopathy, subarachnoid cyst), interval from trauma to first operation, CT-findings (e.g,. high, iso, or low density), and age are not associated with higher recurrence of rate of CSDH after surgery.

178
Q

A 38-year-old male presents to the trauma bay with a Glasgow coma scale (GCS) of 15 following a motor vehicle crash. The patient undergoes a computed tomography (CT) scan of the head that shows a small traumatic subarachnoid hemorrhage over the right frontal lobe and tiny contusions in the right frontal and left occipital lobes. Overnight the patient becomes slightly more agitated but his GCS remains stable at 15. A CT scan the morning after admission shows no change in his intracranial injuries. Additionally, his chemistry panel the following morning shows a sodium level of 130 mEq/L. His admission sodium was noted to be 139 mEq/L

  1. After determining that the effective serum osmolality was within the normal range and urine osmolality was greater than 100 mOsm/kg, the next step in determining the cause of the hyponatremia is:

A. Assess the patient’s volume status.

B. Determine the liver function and cardiac output.

C. Check pituitary function assays for cortisol
derangements.

D. Draw an arterial blood gas to determine the pH of the blood.

E. Determine the urine sodium level.

  1. Your team has determined that the patient is euvolemic and has given him the diagnosis of syndrome of inappropriate antidiuretic hormone secretion (SiADH). His sodium on last draw was 128. On last exam, the patient shows no symptoms of hyponatremia. What treatment would you begin to correct the low serum sodium level?

A. Fluid bolus

B. Hypertonic saline

C. Fluid restriction

D. Vaptan therapy

E. IV steroid dose

A

A.

The diagnosis for the cause of hyponatremia is generally reached in an algorithmic fashion. After a lab measurement shows serum sodium less than 135 mEq/L, you must determine the effective serum osmolality.

This rules out hyperglycemia, hypertriglyceridemia, and hyperproteinemia (so called pseudohyponatremia).

The next step is to evaluate urine osmolality to ensure that the patient is not suffering from water intoxication. After these three steps are complete, the patient’s volumes status must be ascertained. This is extremely important in a neurologic patient as it will help differentiate the most common neurologic causes of hyponatremia: syndrome of inappropriate antidiuretic hormone secretion (SIADH) and cerebral salt wasting (CSW).

C.

As with the diagnosis, the treatment for SIADH also runs as an algorithm. Again, the patient must be confirmed to be euvolemic as the treatment for CSW involves fluid resuscitation and for SIADH often involves fluid restriction.

In the case of a patient with mild hyponatremia (125 135 mEg/L) with no symptoms, treatment should involve fluid restriction.

If there are symptoms, patients should receive saline infusion with 0.9% saline and Lasix.

Finally, in cases with Na levels less than 125 mEq/L and symptoms, patients are treated 3%saline and Lasix.

Newer medications such as the vaptan class, which act as a vasopressin antagonist, can be used in refractory cases.

179
Q

Which of the following statements is true regarding the neurological signs and symptoms associated with severe hyponatremia?

A. Usually not apparent until Na drops below 115mEq/L

B. If the hyponatremia is chronic (>48 hours in duration), the patient will be symptomatic.

C. If symptoms are present, the physician should rapidly correct the sodium to near normal levels at a rate of 57 mEq/L/hr.

D. Symptoms typically include headache and poor
concentration.

E. Hyponatremia can be fatal due to transtentorial
herniation.

A

E

This question relates to the symptoms and general treatment options of hyponatremia. The neurologic symptoms of mild hyponatremia or chronic hyponatremia involve headache, poor concentration, anorexia, and muscle weakness.

The more acute and severe hyponatremia cases can lead to cerebral edema, nausea, seizures, respiratory arrest, coma, and death from transtentorial herniation.

Symptoms generally are easily diagnosed below 125 mEq/L.

Symptoms are more apparent with acute onset (< 48 hours in duration) as there is less time for mechanisms of the brain to compensate. For that reason, chronic hyponatremia (> 48 hours) can be relatively asymptomatic even at low Na levels.

The rate of correction even in symptomatic and acute cases should not exceed 1 to 2 mEq/L/hr and 8-10 mEq/L in 24 hours to avoid central pontine myelinolysis.

180
Q

The patient’s neurological exam has now deteriorated requiring intubation, external ventricular drain placement, and aggressive intracranial pressure (ICP) management. The patient’s urine output has increased to over 300 cc/hr and the serum sodium has increased to over 150 mOsm/hr. What diagnosis would you assign the patient at this time and what is the likely cause?

A. Nephrogenic diabetes insipidus due to medications

B. Neurogenic diabetes insipidus due to undiagnosed skull fracture

C. Neurogenic diabetes insipidus due to herniation

D. Neurogenic diabetes insipidus due to underlying
meningitis

E. Nephrogenic diabetes insipidus due to underlying kidney disease

A

C

Hypernatremia is defined as a serum sodium >150mEq/L. In patients with neurologic injuries, this is commonly due to diabetes insipidus (DI). DI is due to low levels of ADH or renal insensitivity to ADH.

The diagnosis can be reached by high output of inappropriately dilute urine with high serum osmolality and high serum sodium.

The two major etiologies are neurogenic DI and nephrogenic DI.

The primary causes of nephrogenic DI are medications, renal disease, hypokalemia, and hypercalcemia.

Neurogenic DI can be idiopathic, posttraumatic, tumors, infections, vascular, and autoimmune.

In a posttraumatic patient who has worsening mental status, the cause is most likely due to herniation from pituitary stalk injury or hypothalamic injury from impending brain death.

181
Q

Which of the following is part of the best treatment option for this patient?

A. Monitor fluid intake and output hourly and draw the serum sodium every 24 hours.

B. Place on a basal fluid rate with D51/2NS with replacement for urine output above the base rate.

C. Administer DDAVP first and assess impact on urine output.

D. Allow the patient’s thirst mechanism to control the Na level and treat with DDAVP as needed.

A

B. After arriving at the diagnosis of central diabetes insipidus, it is essential to begin treatment of the patient. The treatment of DI is based around antidiuretic hormone (ADH) analogues. These include Pitressin® and the longer acting, more potent form, DDAVP.

In a conscious and ambulatory patient, often their thirst mechanisms will compensate for the hypernatremia.

However, the patient in this scenario is unable to drink or compensate with his thirst mechanism, so D is clearly incorrect. If the patient is not able to drink, they should be monitored for I and O q1 with serum sodium draws every 6 hours. To deal with the fluid loss, the patient should be placed on a basal fluid rate with D51/2 NS with replacement for urine output above the base rate.

If the fluid replacement continues to fall behind and the hypernatremia worsens, Pitressin or DDAVP may be given.

Though all the possible answers are treatments for DI, only the first three could be utilized for this patient.

182
Q

A 22-year-old male who sustained a gunshot wound to the chest arrives in the emergency room at your facility. He is hemodynamically stable and has an entry wound just below his right nipple and probable exit wound just below the tip of the left scapula. Supine chest film shows no retained bullet fragments, contusion of the lung and moderate left apical pneumothorax. A pericardial ultrasound shows no effusion or other abnormality. The initial diagnostic work-up and management of this injury should include:

A. Computerized tomography angiogram (CTA) of the chest after placement of left chest thoracostomy tube.

B. Placement of left thoracostomy tube and observation on the ward.

C. Trans-esophageal echocardiogram, non-contrast CT scan of the chest, and left chest thoracostomy tube.

D. Bronchoscopy, esophagoscopy, esophagography, a catheter angiogram of the thoracic aorta, and left chest thoracostomy tube.

E. Immediate surgical exploration via a median
sternotomy.

A

A.

Although the majority of thoracic gunshot wounds will be localized to one side of the thoracic cavity, it is critical to identify the smaller subgroup of patients with a trans-mediastinal trajectory.

Trans-mediastinal gunshot wounds carry a significantly higher mortality and morbidity, and can present a diagnostic and management dilemma due to the number of critical structures at risk of injury.

Unstable patients require immediate intervention and surgical exploration based on the initial examination and imaging findings.

Stable patients require further radiologic workup, with the goal of excluding injury to the heart, lungs, great vessels, esophagus, and tracheobronchial tree. Historically this mandated an exhaustive series of imaging studies including chest X-ray or CT scan, echocardiogram, esophagoscopy and esophagography, thoracic angiography, and bronchoscopy.

With the improved availability and image quality
obtained with modern CT scanners, this exhaustive workup has largely been replaced by a single high-quality CT scan of the chest with intravenous contrast in most trauma centers. In addition to imaging the critical mediastinal structures of interest, the CT scan can be used to re-create the missile tract and assess the proximity to these structures and to identify any secondary signs of injury (Figure 76-1).

Patients with a negative CT scan can be safely observe, while those with any concern for a potential injury can then undergo additional focused evaluation based on the CT scan results. This patient has a left pneumothorax and thus a left tube thoracostomy should be placed prior to obtaining the CT scan.

Immediate operative exploration is not indicated in the stable patient with no other obvious signs of injury to a mediastinal organ or structure.

183
Q

A 50-year-old male is brought to a local emergency room after a motor vehicle collision. He is complaining of mild mid-sternal chest pain and has some bruising over the sternum. He is a healthy male with no cardiac history. CT scan of the chest, abdomen, and pelvis are unremarkable. Electrocardiogram shows normal sinus rhythm with a heart rate of 65 bpm. What is the most appropriate management of this patient?

A. Admission to the intensive care unit with serial
cardiac panel and repeat chest film in 24 hours.

B. Echocardiogram performed in the emergency
room to evaluate cardiac wall motion.

C. Discharge home with instructions to return to
the hospital if the symptoms worsen.

D. Consultation with cardiologist for suspected
acute coronary syndrome.

E. Repeat CT scan of the chest in 12 hours.

A

C

This patient presents with mild symptoms after motor vehicle collision.

Cardiac contusion should be suspected but unlikely given clinical presentation and normal electrocardiogram.

Serial cardiac panel could yield mildly elevated enzymes but not clinically relevant data.

Echocardiogram would not be indicated either given normal electrocardiogram.

Cardiology consult is not necessary since the patient’s symptoms are not related to myocardial ischemia.

A repeat CT scan of the chest will likely show no changes. This patient can be safely discharged home.

184
Q

A 33-year-old female was admitted after being involved in a motor vehicle collision live days ago. She had a left chest tube placed upon admission for a hemothorax. Initially, 150cc of blood was evacuated but after 24 hours, only additional 100cc came out. A repeat CT scan of the chest now shows a retained hemothorax and there has been minimal drainage from the chest tube for the past 48 hours. The most appropriate management of this patient at this time would be:

A. Video assisted thorascopic surgery (VATS) to evacuate the hemothorax.

B. VATS if hemothorax does not resolve in three more weeks of observation.

C. Instill thrombolytic agent into the chest cavity via the chest tube to break up the clot.

D. Thoracotomy for decortication and evacuation of hemothorax.

E. CT scan guided aspiration of retained hemothorax.

A

A.

This patient has a retained hemothorax.

There is a significant risk for this patient to develop a complicated fluid collection, fibrothorax, or empyema if the hemothorax is not evacuated in a timely manner.

The instillation of thrombolytic agents into the chest cavity or CT scan guided aspiration of hemothorax may not result in complete evacuation of the hematoma since much of the hemotoma is probably solidified clot at this point.

Patients who received early VATS (within 5 days) had lower incidence of developing empyema in several studies.

Thoracotomy would not be warranted at this time since the hemothorax can be evacuated less invasively with VATS.

CT guided drainage is an option but a significant portion of the hematoma has clotted; therefore, this will not likely be an effective therapy.

185
Q

A 21-year-old male sustained multiple injuries after he struck a tree while riding a motorcycle traveling at 60 mph. He was intubated at the scene for
unresponsiveness. He was transported to your medical center and trauma evaluation was significant for a 2 cm right-sided frontal cerebral contusion, fracture of ribs 2 through 6 on the right, right clavicle fracture and moderate right pulmonary contusion. A right-sided chest tube was placed for large pneumothorax. Repeat chest film shows a persistent pneumothorax so a second chest tube was placed. A third set of chest films show that the lung has re-expanded slightly but a moderate apical pneumothorax remains. The two chest tubes have a persistent large air leak present. The next step in this patients management should be:

A. Oxygenate with 100% oxygen for 24 hours to
allow pneumothorax to resolve spontaneously.

B. Repeat chest film in six hours and planned VATS
if pneumothorax persists.

C. Urgent bronchoscopy.

D. Continued observation as long as the patient is
stable and oxygenating well.

E. Thoracotomy for repair of probable large parenchymal lung laceration.

A

C

The patient has sustained significant chest trauma but tracheobronchial injury must be ruled out given persistent pneumothorax.

This is particularly true in the presence of a large and persistent air leak and inability to re-expand the lung despite adequate mechanical ventilation and chest tube drainage.

An urgent bronchoscopy will be needed to evaluate for a tracheobronchial injury that may require intervention or surgical repair.

It would not be wise to perform a blind thoracotomy until bronchoscopy is performed first to identify and localize the injury.

Observation is particularly dangerous in this situation since even a brief period of hypoxia is not tolerated well in a patient with traumatic brain injury.

100% oxygen therapy is usually reserved for non-traumatic pneumothoraces.

186
Q

An 80-year-old male sustained multiple bilateral rib fractures and a mild pulmonary contusion after a motor vehicle collision. He is hemodynamically stable and oxygenating well in the emergency room but complains of chest pain upon deep inspiration. Trauma evaluation did not reveal any other injuries. The appropriate management of this patient should be:

A. Discharge to home on oral pain medications and instructions to return to the hospital if his condition worsens.

B. Admission to medical-surgical ward with supplemental oxygen and orders for repeat chest films in 12 hours.

C. Admission to intensive care unit, pain control with either a thoracic epidural or patient controlled analgesia (PCA) with ketorolac, and incentive spirometry.

D. Admission to intensive care unit and schedule
operative fixation of rib fractures for the following day.

E. Admission to intensive care unit and electively
intubate the patient as he is at significant risk for
respiratory failure.

A

C

The patient is an elderly male who sustained chest
trauma resulting in multiple rib fractures and pulmonary contusion. This patient is at a high risk for pulmonary complications, pneumonia, and death if not managed appropriately and aggressively.

The patient should be admitted to the intensive care unit for close monitoring and pain management. Elderly patients often have limited physiologic reserve and unrecognized decompensation on a medical-surgical ward could have tragic implications.

Pain control with either a thoracic epidural infusion or intravenous/oral narcotics supplemented with a non-steroidal anti-inflammatory agent such as ketorolac should be initiated, and titrated to allow full inspiratory effort without pain.

Intubation should not be done until the patient has failed conservative management (including in selected cases a trial of non-invasive ventilator support).

Operative fixation of rib fractures is usually reserved for patients who are difficult to wean from mechanical ventilation, and would not be indicated this early after the initial injury.

187
Q

A 79-year-old male develops acute respiratory distress syndrome (ARDS) while being treated for pneumonia. He requires increased positive end-expiratory pressure (PEEP) in order to maintain adequate oxygen saturation. Which of the following mechanisms BEST explains this requirement?

a. Ventilation-perfusion (V/Q) mismatch
b. Alveolar flooding and collapse leading to decreased lung compliance
c. Decreased gas exchange due to pulmonary edema
d. Increased number of alveoli which are inflated and engaging in gas exchange

A

b. Alveolar flooding and collapse leading to decreased lung compliance

188
Q

A patient with a recent fall and subsequent subarachnoid hematoma is found to be persistently hyponatremic. He is making large amounts of urine and is profoundly hypovolemic. Which of the following is the BEST treatment option for this patient?

a. fluid restriction
b. fluid restriction with diuretics
c. isotonic saline administration
d. daily sodium replacement

A

d. daily sodium replacement

189
Q

Which of the following choices may be beneficial in a patient diagnosed with acute respiratory distress syndrome (ARDS)?

a. Increase the tidal volumes to 12mL/kg
b. Institute pressure control ventilation
c. Increase the positive end expiratory pressure
d. Place the patient in the prone position

A

c. Increase the positive end expiratory pressure

190
Q

28-year-old female presents to the emergency department after being involved in a motor vehicle accident. Her heart rate is 55 bpm and blood pressure is 70/30 mmHg. She has an area of obvious right upper extremity deformity. FAST is negative. She is complaining of neck pain. On exam, she has midline tenderness of the mid cervical region. What is the MOST appropriate initial treatment for this patient’s hemodynamic instability?

a. Transfusion with packed red blood cells
b. Aggressive resuscitation with crystalloid
c. Exploratory laparotomy
d. Open reduction and internal fixation of the right upper extremity

A

a?

191
Q

A 43 year-old male mauling victim came to the ER confused and tachycardic with systolic pressure of 80mmHg. How much of his blood volume has probably been lost:

a. < 15%
b. 15 - 30%
c. 30 - 40%
d. > 40%

A

c. 30 - 40%

192
Q

A motorcycle accident victim in the ER was noted to have dyspnea. Upon inspection you noted inward movement of the chest wall during inspiration followed by outward with expiration. The patient most probably has?

a. Pulmonary contusion
b. Tension pneumothorax
c. Cardiac tamponade
d. Flail chest

A

d. Flail chest

193
Q

A 25-year-old man, injured in a motor vehicular crash, is admitted to the ER. His pupils react sluggishly and his eyes open to painful stimuli. He does not follow commands, but he does moan periodically. His right arm is deformed and does not respond to painful stimulus; however his left hand reaches toward it. Both legs are stiffly extended. His Glasgow Coma Scale score is

A. Four
B. Six
C. Nine
D. Twelve

A

C. Nine

194
Q

Which one of the following statements concerning massive hemothorax is TRUE?

A. It is usually caused by blunt thoracic trauma.

B. It is commonly confused with a pneumothorax.

C. The diagnosis should be confirmed by upright, plain chest roentgenograms prior to treatment.

D. The initial draining of 1,000mL of blood after chest tube insertion requires immediate thoracotomy.

A

D. The initial draining of 1,000mL of blood after chest tube insertion requires immediate thoracotomy.

195
Q

A 25-year-old man is brought to the hospital after being involved in a motor vehicular crash when his car struck a bridge abutment. He is intoxicated, has a GCS of 13, and complains of abdominal pain. His blood pressure was 80mmHg systolic by palpation on admission to the hospital, but it rapidly increased to 110/70 with the administration of intravenous fluids. His heart rate is now 120 beats per minute. The chest roentgenogram shows loss of the aortic knob, widening of the mediastinum, no rib fractures, and no hemopneumothorax. Contrast CT angiography

A. Is not indicated
B. Should be performed after tube thoracostomy.
C. Is not necessary if non-contrast CT scan of the chest is normal.
D. Should be performed after a FAST scan.

A

D. Should be performed after a FAST scan.

Dx: Aortic rupture/tear
Transient responder
FAST – Pericardium

196
Q

A 22-year-old man is hypotensive and tachycardic after a shotgun wound to the left shoulder. His blood pressure is initially 80/40mmHg. After 2L of crystalloid solution his BP increases to 122/84mmHg. His HR is now 100bpm and his RR is 28breaths/min. His breath sounds are decreased in the left hemithorax, and after initial IV fluid resuscitation, a closed tube thoracostomy is performed for decreased left breath sounds with the return of a small amount of blood and no air leak. After CTT, the most appropriate next step is:

A. Reexamine the chest
B. Perform an aortogram
C. Obtain a CT scan of the chest
D. Obtain arterial blood gas analyses

A

A. Reexamine the chest

?CPS: Blunting - 250-300

197
Q

The first maneuver to improve oxygenation after chest injury is

A. Intubate the patient
B. Assess ABGs
C. Administer supplemental oxygen
D. Ascertain the need for a chest tube.

A

C. Administer supplemental oxygen

198
Q

A 17-year-olf helmeted motorcyclist loses consciousness when he is struck broad side by an automobile at an intersection. He arrives in the emergency department with a BP of 140/92, pulse rate of 88bpm, an RR of 18 breaths/min, and a GCS of 7. Appropriate initial immobilization of this patient should include a semi-rigid cervical collar and

A. A scoop stretcher
B. A long spine board
C. Cervical traction tongs
D. Pneumatic antishock garment.

A

B. A long spine board

199
Q

A 34-year-old man is brought to the hospital after being pinned to the wall of a building by a cement truck. He is in obvious shock, and has deformities and marked swelling of both thighs, although no open wounds are present. His shock

A. Cannot be explained without concomitant pelvic fracture.

B. Signifies a loss of approximately 15% of his blood volume.

C. Is consistent with blood loss from bilateral femoral fractures.

D. Will likely be reverse if appropriate traction splints are applied.

A

C. Is consistent with blood loss from bilateral femoral fractures.

Long bones: 1.5-2L of blood

200
Q

The best guide for adequate fluid resuscitation of burn patients is

A. Adequate urine output

B. Normalization of heart rate

C. A normal central venous pressure

D. Total fluids in 24h = 4x weight (kg) x percent body surface area burned

A

A. Adequate urine output

201
Q

Establishing a diagnosis of shock must include:

A. Confirming hypoxemia

B. Confirming increased vascular resistance

C. Documenting hypotension and low cardiac output

D. Evidence of inadequate perfusion of the body’s organs

A

D. Evidence of inadequate perfusion of the body’s organs

202
Q

A seven-year-old boy is brought to the ER by his parents several minutes after he fell through a window. He is bleeding profusely from a 6cm (2.4inch) wound of his medial right thigh. Immediate management of the wound should consist of:

A. Application of a tourniquet

B. Direct pressure on the wound

C. Apply a hemostat to bleeding vessels

D. Direct pressure on the femoral artery at the groin.

A

B. Direct pressure on the wound

203
Q

For the trauma patient with cerebral edema, hypercarbia should be avoided to prevent

A. Metabolic acidosis

B. Respiratory acidosis

C. Cerebral vasodilatation

D. Neurogenic pulmonary edema

A

C. Cerebral vasodilatation

204
Q

A 23-year-old man sustains four stab wounds to the right upper hemithorax during an altercation and is brought by ambulance to a community hospital. The wound are all above the nipple. He is endotracheally intubated, close tube thoracostomy is performed, and two liters of Ringer’s lactate solution are infused through two large-calibre IVs. His blood pressure now is 60/0, pulse rate is 160bpm, and respiratory rate is 14 breaths/min (ventilated with 100% O2). The most appropriate next step in managing this patient is:

A. Angiography

B. Thoracotomy

C. CT of the chest

D. Application of PASG

A

B. Thoracotomy

205
Q

The cause of upper airway obstruction most likely to be alleviated by a properly placed oral airway is

A. Foreign body

B. Facial fractures

C. Copious, tenacious oral secretions

D. Posterior displacement of tongue

A

D. Posterior displacement of tongue

206
Q

The primary indication for transferring a patient to a higher level trauma center is

A. Multisystem injuries, including severe head injury

B. Unavailability of a surgeon or operating room staff

C. Unavailability of an intensive care unit bed or ICU staff

D. Resource limitations as determined by the referring physician.

A

D. Resource limitations as determined by the referring physician.

207
Q

A patient is brough to the ED 20 minutes after a motor vehicular crash. He is conscious and there is no obvious external trauma. He arrives at the hospital intubated and completely immobilized on a long spine board. His blood pressure is 60/40 and his pulse rate is 70bpm. His skin is warm and he has no rectal tone. Which one of the following statements is TRUE?

A. Vasoactive medications have no role in early management.

B. The hypotension should be managed with volume resuscitation alone.

C. Flexion and extension views of the c-spine should be performed early.

D. Occult abdominal visceral injuries can be excluded as a cause of hypotension.

A

A. Vasoactive medications have no role in early management.

After the airway is secured and ventilation is ade- quate, fluid resuscitation and restoration of intravascular volume often will improve perfusion in neurogenic shock. Most patients with neurogenic shock will respond to restoration of intravascular volume alone, with satisfactory improvement in perfusion and resolution of hypotension.

Administration of vasoconstrictors will improve peripheral vascular tone, decrease vascular capacitance, and increase venous return, but should only be considered once hypovolemia is excluded as the cause of the hypotension and the diagnosis of neurogenic shock is established.

If the patient’s blood pressure has not responded to what is felt to be adequate volume resuscitation, dopamine may be used first. A pure α-agonist, such as phenylephrine, may be used primarily or in patients unresponsive to dopamine.

Specific treatment for the hypotension is often of brief duration, as the need to administer vasoconstrictors typically lasts 24 to 48 hours. On the other hand, life-threatening cardiac dysrhythmias and hypotension may occur up to 14 days after spinal cord injury.

The duration of the need for vasopressor support for neurogenic shock may correlate with the overall prognosis or chances of improvement in neurologic function.

Appropriate rapid restoration of blood pressure and circulatory perfusion may improve perfusion to the spinal cord, prevent progressive spinal cord ischemia, and minimize secondary cord injury.

Restoration of normal blood pressure and adequate tissue perfusion should precede any operative attempts to stabilize the vertebral fracture.

208
Q

An 18-year-old man is brought to the hospital after smashing his motorcycle into a tree. He is conscious and alert, but paralyzed in both arms and legs. His skin is pale and cold. He complains of thirst and difficulty in breathing. His airway is clear. His BP is 60/40 and his pulse rate is 140bpm. Breath sounds are full and equal bilaterally. He should

A. Undergo exploratory celiotomy (laparotomy)

B. Be treated for neurogenic shock

C. Be treated for hypovolemic shock

D. Undergo immediate nastotracheal intubation.

A

C. Be treated for hypovolemic shock

209
Q

A 32-year-old man is trapped from the waist down beneath his overturned car for a period exceeding eight hours. On arrival in the emergency department, both lower extremities are cool, mottled, insensate, and motionless. Despite normal vital signs, pulses cannot be palpated below the femoral vessels and the muscles of the lower extremities are firm and hard. During the initial management of this patient, which of the following is most likely to improve the chances for limb salvage?

A. Applying skeletal traction

B. Administering anticoagulant drugs

C. Administering thrombolytic therapy

D. Performing lower extremity fasciotomies

A

D. Performing lower extremity fasciotomies

210
Q

Which one of the following statements concerning intraosseous infusion in children is TRUE?

A. Only crystalloid solutions may be safely infused through the needle

B. Aspiration of bone marrow confirms the appropriate positioning of the needle

C. Intraosseous infusion may be utilized indefinitely in the management of injured children.

D. Swelling in the soft tissue around the intraosseous site is not a reason to discontinue infusion.

A

B. Aspiration of bone marrow confirms the appropriate positioning of the needle

IO: Indicated for <6 years
- proximal tibia

211
Q

A construction worker falls two stories from a building and sustains bilateral calcaneal fractures. In the emergency department, he is alert, vital signs are normal, and he is complaining of severe pain in both heels and his lower back. Lower extremity pulses are strong and there is no other deformity. The suspected diagnosis is most likely to be confirmed by:

A. Angiography
B. Compartment pressures
C. Retrograde urethrogram
D. Complete spine x-ray series

A

D. Complete spine x-ray series

Dx: Compression fx

212
Q

A four-year-old girl, weighing approximately 20kg (44pounds), is admitted in shock after an automobile crash. How much crystalloid should be given as an initial fluid bolus?

A. 200mL
B. 400mL
C. 440mL
D. 880mL

A

B. 400mL

213
Q

The principle of balanced resuscitation is

A. Permissive hypotension and early plasma infusion

B. Equal amounts of crystalloid and colloids

C. Simultaneous management of breathing and circulation

D. Maintenance of a normal acid base balance

A

A. Permissive hypotension and early plasma infusion

214
Q

In managing the head-injured patient, the most important initial step is to

A. Secure the airway.

B. Immobilize the neck.

C. Support the circulation.

D. Control scalp hemorrhage.

E. Determine the GCS.

A

A. Secure the airway.

215
Q

A previously healthy, 70kg man suffers an estimated acute blood loss of two liters. Which one of the following statements apply to this patient?

A. His pulse pressure will be widened.

B. His urinary output will be at the lower limits of normal.

C. He will have tachycardia, but no change in his systolic blood pressure.

D. His systolic blood pressure will be decreased with a narrowed pulse pressure.

A

D. His systolic blood pressure will be decreased with a narrowed pulse pressure.

216
Q

The physiologic hypervolemia of pregnancy has a clinical significance in the management of the severely injured, gravid woman by

A. Reducing the need for blood transfusion.

B. Increasing the risk of pulmonary edema

C. Complicating the management of closed head injury

D. Increasing the volume of blood loss to produce shock.

A

D. Increasing the volume of blood loss to produce shock.

217
Q

Which one of the following findings should prompt immediate management during the primary survey?

A. Distended abdomen

B. Glasgow Coma Scale Score of 11

C. Pulse rate of 120 beats per minute

D. Respiratory rate of 32 breaths per minute

A

D. Respiratory rate of 32 breaths per minute

218
Q

During the primary and secondary surveys, the injured patient should be completely immobilized until

A. The neurologic examination has been completed

B. The patient is transferred to a definitive care area.

C. The patient is able to indicate that he has no neck pain.

D. A spinal fracture has been excluded clinically or by roentgenograms.

A

D. A spinal fracture has been excluded clinically or by roentgenograms.

219
Q

The most important, immediate step in the management of an open pneumothorax is

A. Endotracheal intubation

B. Operation to close the wound

C. Placing a chest tube through the wound

D. Placement of an occlusive dressing that is open on one side over the wound.

A

D. Placement of an occlusive dressing that is open on one side over the wound.

220
Q

Important screening roentgenograms to obtain in the multiple-system trauma patient during the primary survey are

A. Chest, and abdomen

B. Chest and pelvis

C. Cervical spine and pelvis

D. Skull and chest

A

B. Chest and pelvis

221
Q

A 56-year-old man is thrown violently against the steering wheel of his truck during a motor vehicular crash. On arrival in the emergency department he is diaphoretic and complains of chest pain. His blood pressure is 60/40 and his respiratory rate is 40 breaths/min. Which of the following would best differentiate cardiac tamponade from tension pneumothorax as the cause of his hypotension?

A. Tachycardia

B. Pulse volume

C. Breath sounds

D. Jugular venous pressure

A

C. Breath sounds

222
Q

A 52-year-old woman sustains 50% total body surface area flame burns in an explosion. She has circumferential burns around the chest and both upper arms. Adequate resuscitation is initiated. She is nasotracheally intubated and is being technically ventilated. Her carboxyhemoglobin level is 10%. Her arterial blood gas analysis reveals PaO2 of 40mmHg, PaCO2 of 60mmHg, and pH of 7.25. Appropriate immediate management at this time is to

A. Ensure adequate tissue perfusion

B. Increase the rate of fluid resuscitation

C. Add positive end-expiratory pressure (PEEP)

D. Reassess for the presence of a pneumothorax.

A

C. Add positive end-expiratory pressure (PEEP)

223
Q

Which one of the following statements regarding abdominal trauma in the pregnant patient is TRUE?

A. The fetus is in jeopardy only with major abdominal trauma.

B. Leakage of amniotic fluid is an indication for hospital admission.

C. Indications for peritoneal lavage are different from those in the nonpregnant patient.

D. Penetration of an abdominal hollow viscus is more common in late than in early pregnancy.

A

B. Leakage of amniotic fluid is an indication for hospital admission.

224
Q

A 26-year-old seat-belted driver is brought to the hospital after a car crash. Primary survey reveals no evidence of serious injury except for diffuse mild abdominal tenderness. Bowel sounds are hypoactive and liver dullness is questionable. Abdominal films reveal free air. The patient should

A. Undergo peritoneal lavage

B. Undergo prompt laparotomy

C. Be carefully observed for further evidence of intra-abdominal injury

D. Have a contrast roentgenographic study of the GI tract.

E. Be suspected of having a ruptured diaphragm and accompanying pneumothorax.

A

B. Undergo prompt laparotomy

225
Q

Which of the following injuries is most likely to be missed by FAST or DPL?

A. Mesenteric laceration

B. Splenic capsular laceration

C. Ileal rupture

D. Hepatic fractures

E. Duodenal rupture

A

E. Duodenal rupture

Retroperitoneal (CT scan)

226
Q

A 25-year-old man is brought to a hospital with a general surgeon after being involved in a motor vehicle crash. Computed tomography shows an aortic injury and splenic laceration with free abdominal fluid. His BP falls to 70mmHg after CT. The next step is:

A. Contrast angiography

B. Transfer to a higher level trauma center

C. Exploratory laparotomy

D. Infuse additional crystalloids

A

C. Exploratory laparotomy

227
Q

The physiologic hypervolemia of pregnancy has clinical significance in the management of the severely injured gravid woman by:

A. Reducing the need for blood transfusion

B. Increasing the risk of pulmonary edema

C. Reducing the volume of crystalloid required for resuscitaiton

D. Increasing the volume of blood loss to produce maternal hypotension.

A

D. Increasing the volume of blood loss to produce maternal hypotension.

228
Q

A hemodynamically normal 10-year-old girl is admitted to the Pediatric Intensive Care Unit (PICU) for observation after a Grade III (moderately severe) splenic injury has been confirmed by computed tomography (CT). Which of the following mandates prompt celiotomy (laparotomy)?

A. A serum amylase of 200

B. A leukocyte count of 14,000

C. Extraperitoneal bladder rupture

D. Free intraperitoneal air demonstrated on follow-up CT

A

D. Free intraperitoneal air demonstrated on follow-up CT

229
Q

A healthy young male in a motor vehicle crash is brought to the ER with a BP of 84/60, pulse 123, GCS 10. The patient moans when his pelvis is palpated. After initiating fluid resuscitation, the next step in management is:

A. Placement of a pelvic binder

B. Transfer to a trauma center

C. Pelvic X-ray

D. Repeat examination of pelvis

A

A. Placement of a pelvic binder

230
Q

Which of the following statement typically characterizes the syndrome of overwhelming postsplenectomy sepsis?

A. A syndrome of gram negative bacteremia and septicemia

B. A syndrome caused by impaired host ability to mount an effective humoral response to infection

C. A syndrome of rapidly appearing septic shock unresponsive to antibiotic therapy, with an average mortality of 50%

D. A syndrome that maybe prevented by preserving as little as 15% of splenic mass in adult trauma victims

A

C. A syndrome of rapidly appearing septic shock unresponsive to antibiotic therapy, with an average mortality of 50%

231
Q

Which is incorrect of abdominal trauma?

A. Spleen is most commonly injured viscus in blunt trauma

B. Liver is most commonly injured viscus in penetrating trauma

C. Conservative management of ruptured spleens is more successful in children

D. Delayed splenic rupture is seen in splenic injuries due to liquefaction of haematoma

A

D. Delayed splenic rupture is seen in splenic injuries due to liquefaction of haematoma

232
Q

Which is incorrect of abdominal trauma?

A. Abdominal exam is not included in the primary survey.

B. Presence of free gas on plain radiography indicates need for immediate laparotomy.

C. Warmed ringers or saline of 10ml/kg should be instilled for DPL.

D. Free aspiration of 20mL of blood in adults and 10mL in children is a positive DPL.

A

A. Abdominal exam is not included in the primary survey.

233
Q

Formula to compute GCS if patient is intubated or unable to verbalize?

A

GCS = E + DVS + M

DVS: Derived Verbal Score
= E(0.4) + M (0.5)

234
Q

What is the GCS-P Score?

A

Developed to improve ability to predict patient prognosis.

Pupil Reactivity Score: # of non-reactive pupils

  • both pupils unreactive = 2 (59% mortality)
  • only one pupil unreactive = 1 (38% mortality)
  • both pupils reactive = 0 (16% mortality)

GCS-P score = GCS - P (range = 1 to 15)

235
Q

Canadian Head CT Rules?

A

High-risk factors (for neurological intervention)

  • GCS <15 at 2h post injury
  • Suspected open skull fracture
  • Signs of basal skull fracture
  • > 2 episodes of vomiting
  • Age >65 years old

Medium-risk factors (for brain injury on CT)

  • amnesia before impact >30mins
  • dangerous mechanism of injury
236
Q

Imaging findings that warrant urgent referral to Neurosurgery:

A
Acute hemorrhage
Signs of cerebral edema
Attenuation of gray-white junction
Midline shift
Obliteration of cisterns
Tonsillar hernia roon
Intraventricular hemorrhage with acute hydrocephalus
237
Q

Discuss central cord syndrome.

A

Affected spinal tracts:
Incomplete cervical white matter injury
Bilateral central CST
Lateral STT

Etiology:
Hyperextension injury (eg, car crash) associated with chronic cervical spondylosis
Spinal cord compression

Clinical:
Bilateral paresis
(Upper > lower extremities)

238
Q

Discuss anterior cord syndrome.

A

Affected spinal tracts:
Anterior gray matter
Descending CST and STT with preservation of posterior columns (DCP)

Etiology:
Trauma (eg penetrating injury, burst fracture of vertebra)
Occlusion of anterior spinal artery

Clinical:
Bilateral motor paralysis
Loss of pain and temperature sensation
Autonomic dysfunction below level of lesion

239
Q

Discuss posterior cord syndrome.

A

Affected spinal tracts:
Bilateral posterior columns (DCP)

Etiology:
Trauma (penetrating injury)
Occlusion of posterior spinal artery
Multiple sclerosis

Clinical:
Ipsilateral loss of proprioception, vibration, and touch sensation below the level of the lesion

240
Q

Discuss Brown-Sequard syndrome (hemisection syndrome).

A

Affected spinal tracts:
Hemisection of the cord

Etiology:
Trauma (penetrating injury)
Spinal cord compression

Clinical:
Ipsilateral
Loss of proprioception, vibration, tactile discrimination below level of lesion
Segmental flaccid paresis at the level of the lesion
Spastic paralysis below the level of the lesion
Ipsilateral Babinski sign

Contralateral
Loss of pain and temperature sensation one or two levels below the lesion

241
Q

Discuss Cauda Equina syndrome.

A

Affected spinal tracts:
Results In a LMNL

Etiology:
Injuries below the L1 vertebral level
Usually incomplete, but can be complete

Clinical features:
Flaccidity
Areflexia
Impairment of bowel and bladder function
Regeneration of peripheral nerves is possible
242
Q

Postsplenectomy vaccines?

A

1) Polyvalent pneumococccal
0. 5mL SC deltoid or upper thigh, q6 years

2) Quadravalent meningococcal or diphtheria conjugate
0. 5mL IM upper deltoid q3-5years

3) Quadravalent meningoccocal polysaccharide 0.5mL
SC deltoid or upper thigh
q3-5years

4) Haemophilus B conjugate
0. 5mL IM (single dose)

243
Q

Grading of intraabdominal hypertension?

A

Grade I: 12-5mmHg
Grade II: 16-20mmHg
Grade III: 21-25mHg
Grade IV: 25mmHg onwards

244
Q

Management of Liver injuries in trauma?

A

1) Nonoperative
- Simple bleeding lacerations

2) Direct suture
- Shallow slowly oozing injuries

3) Hepatotomy/Tractotomy
- Tracing and unroofing of the wound and bleeding sites directly ligated

4) Pringle maneuver
- Intermittent clamping of the hepatoduodenal ligament at the epiploic foramen in cycles (up to 10-15 minutes)

5) Selective hepatic artery ligation
- Either left or right hepatic artery may be ligated following a positive Pringle’s maneuver (bleeding stops while the hepatoduodenal ligament is clamped and resumes when released)

6) Resection or debridement
- For extensive injury and devitalized tissues

7) Atriocaval shunt
- For significant hemorrhage likely due to major hepatic venous or retrohepatic vena caval injury (ie, does not stop with Pringle technique)

8) Perihepatic packing
- Performed as damage control (coagulopathic, acidotic and hypothermic)

245
Q

Management of Splenic Injuries in trauma?

A

1) Splenorrhaphy
- For stable patients without associated multiple injuries

2) Splenectomy
- Hemodynamic instability, multiple associated injuries, completely shattered parenchyma

3) Nonoperative
- Patients remaining hemodynamically stable

4) Postsplenectomy measures
- Giving vaccines 2 weeks post-splenectomy

246
Q

Management of Pancreatic injuries in trauma?

A

1) Simple repair and drainage
- Minor pancreatic lacerations or contusions

2) Pyloric exclusion or diverticularization
- Repair of the duodenum, drainage, and closure of the pylorus with diversion of gastric contents through a gastrojejunostomy

3) Pancreaticoduodenal resection
- indicated only for severe injuries to the head of the pancreas and the duodenum (rare occurrence)

4) Distal pancreatectomy
- For distal pancreatic injuries with major ductal involvement

5) Damage control surgery and postoperative ERCP
- For highly unstable patients

247
Q

Management of hollow viscus injury in trauma?

A
  • Involves initial hemostasis, and subsequent repair or resection
  • Assume even number of wounds (each entry wound has an associated exit wound)
  • Mesenteric border of the bowel, the retroperitoneal duodenum and colon, the cardia, posterior wall, and greater and lesser curvature of the stomach are areas that might be missed

Stomach:
1) Primary repair for small perforations

2) Proximal or distal gastrectomy for extensive injuries

Small bowel
1) Primary repair for perforations <50% of bowel circumference

2) Segmental resection and primary anastomosis for perforation >50% of the bowel circumference, multiple injuries in close proximity

248
Q

Management of colon injuries in trauma?

A

1) Primary suture repair without colostomy
- For nondestructive injuries (<50% of bowel wall without devascularization)

2) Segmental resection and primary anastomosis
- For destructive injuries (>50% of bowel wall or with devascularization)

3) Alternatives for destructive injuries:
- Debridement
- Primary repair and exteriorization
- Proximal colostomy with or without repair

4) Classic Criteria for Colostomy (Stone and Fabian, 1979):
- Shock: BP <90/60
- Hemorrhage: intraperitoneal blood loss >1L
- Multiple injuries: >2 other intraabdominal injuries
- Contamination: significant fecal spill
- Time: >8h after injury
- Colon wound: So destructive as to require resection
- Abdominal wall: Major loss of substance or requiring mesh replacement

5) Timing of colostomy closure after trauma
- Within 2 weeks after colostomy creation provided all wounds have completely healed and nutritional state is optimal

249
Q

Management of rectal injuries in trauma?

A

1) Intraperitoneal injuries
- Manage as colonic injuries

2) Extraperitoneal injuries
- 4 D’s:
- - Debridement (of devitalized tissues)
- - Diversion (select cases when satisfactory repair cannot be performed because of anatomic location or extent of injury)
- - Drainage (presacral– not supported by current literature)
- - Distal rectal washout (no evidence for reducing morbidity)

250
Q

Differentiate intraabdominal hypertension and ACS.

A

Intraabdominal pressure (IAP)

  • Steady-state pressure concealed within the abdominal cavity
  • mmHg, measured at end-expiration in supine position after ensuring that abdominal muscle contractions are absent, and with transducer zeroed at the level of the MAL
  • IAP: ~5-7mmHg in critically ill adults
  • Reference standard for intermittent IAP measurements is via the bladder (maximal instillation of 25mL sterile saline)
Intraabdominal hypertension ( IAH)
- Sustained or repeated pathological elevation in IAP greater than or equal to 12mmHg

Abdominal perfusion pressure (APP)

  • Difference between mean arterial pressure (MAP) and intraabdominal pressure (IAP)
  • AFP = MAP - IAP

Abdominal compartment syndrome (ACS)
- Sustained IAP >20mmHg with or without APP <60mmHg that is associated with new organ dysfunction or failure.

251
Q

Risk factors for primary ACS?

A
  • Blunt/penetrating trauma with hemorrhage
  • Abdominopelvic trauma or injury and retroperitoneal hemorrhage
  • Ruptured aortic aneurysm
  • Ascites due to liver cirrhosis
  • Other conditions that require early surgical or angioradiologic intervention (secondary peritonitis)
252
Q

Risk factors for secondary ACS?

A
  • Extensive fluid resuscitation following major trauma
  • Forced abdominal wall closure following peritonitis, ileum or abscess
  • Laparoscopy and pneumoperitoneum
  • Abdominal packing following hemorrhage
  • Other conditions not originating from an abdominal disease (eg sepsis, major burns)
253
Q

Risk factors for tertiary ACS?

A
  • Recurrent or persistent ACS following prophylactic or therapeutic surgical or medical treatment of ACS (eg, persistence after decompressive laparotomy, new ACS after definite abdominal wall closure)
254
Q

Management of intraabdominal hypertension (>12mmHg)?

A

Conservative

  • Paracentesis
  • NGT
  • Gastric prokinetics
  • Rectal enemas and suctioning
  • Colonic pro kinetics (prostigmine)
  • Furosemide either alone or with human albumin 20%
  • Continuous venovenous hemofiltration or ultrafiltration
  • Continuous negative abdominal pressure
  • Sedation and muscle relaxation
  • Upright (sitting) body positioning (pilot seat)
255
Q

Management of abdominal compartment syndrome (>20mmHg)?

A

Decompressive laparotomy

- Perform or revise abdominal decompression with temporary abdominal closure.

256
Q

Signs of urethral disruption after lower abdominal trauma?

A
Inability to void
Blood on urethral meatus
Perineal or scrotal hematoma
High riding or boggy prostate on DRE
Voided specimen with hematuria

*Blind urethral catheterization is contraindicated unless urethral injury is ruled out by a urethrogram.

257
Q

Management of Kidney injuries in trauma?

A

1) Nonoperative
- Bed rest, hemodynamic monitoring, serial hematocrit

2) Embolization
- For persistent bleeding from a segmental renal artery, unstable patients with complex renal injury, pseudo aneurysm or AVM, persistent gross hematuria, rapidly declining hematocrit.

3) Absolute indications for renal exploration for primary repair, or possible total or partial nephrectomies
- Expanding, pulsatile, uncontained retroperitoneal hematomas
- Renal pedicle avulsion
- Persistent, life-threatening hemorrhage or shock
- Ureteropelvic junction disruption

258
Q

Management of ureteral injuries in trauma?

A

1) Proximal and mid injuries
- Ureteroureterostomy (primary anastomotic repair) over a double-J stent

2) Distal injuries
- Ureteroneocystostomy (reimplantation of the ureter to adjacent bladder)
- Psoas hitch
- Boari flap: For bridging wide gaps in ureteral length

259
Q

Management of urinary bladder injuries in trauma?

A

1) Intraperitoneal bladder perforations
- Primary water tight absorbable suture repair in 2-3 layers and suprapubic cystostomy

2) Extraperitoneal bladder perforations
- Transurethral catheter drainage alone for 7-10 days

260
Q

Management of urethral injuries in trauma?

A

1) Retrograde urethrogram
- Done to rule out urinary extravasation

2) Best managed by suprapubic cystotomy and delayed urethral repair

261
Q

Hard signs of vascular injury (need for operative intervention)?

A
Pulsatile bleeding
Expanding hematoma
Palpable thrill
Audible bruit
Acute limb ischemia (pallor, paresthesia, paralysis, pain, pulselessness, poikilothermia)
262
Q

Soft signs of vascular injury (need for further evaluation)?

A

History of moderate hemorrhage
Injury (fracture, dislocation, penetrating wound)
Diminished but palpable pulse (A-A index of <0.9)
Peripheral nerve deficit

  • Diagnostics:
  • Doppler assessment of pulsatile flow
  • Angiography
263
Q

Management of peripheral vascular injuries in trauma?

A
  • Preoperative antibiotics
  • Adequate surgical exposure with initial proximal and distal vascular control
  • Systemic heparin (5,000-10,000 units IV) unless contraindicated
  • Primary repair: if vessels can be repaired without tension or narrowing
  • Vein patch or autogenous interposition graft (rather than prosthetic graft)
  • Use of temporary shunts for prolonged procedures
  • Completion arteriograms to confirm adequacy of the repair and distal flow
264
Q

Vessels for which repair should ALWAYS be attempted?

A
ARTERIES
Aorta
Carotid
Brachiocephalic
Brachial
Superior mesenteric
Proper hepatic
Renal
Iliac
Femoral
Popliteal

VEINS
Superior vena cava
Suprarenal inferior vena cava
Portal vein

265
Q

Indications for ED Thoracotomy?

A

1) Salvageable postinjury cardiac arrest
- Patients sustaining witnessed penetrating trauma with <15mins of prehospital CPR
- Patients sustaining witnessed blunt trauma with <10mins of prehospital CPR
- Patients sustaining witnessed penetrating trauma to the neck or extremities with <5mins of prehospital CPR.

2) Persistent severe post injury hypotension (SBP less than or equal to 60mmHg)
- Cardiac tamponade

  • Hemorrhage (intrathoracic, intraabdominal, extremity, cervical)
  • Air embolism
266
Q

Contraindications for ED thoracotomy?

A

1) Penetrating trauma: CPR> 15mins and no signs of life (pupillary response, respiratory effort, motor activity)
2) Blunt trauma: CPR> 10mins and no signs of life or asystole

267
Q

A 35-year-old man is admitted with systolic
blood pressure (BP) of 60 mm Hg and a heart
rate (HR) of 150 bpm following a gunshot
wound to the liver (Fig. 1–1). What is the effect
on the kidneys?

(A) They tolerate satisfactorily ischemia of
3–4 hours duration.

(B) They undergo further ischemia if
hypothermia is present.

(C) They can become damaged, even though
urine output exceeds 1500 mL/d.

(D) They are affected and cause an
increased creatinine clearance.

(E) They are prevented from further damage
by a vasopressor.

A

(C)

High-output renal failure should be suspected if the BUN continues to rise with urine output >1000–1500 mL/d.

It is associated with mild-to-moderate renal insufficiency; in comparison, severe renal injury results in oliguric renal failure.

The kidneys do not tolerate ischemia for more than 30–90 minutes.

Hypothermia is protective.

There is a decrease in creatinine clearance.

Vasopressors aggravate the deleterious effects of shock.

268
Q

Twenty-four hours after colon resection, urine
output in a 70-year-old man is 10 mL/h. Blood
chemistry analysis reveals sodium, 138 mEq/L;
potassium, 6 mEq/L; chloride, 100 mEq/L; bicarbonate, 14 mEq/L. His metabolic abnormality
is characterized by which of the following?

(A) Abdominal distension

(B) Peaked T waves

(C) Narrow QRS complex

(D) Cardiac arrest in systole

(E) J wave or Osborne wave

A

(B)

Hyperkalemia can manifest by GI or cardiovascular signs. GI symptoms include nausea, vomiting, intestinal colic, and diarrhea.

Abdominal distension as a result of paralytic ileus is due to hypokalemia. An ECG is useful to monitor potassium levels.

Hyperkalemia is characterized by peaked T waves. ECG changes also include ST-segment depression, widened QRS complex, and heart block.

Cardiac arrest occurs in diastole with increasing levels of potassium.

Osborne (J) wave is seen in hypothermia.

269
Q

A 24-year-old woman has acute renal failure
following postpartum hemorrhage. Laboratory
studies showed serum glucose, 150 mg/dL;
sodium, 135 mEq/L; potassium, 6.5 mEq/L;
chloride, 105 mEq/L; and bicarbonate, 15 mEq/L.
Therapy should include which of the following?

(A) Decrease potassium chloride to
10 mEq/L

(B) Intravenous 0.9% sodium chloride

(C) 100 mL of 50% glucose water with
10 U insulin

(D) Intravenous calcitonin

(E) Intravenous magnesium sulfate

A

(C)

In hyperkalemia, all oral and intravenous potassium must be withheld. Sodium chloride worsens the metabolic acidosis. Sodium bicar- bonate intravenously is given to divert potassium intracellularly by causing alkalosis.

Calcium gluconate (1 g [10 mL of 10% solu- tion]) is given to counteract the effect of potassium on the myocardium. The hypertonic glucose solution stimulates the synthesis of glycogen, which causes cellular uptake of potassium. Small amounts of insulin (1 U/5 g of glucose) is helpful. The usual recommen- ded dose is 100 mL of 50% glucose with 10 U of insulin. Calcitonin is used for treating hypercalcemia. Serum magnesium is also ele- vated in renal failure.

270
Q

A 55-year-old man with Crohn’s disease had
undergone resection of small bowel and anastomosis. Ten days later, he is found to have bilious drainage of 1 L/d from the drains. He is
started on total parenteral nutrition (TPN).
Four days later, his arterial blood gases (ABGs)
are pH, 7.25; PO2
, 98 mm Hg; and PCO2
, 40 mm
Hg. His anion gap is 10. The most likely cause
is which of the following?

(A) Diabetic ketoacidosis

(B) Renal failure

(C) Hypovolemic shock

(D) Small-bowel fistula

(E) Uncompensated metabolic alkalosis

A

(D) This patient has metabolic acidosis with normal anion gap. The normal value of anion gap is 10–15. Loss of bicarbonate (e.g., small- bowel fistula, pancreatic fistula, or diarrhea) and gain of chloride (e.g., administration of ammo- nium chloride or HCl and decreased excretion as in distal renal tubular acidosis) result in meta- bolic acidosis with normal anion gap. In con- trast, in acidosis due to increased production of an organic acid (e.g., ketoacids in diabetes, sulfur and phosphoric acid in renal failure, and lactic acid in shock), the anion gap is increased.

271
Q

A 55-year-old man sustains numerous injuries
involving the abdomen and lower extremities.
During the intra- and postoperative periods,
he is resuscitated with 10 L of Ringer’s lactate
and 2 U of packed red blood cells (RBC). After
initial improvement, he has severe dyspnea on
the second postoperative day. The most useful
initial diagnostic test is which of the following?

(A) Electrocardiogram

(B) Analysis of arterial blood gas

(C) Insertion of a central venous line

(D) Ventilation-perfusion scan

(E) Computed tomography (CT) scan of
abdomen

A

(B) The patient has acute respiratory distress syndrome (ARDS). Measurement of ABGs pro- vides initial evaluation of pulmonary function in terms of oxygenation and ventilation. ECG is valuable for diagnosing myocardial ischemia or cardiac arrhythmias. Ventilation perfusion scanning is used for diagnosing pulmonary embolism. A central venous line provides infor- mation regarding the volume status of the patient, which may be low to normal in ARDS.

272
Q

A 20-year-old man involved in a car crash sustained severe injuries to the chest, abdomen,
and lower extremities. He is intubated and
requires increasing concentration of oxygen to
maintain his PO2
. The pathologic changes do
which of the following?

(A) They cause the alveolar capillary
membrane to become more
impermeable.

(B) They most frequently occur after severe
injuries.

(C) They are associated with low compliance.

(D) They show a characteristic localized
pattern on x-ray.

(E) They involve a decrease in dead-space
ventilation.

A

(C) Increased airway resistance (stiff lung) may be noted early in shock lung. The alveo- lar capillary membrane becomes more per- meable. There is a leak of a high-protein fluid from the capillary to the interstitial tissues and then into the alveoli. This is commonly called ARDS. Sepsis syndrome is the most frequent cause of ARDS (39%), followed by aspiration, multiple transfusion, massive soft- tissue injury, multiple trauma, near drown- ing, fat embolism, DIC, and pancreatitis. ARDS is associated with ventilation–perfusion imbalance. In some areas of lung, there is ven- tilation with no perfusion, whereas, in other areas, nonventilated alveoli are being per- fused. The net result is decrease in functional residual capacity, shunting, and increased dead space ventilation. Chest x-ray reveals diffuse alveolar infiltration, and findings are normal in the initial stage.

273
Q

A 24-year-old woman is scheduled for an elective cholecystectomy. The best method of identifying a potential bleeder is which of the
following?

(A) Platelet count

(B) A complete history and physical
examination

(C) Bleeding time

(D) Lee-White clotting time

(E) Prothrombin time (PT)

A

(B) A history of bleeding should alert the clini- cian to evaluate the underlying cause. The bleed- ing time is influenced by those factors affecting platelet and capillary integrity. Prolongation of the PT may be attributed to decreased absorption of fat-soluble vitamin K, liver impairment, or decrease in the blood components because of consumption.

274
Q

A 24-year-old man who is admitted to the
intensive care unit (ICU) following severe head
injury develops seizures on the fourth day of
hospitalization. His urine output is 500 mL
over 24 hours, sodium is 115 mEq/L, and
serum and urine osmolality are 250 and 800
mOsm, respectively. The metabolic abnormality is due to which of the following?

(A) Administration of D5
W (5% dextrose in
water) and 0.33 normal saline

(B) Syndrome of inappropriate secretion of
antidiuretic hormone (SIADH)

(C) Decreased antidiuretic hormone (ADH)
secretion

(D) Nasogastric suction

(E) Renal insufficiency

A

(B) Possible causes of this syndrome include head injury, central nervous system (CNS) dis- orders, neoplastic diseases, pulmonary dis- eases, drugs, and idiopathic. It results in impaired water excretion characterized by olig- uria, hyponatremia, significantly decreased serum osmolality, and increased urinary osmo- lality. Administration of a hypotonic solu- tion(D5/0.33 NS) would not result in decreased urine output. Decreased ADH secretion would result in an increased urine output as opposed to a decreased urine output. Naso-gastric suc- tion while it can result in a hypo-kalemia, hyponatremia is less likely. Renal insufficiency would likely result in a decreased urine osmolality.

275
Q

A 40-year-old man who weighs 65 kg is being
observed in the ICU. Twenty-four hours postoperatively, he develops convulsions. His
serum sodium is 118 mEq/L. Appropriate
management includes which of the following?

(A) Administration of normal saline (0.9%)

(B) Administration of hypertonic saline (3%)

(C) Emergency hemodialysis

(D) Administration of vasopressin

(E) Administration of Lasix, 40 mg
intravenously (IV)

A

(B) Hyponatremia occurs because of overhydra- tion and/or inadequate sodium replacement. When serum sodium is <130 mEq/L, acute symptomatic hyponatremia is manifested by CNS symptoms due to increased intracranial pressure. Muscle twitching and increased tendon reflexes seen in moderate hyponatremia progress to convulsions, loss of reflexes, and hyperten- sion with severe hyponatremia. Oliguric renal failure may become irreversible if not immedi- ately treated. Mild asymptomatic hyponatremia is treated with fluid restrictions. In the presence of CNS symptoms, the patient should be given hypertonic saline.

276
Q

A 30-year-old man who weighs 60 kg has the following laboratory values: hemoglobin, 10 g/dL;
serum sodium, 120 mEq/L; serum potassium,
4 mEq/L; serum chloride, 90 mEq/L; and serum
CO2 content, 30 mEq/L. What is his sodium
deficit approximately?

(A) 20 mEq

(B) 200 mEq

(C) 400 mEq

(D) 720 mEq

(E) 120 mEq

A

(D) Sodium deficit is estimated by multiplying the decrease in serum sodium times the total body water, which is 60% of body weight:

(normal serum sodium − observed serum sodium) × 0.6 × (total body weight)
= (140 − 120) × 0.6 × 60 = 720 mEq

Half of this amount should be administered over 12–18 hours.

277
Q

A 65-year-old man has urine output of 10 mL/h
following abdominal aortic aneurysmectomy.
Acute tubular necrosis is suggested by the presence of which of the following?

(A) Urine osmolality of more than
500 mOsm/kg

(B) Urine sodium of more than 40 mEq/L

(C) Fractional excretion of sodium of <1%

(D) Blood urea nitrogen (BUN)-to-serum
creatinine ratio (SCR) of more than 20

(E) Urine-to-plasma creatinine ratio (PCR)
of more than 40

A

(B) Oliguria may be prerenal or renal. The fol- lowing table characterizes findings in prerenal failure versus those observed in intrinsic renal failure (acute tubular necrosis).

TABLE 1–1. PRERENAL VS. INTRINSIC RENAL FAILURE (Lange Q&A Surgery)

278
Q

A 30-year-old man with a history of Crohn’s disease of the small bowel is admitted with enterocutaneous fistula. The daily output from the
fistula is 2 L. The approximate composition of
the fluids in mEq/L is which of the following?

      Na   K   Cl   HCO3
(A) 10   26   10     30
(B) 60   10   130    0
(C) 140  5   104    30
(D) 140  5    75    115
(E) 60   30  40    40
A

(C) The composition of various GI secretions is different. They are as follows: A, saliva; B, gas- tric; C, ileal; D, pancreatic; and E, colonic. The composition of intestinal fluid is the closest to that of plasma.

279
Q

A 70-year-old woman has a small-bowel fistula
with output of 1.5 L/d. Replacement of daily
losses should be handled using the fluid solution
that has the following composition in mEq/L.
Na K Cl HCO3
(A) 130 4 109 28
(B) 154 0 154 40
(C) 77 0 77 0
(D) 167 0 0 167
(E) 513 0 513 0

A

(A) The composition of small intestinal fluid is sodium, 140 mEq/L; potassium, 5 mEq/L; chlo- ride, 104 mEq/L; and bicarbonate, 30 mEq/L. Daily losses are best replaced by administra- tion of balanced salt solution (Ringer’s lactate) whose composition is depicted in A. B repre- sents normal saline (0.9%), C is half normal saline (0.45%), D is M/6 sodium lactate, and E is 3% sodium chloride.

280
Q

A 70-year-old man has undergone anterior resection for carcinoma of the rectum. He is extubated in the operating room (OR). In the recovery room, he is
found to be restless with an HR of 136 bpm and a BP of 144/80 mm Hg. ABG analysis on room air reveals pH, 7.24; PCO2, 60 mm Hg; PO2, 54 mm Hg; HCO3, 25 mEq/L; and SaO2, 90%.

The physiologic status can best be described
as which of the following?

(A) Respiratory alkalosis

(B) Respiratory acidosis

(C) Metabolic acidosis

(D) Metabolic alkalosis

(E) Combined respiratory and metabolic
acidosis

A

(B) A decrease in pH below 7.4 indicates acido- sis. PCO2 is increased over 40 mm Hg, suggest- ing respiratory acidosis. To differentiate pure from combined acidosis, pH is calculated based on changes in CO2. A change of 10 mm Hg from 40 mm Hg changes pH by 0.08 from 7.4. In this case, there is a 20 mm Hg increase in PCO2, which would decrease pH by 2 × 0.08 = 0.16 from 7.4 or 7.24. The measured pH is 7.24. Therefore, the patient has pure respiratory acidosis.

281
Q

A 70-year-old man has undergone anterior resection for carcinoma of the rectum. He is extubated in the operating room (OR). In the recovery room, he is
found to be restless with an HR of 136 bpm and a BP of 144/80 mm Hg. ABG analysis on room air reveals pH, 7.24; PCO2, 60 mm Hg; PO2, 54 mm Hg; HCO3, 25 mEq/L; and SaO2, 90%.

Appropriate management for this patient should
be which of the following?

(A) To administer 40% oxygen by mask

(B) Morphine, 2 mg IV

(C) Ringer’s lactate, 250 mL over 1 hour

(D) Intubation and ventilatory support

(E) Deep breathing and coughing

A

(D) Respiratory acidosis in the immediate post- operative period is due to inadequate ventila- tion. Adequate ventilation needs to be restored by prompt intubation and ventilatory support. Use of morphine will further depress the respiration.

282
Q

A 60-year-old woman with mild hypertension is
admitted for elective hysterectomy. On preoperative evaluation, she is found to have osteoarthritis; over the previous 6 months, she had noted
watery diarrhea that was becoming progressively worse. The serum potassium is 3 mEq/L.
Which is the most likely cause of hypokalemia?

(A) Myoglobinemia

(B) Villous adenoma of colon

(C) High-output renal failure

(D) Massive blood transfusion

(E) Spironolactone (Aldactone)

A

(B) Villous adenoma of colon can result in watery diarrhea and hypokalemia. Massive tissue injury producing myoglobinemia is associated with significant release of intracellular potassium. Massive blood transfusion results in release of large amounts of potassium. The ability to excrete potassium is impaired in high-output renal failure. Spironolactone is a potassium- sparing diuretic.

283
Q

A 64-year-old man underwent major abdominal surgery to remove a ruptured aortic aneurysm. Four days after the operation, an attempt was made to wean him off the ventilator. ABG analysis reveals pH, 7.54; PCO2, 30 mm Hg; PO2, 110 mm Hg; HCO3, 30 mEq/L; and SaO2, 99%.

Blood gas analysis reveals which of the
following?

(A) Respiratory acidosis

(B) Metabolic alkalosis

(C) Respiratory alkalosis

(D) Compensated respiratory acidosis

(E) Combined respiratory and metabolic alkalosis

A

(E) A change in PCO2 of 10 mm Hg from the normal value of 40 mm Hg produces a 0.08 change in pH (from 7.4). A PCO2 of 30 mm Hg, representing a decrease of 10 mm Hg, can account for an increase in pH by 0.08 (i.e., 7.4–7.48). The patient’s measured pH is 7.54. The additional increase in pH is due to meta- bolic alkalosis.

284
Q

A 64-year-old man underwent major abdominal surgery to remove a ruptured aortic aneurysm. Four days after the operation, an attempt was made to wean him off the ventilator. ABG analysis reveals pH, 7.54; PCO2, 30 mm Hg; PO2, 110 mm Hg; HCO3, 30 mEq/L; and SaO2, 99%.

What will be the most likely complication due to the metabolic changes experienced by the
patient?

(A) Hypokalemia

(B) Shift of oxyhemoglobin dissociation to
the right

(C) Hyperkalemia

(D) Hypercalcemia

(E) Hyperchloremia

A

(A) Alkalosis is associated with hypokalemia. Hypokalemia can be sudden and severe. It is related to (a) intracellular shift of potassium in exchange for hydrogen and (b) excessive uri- nary potassium loss. The oxyhemoglobin dis- sociation curve is shifted to left, and a decrease in levels of ionized calcium can result in tetany and convulsions.

285
Q

A 42-year-old man with small-bowel fistula has
been receiving TPN with standard hypertonic
glucose-amino acid solution for 3 weeks. The
patient is noticed to have scaly, hyperpigmented
lesions over the acral surfaces of elbows and
knees, similar to enterohepatic acrodermatitis.
What is the most likely cause of the condition?

(A) Copper deficiency

(B) Essential fatty acid deficiency

(C) Excess glucose calories

(D) Hypomagnesemia

(E) Zinc deficiency

A

(E) Zinc is one of the metalloenzymes involved in lipid, carbohydrate, protein, and nucleic acid metabolism. Skin lesions similar to enterohep- atic acrodermatitis are the most common sign seen in zinc deficiency. Other manifestations include hypogonadism, diminished wound healing, and immunodeficiencies. Copper defi- ciency is characterized by microcytic hypo- chromic anemia.

286
Q

A 27-year-old man is involved in a car crash while traveling in excess of 70 mi/h. He sustains an intraabdominal injury and a fracture of the femur. The BP is 60/40 mm Hg, and the hematocrit is 16%.

Which physiologic changes will ensue?

(A) Peripheral vasodilation

(B) Inhibition of sympathetic tone

(C) Temperature rise to 103.8°F

(D) Eosinophilia

(E) Lactic acidosis

A

(E) The fall in pressure will signal changes via baroreceptors located in the arch of the aorta and carotid sinus and will cause sympathetic stimulation with tachycardia, peripheral vaso- constriction, and hypothermia. Eosinopenia rather than eosinophilia is more likely to be present. There is a switch from aerobic to anaer- obic metabolism. Lactic acid accumulation indi- cates an adverse prognosis in shock. There is a progressive deterioration in prognosis as the blood lactate level increases from 1 to above 3 mm/L.

287
Q

A 27-year-old man is involved in a car crash while traveling in excess of 70 mi/h. He sustains an intraabdominal injury and a fracture of the femur. The BP is 60/40 mm Hg, and the hematocrit is 16%.

There is likely to be a proportionately greater
increase in blood flow to which of the following?

(A) Kidneys

(B) Liver

(C) Heart

(D) Skin

(E) Thyroid gland

A

(C) The fall in cardiac output results in a rela- tively larger proportion of blood to be distrib- uted to the heart. The changes are mediated mainly by sympathetic stimulation. There is increased arteriolar and precapillary sphincter tone in the skin and in the renal and splanchnic circulation. In the heart, coronary artery vasodi- lation occurs, which is brought about partly by local release of vasodilator substances (due to hypoxemia and acidosis).

288
Q

A 27-year-old man is involved in a car crash while traveling in excess of 70 mi/h. He sustains an intraabdominal injury and a fracture of the femur. The BP is 60/40 mm Hg, and the hematocrit is 16%.

Initial resuscitation is best done by administration of which of the following?
(A) D5W

(B) D5W and 0.45% normal saline

(C) Ringer’s lactate solution

(D) 5% plasma protein solution

(E) 5% hydroxyethyl starch solution

A

(C) Initial resuscitation of a trauma patient is best done by administering Ringer’s lactate, because it is isotonic, and it is similar to plasma in electrolyte composition. There is no conclu- sive evidence that colloid solutions (albumin, plasma protein solution, or hydroxyethyl starch solution) improve the rate of resuscitation or eventual outcome. D5W and D5W and 0.45% normal saline are hypotonic. Use of crystalloid solutions also aids in the resuscitation of the interstitial compartment.

289
Q

A 30-year-old man is brought to the emergency
department following a high-speed car accident.
He was the driver, and the windshield of the car
was broken. On examination, he is alert, awake,
oriented, and in no respiratory distress. He is
unable to move any of his four extremities; however, his extremities are warm and pink. His vital signs on admission are HR 54 bpm and BP
70/40 mm Hg. What is the diagnosis?

(A) Hemorrhagic shock

(B) Cardiogenic shock

(C) Neurogenic shock

(D) Septic shock

(E) Irreversible shock

A

(C) Neurogenic shock (not to be confused with spinal shock, which is defined by loss of reflexes below the area of spinal cord injury, a neurologic phenomena) is secondary to high spinal cord injury as evidenced by inability to move all four extremities. Neurogenic shock is clinically man- ifested by warm skin, bradycardia, and hypoten- sion. In septic shock, while the skin is warm, the patient usually has tachycardia. In all other types of shock, the skin is cold. Treatment consists of volume replacement with balanced salt solution (lactated Ringer’s solution). On rare occasions, some patients may need vasoconstrictors (e.g., phenylephrine hydrochloride).

290
Q

A 48-year-old man with severe vomiting as a result of gastric outlet obstruction is admitted to the hospital.
There is marked dehydration, with urine output 20mL/h, and the hematocrit is 48%.

Clinical confirmation of pyloric obstruction is
most readily established by which of the following?

(A) Observation of peristalsis from left to
right

(B) Observation of peristalsis from right to
left

(C) Percussion of the upper abdomen

(D) Succussion splash

(E) Auscultation of the upper left abdomen

A

(D) Succussion splash is elicited by placing one hand behind and the other in front of the left abdomen and rib cage and rocking the patient gently between the two hands. In pyloric obstruction, one can feel the fluid hitting the fingers (succussion). Peristalsis is likely to be observed in infants with congenital pyloric stenosis.

291
Q

A 48-year-old man with severe vomiting as a result of gastric outlet obstruction is admitted to the hospital.
There is marked dehydration, with urine output 20mL/h, and the hematocrit is 48%.

What is the predominant metabolic abnormality?

(A) Aspiration pneumonia with respiratory
alkalosis

(B) Hypochloremic alkalosis

(C) Salt-losing enteropathy

(D) Intrinsic renal disease

(E) Metabolic acidosis

A
  1. (B) Duodenal ulcer and gastric carcinoma are the most likely causes of pyloric obstruction in adults. Metabolic alkalosis results from loss of fixed acids from the stomach. The bicarbonate content of the blood accompanies the elevation in pH. In severe metabolic alkalosis, paradoxical loss of acid (hydrogen) in the urine occurs in an attempt to conserve potassium. Hypokalemia worsens the metabolic consequences of meta- bolic alkalosis.
292
Q

A 48-year-old man with severe vomiting as a result of gastric outlet obstruction is admitted to the hospital.
There is marked dehydration, with urine output 20mL/h, and the hematocrit is 48%.

Initial treatment for this patient should include
which of the following?

(A) Administration of 10% dextrose (D10W)
in one-third saline solution IV

(B) Antiemetic

(C) Hemodialysis to correct azotemia

(D) Saline fluid replacement with appropriate
potassium administration

(E) Ringer’s lactate solution

A
  1. (D) Potassium should not be given initially until moderate hydration has been achieved, and urine flow is adequate. Normal saline is required initially to correct the hypochloremia.
293
Q

A 48-year-old man with severe vomiting as a result of gastric outlet obstruction is admitted to the hospital.
There is marked dehydration, with urine output 20mL/h, and the hematocrit is 48%.

Severe hypochloremic metabolic alkalosis fails
to respond to standard therapy. His metabolic
abnormality can be corrected by infusing which
of the following?

(A) Normal saline

(B) Ringer’s lactate solution

(C) Hypertonic saline

(D) 0.1 N hydrochloric acid

(E) 1 N hydrochloric acid

A
  1. (D) Initial management of hypochloremic meta- bolic alkalosis includes administration of isotonic sodium chloride solution with replacement of potassium chloride. In patients refractory to stan- dard therapy use of O.1 N and 0.2 N hydrochlo- ric acid has been shown to be safe and effective therapy. Ammonium chloride solution has also been used, but this can lead to ammonia toxicity, especially in patients with hepatic insufficiency.
294
Q

A 48-year-old man with severe vomiting as a result of gastric outlet obstruction is admitted to the hospital.
There is marked dehydration, with urine output 20mL/h, and the hematocrit is 48%.

In the absence of malignancy, further treatment after appropriate resuscitation should include
which of the following?

(A) Jejunostomy feeding

(B) Vagotomy and drainage

(C) Steroids

(D) No foods given orally (PO) for 6 weeks

(E) Pyloromyotomy alone

A
  1. (B) The actual surgical treatment for obstruc- tion caused by peptic ulcer is controversial. Appropriate gastric surgery with drainage usu- ally is required if pyloric stenosis is severe. Drainage procedures include pyloroplasty, gas- trojejunostomy, or antrectomy. An alternative to vagotomy and drainage would be vagotomy, antrectomy with gastrojejunostomy, or a Billroth II subtotal gastrectomy with gastrojejunal anas- tomosis. It is important to be certain that a gas- tric carcinoma is not the cause of the pyloric outlet obstruction.
295
Q

During cholecystectomy in a 67-year-old woman,
there is severe bleeding from accidental injury to the hepatic artery. The patient requires transfusion of 2000 mL of blood. After the operation, 24-hour urine output varies between 1250 and 2700 mL/d. She was
adequately hydrated, but BUN levels continue to rise 10–12 mg daily over a 5-day period.

What is the main finding?

(A) Progressive bleeding

(B) High-output renal failure

(C) Postcholecystectomy syndrome

(D) Glomerulonephritis

(E) Obstructive jaundice

A

(B) The presence of an adequate urine output does not preclude a diagnosis of high-output renal failure. The mechanism is based in part on prior ischemia to the nephron structure of the kidneys. There may be an initial period of olig- uria. Urea, potassium, and acids are still partly excreted in the urine, and lactate or bicarbonate is given to avoid development of acidosis.

296
Q

During cholecystectomy in a 67-year-old woman,
there is severe bleeding from accidental injury to the hepatic artery. The patient requires transfusion of 2000 mL of blood. After the operation, 24-hour urine output varies between 1250 and 2700 mL/d. She was
adequately hydrated, but BUN levels continue to rise 10–12 mg daily over a 5-day period.

Metabolic changes likely to occur include which
of the following?

(A) Hyperkalemia

(B) Hyponatremia

(C) Hypophosphatemia

(D) Metabolic alkalosis

(E) Hypomagnesemia

A
  1. (A) Potassium should not be given, and potas- sium levels must be monitored carefully to avoid hyperkalemia. Phosphorous and mag- nesium levels may be increased, and hyperna- tremia is likely to occur when fluid is restricted and a solute-poor urine is excreted. Autopsy in patients dying early shows that the distal nephron is affected more than the proximal nephron. Although mortality figures are high, if the patient survives the postoperative period, satisfactory renal function can be anticipated.
297
Q

During cholecystectomy in a 67-year-old woman,
there is severe bleeding from accidental injury to the hepatic artery. The patient requires transfusion of 2000 mL of blood. After the operation, 24-hour urine output varies between 1250 and 2700 mL/d. She was
adequately hydrated, but BUN levels continue to rise 10–12 mg daily over a 5-day period.

Management includes which of the following?

(A) Restriction of fluids to 750 mL/d

(B) 8 L of fluid daily to remove urea

(C) Replacement of fluid loss plus insensible loss

(D) 80 mEq potassium chloride (KCl) per
12 hour

(E) Ammonium chloride IV

A
  1. (C) Marked fluid restriction may result in hypernatremia. If the condition is treated appro- priately, urea nitrogen usually falls after 1 or 2 weeks. In elderly and cardiac patients, pul- monary edema occurs more readily, and diuret- ics may be contraindicated because azotemia may be made more severe. Ammonium chlo- ride would make the acidosis worse. Potassium has to be monitored carefully, because severe hyperkalemia is readily induced.
298
Q

A 14-year-old boy with a known bleeding tendency
since infancy has severe epistaxis. Examination
reveals an equinus contracture of the right leg and a large hemarthrosis.

What is the most likely diagnosis?

(A) Diethylstilbestrol (DES) was taken by the mother during pregnancy

(B) Aplastic anemia

(C) Henoch-Schönlein purpura

(D) Hemophilia

(E) Wilson’s disease with cirrhosis

A
  1. (D) Hemophilia (factor VIII deficiency) usually occurs during infancy. It is sex-linked, recessive, and affects males almost exclusively. DES administered during the mother’s pregnancy has not been incriminated in coagulation disor- ders but is associated with vaginal carcinoma in adolescent girls. Henoch-Schönlein purpura usually occurs about 3 weeks after a streptococ- cal infection and includes joint pain, purpura, and nephritis. Wilson’s disease is associated with a disturbance in copper metabolism.
299
Q

A 14-year-old boy with a known bleeding tendency
since infancy has severe epistaxis. Examination
reveals an equinus contracture of the right leg and a large hemarthrosis.

Treatment should include which of the following?

(A) Penicillamine

(B) Transfusion of factor VIII to 30% of
normal factor levels

(C) Transfusion of factor VIII to 10% of
normal factor levels

(D) Platelet transfusion

(E) Exploration of joint

A
  1. (B) Spontaneous bleeding occurs when factor VIII is reduced below 2–3%. Once serious bleeding occurs, a higher factor VIII activity— probably approaching 30%—is required for adequate hemostasis. The half-life of factor VIII is 8–12 hours. In minor lesions, 10 U/kg body weight of factor VIII is administered. For severe lesions, the dosage is 40–50 U/kg body weight of factor VIII. After major surgical procedures, factor VIII must be given daily for 7–10 days. Penicillamine is used to inhibit excess copper deposition (e.g., in Wilson’s disease).
300
Q

A 10-year-old boy with history of prolonged
bleeding after minor injury is scheduled for tonsillectomy. The bleeding time, PT, and fibrinogen are normal. What would be the most helpful investigation?

(A) Fibrinolysis (euglobulin clot lysis time)

(B) Platelet count

(C) Thrombin time

(D) Partial thromboplastin time (PTT)

(E) Factor VII assay

A
  1. (D) The clinical picture is suggestive of hemo- philia. The normal bleeding time excludes cap- illary fragility or platelet deficiency. If fibrinolysis was evident, the fibrinogen level would be reduced. In the presence of a normal PT, a pro- longed PTT indicates a deficiency of factor VIII, IX, XI, or XII. The PT evaluates the extrinsic coagulation pathway. The normal PT excludes factor VII deficiency. The thrombin time eval- uates fibrinogen to fibrin conversion with an external source of thrombin and will be normal as fibrinogen levels are normal.
301
Q

A 22-year-old man is brought into the emergency department in profound shock after a fall from the fourth floor of a building. After resuscitation, small bowel resection and hepatic segmentectomy are performed at laparotomy. He receives 15 U of packed RBCs, 4 U of fresh-frozen plasma, and 8 L of Ringer’s lactate. On closure, diffuse oozing of blood is noted.

What is the most likely cause?

(A) Hepatic failure

(B) Hypersplenism

(C) Platelet deficiency

(D) Factor IX (Christmas factor) deficiency

(E) Congenital hypoprothrombinemia

A
  1. (C) Thrombocytopenia is the major hemostatic disorder in massive blood transfusion. Platelet transfusion usually is indicated when more than 6–8 U of blood is transfused rapidly. There is the risk of causing hepatitis. Stored blood is defi- cient in factors V and VIII; as such, PT and PTT may be slightly prolonged after massive blood transfusion. Fresh-frozen plasma is the source of factors V and VII, which would be deficient in banked blood. Unless there is previous liver cir- rhosis, the procedures enumerated are unlikely to lead to liver failure. Hypersplenism occurs in patients with enlarged spleens.
302
Q

A 22-year-old man is brought into the emergency department in profound shock after a fall from the fourth floor of a building. After resuscitation, small bowel resection and hepatic segmentectomy are performed at laparotomy. He receives 15 U of packed RBCs, 4 U of fresh-frozen plasma, and 8 L of Ringer’s lactate. On closure, diffuse oozing of blood is noted.

Which test is most likely to be helpful in management of this patient?

(A) Platelet count

(B) Bone marrow biopsy

(C) Liver-spleen scan

(D) Factor VIII assay

(E) Smear for Howell-Jolly bodies

A
  1. (A) Platelet deficiency is likely to be evident, but tests to exclude other causes of bleeding are indicated. The possibility of defibrinogenation, intravascular coagulopathy, or fibrinolysis must be excluded by appropriate coagulation studies. Bleeding from a vein or artery, incom- patible blood transfusion, DIC, acidosis, and hypothermia are other considerations to explain any unusual bleeding after a major surgical procedure.
303
Q

A 22-year-old man is brought into the emergency department in profound shock after a fall from the fourth floor of a building. After resuscitation, small bowel resection and hepatic segmentectomy are performed at laparotomy. He receives 15 U of packed RBCs, 4 U of fresh-frozen plasma, and 8 L of Ringer’s lactate. On closure, diffuse oozing of blood is noted.

Bleeding persists despite all appropriate blood
coagulant replacement, and laparotomy reveals
multiple sites of bleeding from the liver and the
rest of the abdomen. Treatment should include
which of the following?

(A) Hepatic artery ligation

(B) Packing with laparotomy towels

(C) Immediate closure

(D) A large dose of heparin

(E) Solu-Medrol, 1 g IV

A
  1. (B) In desperate cases where bleeding persists despite all other measures, packing the abdomen with laparotomy packs may offer temporary control. The patient is taken to the OR 24–48 hours later for removal of packing after stabi- lization of hemodynamic status and correction of coagulopathy.
304
Q

A 50-year-old suffering from chronic alcoholism
is admitted to the hospital. He has muscle tremors and hyperactive tendon reflexes. Serum magnesium is 1.8 mEq/L (normal 1.5–2.5 mEq/L).
Concerning magnesium, which of the following statements is true?

(A) It is mainly extracellular

(B) Excess may cause a positive Chvostek’s
sign (carpopedal spasm)

(C) Deficiency is treated with parenteral
bicarbonate

(D) Symptoms are due to deficiency of
magnesium

(E) It may become elevated in acute
pancreatitis

A
  1. (D) Symptoms are due to magnesium deficiency. Magnesium is mainly intracellular. Magnesium deficiency occurs in the presence of starvation, malabsorption syndrome, acute pancreatitis, and chronic alcoholism. Symptoms are charac- terized by neuromuscular and CNS hyperactiv- ity, such as muscle tremors, hyperactive tendon reflexes, and tetany with a positive Chvostek sign. The syndrome of magnesium deficiency can exist in the presence of normal serum mag- nesium levels. Magnesium deficiency is treated with parenteral magnesium sulfate or magne- sium chloride.
305
Q

A 30-year-old man with multiple injuries has severe renal insufficiency. On the third day of hospitalization, he is lethargic with generalized weakness and decreased deep tendon reflexes. An electrocardiogram (ECG) reveals a widened QRS complex with elevated T waves.

What is the most likely cause of the patient’s
condition?

(A) Hypokalemia

(B) Hyponatremia

(C) Hypermagnesemia

(D) Hypocalcemia

(E) Hypophosphatemia

A
  1. (C) Symptomatic hypermagnesemia is seen after early thermal injury, massive trauma, surgical stress, and in the presence of severe renal insufficiency. ECG changes resemble those seen with hyperkalemia. Hypokalemia and hypophosphatemia can cause symptoms of generalized weakness, but potassium and phosphorus are increased in renal failure. Hypokalemia is characterized by flattening of T waves and U waves. Hypocalcemia is characterized by hyperactive tendon reflexes. Hyponatremia is characterized by nervous irri- tation as restlessness and convulsions with no specific ECG changes.
306
Q

A 30-year-old man with multiple injuries has severe renal insufficiency. On the third day of hospitalization, he is lethargic with generalized weakness and decreased deep tendon reflexes. An electrocardiogram (ECG) reveals a widened QRS complex with elevated T waves.

What should be the immediate management
of the patient?

(A) Administration of potassium chloride

(B) Administration of calcium chloride

(C) Restriction of fluid intake

(D) Use of Kayexylate enemas

(E) Administration of hypertonic saline

A
  1. (B) Administer calcium chloride. Management of hypermagnesemia involves correction of extracellular volume deficit and acidosis and withholding exogenous magnesium. Calcium chloride should be administered to reverse the ECG changes temporarily. Peritoneal dialysis or hemodialysis is necessary for persistent symptoms or toxicity. Calcium chloride stabi- lizes the cardiac cell membrane and thereby reduces the risk of dysrhythmias.
307
Q

A 45-year-old male with a known history of
alcoholism is admitted with acute pancreatitis.
His serum calcium is 7 mg/dL. Management is
based upon which of the following?

(A) One-fourth of calcium in serum is ionized

(B) Alkalosis increases the ionized calcium
component

(C) Hypocalcemia may cause polyuria and
polydypsia

(D) Determination of serum albumin is
necessary

(E) Treatment should involve intravenous
administration of calcium chloride

A
  1. (D) Determination of serum albumin or protein level is necessary for proper determination of serum calcium level. For every 1 g decrease of serum albumin, the serum calcium level is cor- rected by 0.8. Intravenous administration of calcium chloride is indicated in the presence of symptoms. Approximately 45% of serum cal- cium is ionized and responsible for neuromus- cular stability. Half of the calcium in the blood is bound to protein, and an additional 5% is attached to substances other than protein. Alkalosis decreases the ionized component. Hypercalcemia causes polydypsia and polyuria.
308
Q

A 36-year-old diabetic woman develops metabolic changes following salpingo-oophorectomy.
Serum osmolality of the blood can be calculated
from serum values of which of the following?

(A) Sodium, potassium, chloride, and
bicarbonate

(B) Sodium, potassium, urea, and
hemoglobin

(C) Sodium, potassium, glucose, and urea

(D) Sodium, albumin, urea, and glucose

(E) Sodium, potassium, albumin, and glucose

A
  1. (C) Serum osmolality is calculated from serum values of sodium, potassium, glucose, and BUN by using the formula 2 (Na + K) + BUN/2.8 + glucose/18.
309
Q

In a 12-year-old boy who sustained severe head
injury caused by a fall from the third floor of a
building, the syndrome of diabetes insipidus is
characterized by which of the following?

(A) Low serum sodium

(B) High urinary specific gravity or
osmolality

(C) High serum osmolality

(D) Low urine output

(E) Expanded extracellular fluid volume

A
  1. (C) Injury to the pituitary stalk in major skull fractures involving the base of the skull can result in decreased secretion of vasopressin (ADH). There is increased urine output that is diluted (osmolality <270 mOsm/kg). The extracellular fluid volume is contracted, resulting in high serum osmolality (>300 mOsm/kg) and increased serum sodium.
310
Q

In a 40-year-old woman receiving TPN for
small-bowel fistula, what finding can be attributed to hypophosphatemia?

(A) Increased cardiac output

(B) Diarrhea

(C) Increased energy production

(D) Rhabdomyolysis

(E) Increased white blood cells (WBC)
function

A
  1. (D) Hypophosphatemia results in decreased syn- thesis of phosphorylated intermediate metabo- lites such as adenosine triphosphate (ATP), 2, 3-DPG, and cyclic adenosine monophosphate (cAMP). Deficiency can result in erythrocyte membrane instability, WBC dysfunction, platelet dysfunction, congestive heart failure, arrhyth- mias, weakening of respiration muscles, hemol- ysis, and rhabdomyolysis.
311
Q

A 60-year-old woman who underwent a mastectomy for breast cancer 2 years earlier presents to the emergency department with headache, backache, and frequent vomiting. She is extremely thirsty and stuporous.

Which test is most likely to identify the cause?

(A) CT scan of the head

(B) X-ray of spine

(C) Serum sodium determination

(D) Serum calcium determination

(E) Serum glucose determination

A
  1. (D) The symptoms are suggestive of hypercal- cemia. Major causes of hypercalcemia are cancer with bony metastasis and hyperparathyroidism. Symptoms involve the GI, renal, musculoskele- tal, and CNS.
312
Q

A 60-year-old woman who underwent a mastectomy for breast cancer 2 years earlier presents to the emergency department with headache, backache, and frequent vomiting. She is extremely thirsty and stuporous.

What should be the initial management of the
patient?

(A) Restrict fluid intake

(B) Normal saline infusion

(C) D5W infusion

(D) Thiazide

(E) Hemodialysis

A
  1. (B) Patients with hypercalcemia have decreased extracellular fluid volume due to vomiting and polyuria. Vigorous resuscitation with salt solu- tion will lower the serum calcium by dilution and increased renal excretion. Furosemide and not thiazides increase renal excretion of calcium. Additional therapy includes administration of oral or intravenous inorganic phosphates, corti- costeroid, mithramycin, and calcitonin.
313
Q

A 40-year-old man is found to have severe
metabolic acidosis with a high anion gap. What
is the most likely cause?

(A) Diarrhea

(B) Methanol ingestion

(C) Proximal renal tubular acidosis

(D) Distal renal tubular acidosis

(E) Ureterosigmoidostomy

A
  1. (B) Methanol ingestion results in increased pro- duction of lactic acid causing an increased anion gap. The other conditions listed are associated with normal anion gap. Diarrhea, proximal renal tubular acidosis, and ureterosigmoidostomy result in loss of bicarbonate, while distal renal
    tubular acidosis is associated with decreased acid excretion.
314
Q

An 18-month-old boy slipped and hurt his
right knee while walking. He presents with a
tender, swollen, warm knee with significant
hemarthrosis. His PT is 12 (normal, 13 seconds),
PTT is over 100 (normal, 25 seconds), platelet
count is 300,000/mm3, and bleeding time is
normal. Initial management should consist of which of the following?

(A) Fresh-frozen plasma

(B) Aspiration of knee

(C) Factor VIII concentrate

(D) Passive exercise

(E) Long-leg cast

A
  1. (C) The boy has hemophilia. Management con- sists of infusion of factor VIII concentrate. Bed rest and local cold packs are helpful. Aspiration of the knee to remove blood and passive exer- cise are not recommended for fear of recurrent bleeding. In contrast, active exercise is benefi- cial because movement beyond the point when bleeding can recur is limited owing to pain. Fresh-frozen plasma has a low level of factor VIII (0.6 U/mL) and is not useful because the required volume is excessive. Patients can use long leg splint.
315
Q

A 30-year-old woman with a history of an
uneventful tonsillectomy at age four is scheduled for exploratory laparotomy. Preoperative
assessment that identifies the risk of intraoperative bleeding is which of the following?

(A) Bleeding time

(B) Platelet count

(C) PT and PTT

(D) Complete blood cell count

(E) Obtaining a detailed history

A
  1. (E) Obtaining a detailed history is the most important preoperative information that pre- dicts the risk of unexpected intraoperative bleeding complication. It is even more reliable than laboratory tests.
316
Q

A 43-year-old woman with von Willebrand’s
disease is scheduled for cholecystectomy. It can
be stated that preoperative evaluation will reveal which of the following?

(A) Normal bleeding time, PT, and PTT

(B) Platelet aggregate with restocetin

(C) Increased bleeding time and PTT, and
normal PT

(D) Increased bleeding time and PT, and
normal PTT

(E) Increased bleeding time, and normal PT
and PTT

A
  1. (C) von Willebrand disease is characterized by decreased level of factor VIIIc (procoagulant). It has autosomal-dominant inheritance. These patients have prolonged bleeding times and PTT, with normal PTs. In contrast to platelets of normal patients that aggregate when resto- cetin is added, in von Willebrand’s resease, platelets fail to aggregate in presence of restocetin.
317
Q

Following admission to the emergency department, a 26-year-old woman with severe menorrhagia states that both her father and sister have a bleeding disorder. The hemostatic disorder transmitted by autosomal-dominant
mode is which of the following?

(A) Factor X deficiency

(B) von Willebrand’s disease

(C) Factor VIII deficiency (true hemophilia)

(D) Factor IX deficiency (Christmas disease)

(E) Factor V deficiency (parahemophilia)

A
  1. (B) von Willebrand disease is the most common hemostatic disorder transmitted by autosomal- dominant mode. Other disorders transmitted by this mode are hereditary hemorrhagic telangiectasia and factor XI deficiency. Diseases transmitted by an autosomal-recessive mode are factor X, factor V, factor VII, and factor I deficiencies. Factor VIII (true hemophilia) and factor IX (Christmas disease) deficiencies are sex-liked recessive.
318
Q

A 75-year-old man is found to have prolonged
bleeding from intravenous puncture sites. Platelet aggregation is inhibited by which of
the following?

(A) Adenosine diphosphate (ADP)

(B) Calcium

(C) Magnesium

(D) Aspirin

(E) Serotonin

A
  1. (D) ADP, serotonin, and thromboxane A2 are important mediators of platelet aggregation. In the presence of calcium, magnesium, and platelet factor 4, they cause release of platelet content and their granules resulting in the for- mation of a platelet plug. This process is inhib- ited by aspirin.
319
Q

A 45-year-old woman with deep vein thrombosis is taking warfarin (Coumadin), 5 mg/d.

Seven days after initiation of therapy, she has
warfarin-induced skin necrosis. Which of the
following statements regarding this condition
is true?

(A) It commonly occurs after warfarin
therapy.

(B) It usually involves the upper extremities.

(C) It improves with an increase in the dose
of Coumadin.

(D) It improves with a decrease in the dose
of Coumadin.

(E) It requires cessation of Coumadin and
infusion of heparin.

A
  1. (E) Requires cessation of Coumadin and infu- sion of heparin. Warfarin (Coumadin)-induced skin necrosis is a rare complication with high morbidity and mortality. It usually occurs 3–10 days after initiation of therapy, affects women more commonly than men, and most often involves the skin of thighs, buttocks, abdomen, and breast. The exact mechanism is unknown but may be related to depression of protein C levels in some patients. Management involves immediate cessation of Coumadin and admin- istration of heparin IV.
320
Q

A 50-year-old man with atrial fibrillation is taking
warfarin (Coumadin). The effect of Coumadin is
decreased by which of the following?

(A) The presence of vitamin K deficiency

(B) Phenylbutazone

(C) Quinidine

(D) Barbiturates

(E) Thyrotoxicosis

A
  1. (D) Patients receiving barbiturates, oral contra- ceptive agents, and corticosteroids often require larger amounts of Coumadin to maintain ade- quate anticoagulation. In patients with vitamin K deficiency or impaired liver function and in those with thyrotoxicosis, there is increased effect of Coumadin. Also, the cholesterol-lowering agent clofibrate, D-thyroxine, and certain antibiotics given concomitantly with Coumadin enhance its anticoagulant effect. It is important to adjust the dose of Coumadin when initiating anticoag- ulation therapy in such patients.
321
Q

After undergoing a transurethral resection of the
prostate, a 65-year-old man experiences excessive
bleeding attributed to fibrinolysis. It is appropriate to administer which of the following?

(A) Heparin

(B) Warfarin (Coumadin)

(C) Volume expanders and cryoprecipitate

(D) Aminocaproic acid (Amicar)

(E) Fresh-frozen plasma and vitamin K

A
  1. (D) Fibrinolysis may be primary or acquired. Primary fibrinolysis is seen after fibrinolytic ther- apy with streptokinase or urokinase; surgical procedures on the prostate gland (which is rich in urokinase) and severe liver failure. Secondary fibrinolysis is most commonly seen in DIC. If the PT, PTT, and platelet count are normal, DIC is unlikely to be present. Aminocaproic acid inhibits plasminogen activation to plasmin and can be used if there is excessive fibrinolysis. It must not be given in DIC, because serious intravascular clotting may occur.
322
Q

A 64-year-old woman undergoing radical hysterectomy under general anesthesia is transfused with 2 U
of packed RBCs.

A hemolytic transfusion reaction during anesthesia will be characterized by which of the following?

(A) Shaking chills and muscle spasms

(B) Fever and oliguria

(C) Hyperpyrexia and hypotension

(D) Tachycardia and cyanosis

(E) Bleeding and hypotension

A
  1. (E) In the anesthetized patient, the classic signs of transfusion reaction are masked. The sudden unexplained onset of bleeding and hypoten- sion should include transfusion reaction in the differential diagnosis. In the conscious patient, chills, fever, pain in the lumbar region, a tight sensation over the chest, flushing of the face, and dark-colored urine may be evident.
323
Q

A 64-year-old woman undergoing radical hysterectomy under general anesthesia is transfused with 2 U
of packed RBCs.

The specific test to identify the cause of transfusion reaction for the patient is which of the following?

(A) PT

(B) PTT

(C) Platelet count

(D) Bleeding time

(E) Free plasma hemoglobin

A
  1. (E) Acute hemolytic transfusion reaction due to transfusion of incompatible blood in a patient
    under general anesthesia usually presents as gen- eralized bleeding due to DIC. PT, PTT, and bleed- ing time will be abnormally high, and platelets may be decreased because of DIC. The most spe- cific tests to determine hemolysis are free plasma hemoglobin and hemoglobinuria. The labora- tory criteria are hemoglobinuria with a concentration of free hemoglobin over 5 mg/dL, a serum hepatoglobin level below 50 mg/dL, and serological criteria to show antigen incompati- bility of the donor and recipient blood.
324
Q

A 41-year-old woman has an episode of mild
right upper quadrant (RUQ) pain associated
with jaundice that resolves completely with
antibiotics. Workup reveals numerous large
stones in the gallbladder. The patient has polycythemia vera, a hematocrit of 58%, and a
platelet count of 1.8 million. What is the preferred course of treatment for this patient?

(A) She should be referred to the medical
clinic for follow-up care and be observed.

(B) She should undergo phlebotomy and
then be scheduled for cholecystectomy.

(C) She should be treated with chlorambucil
for 6 weeks and then undergo
cholecystectomy.

(D) She should receive miniheparin and
urgent cholecystectomy.

(E) She should undergo cholecystectomy.

A
  1. (C) Patients with polycythemia vera do poorly in general surgery if they have not had appro- priate treatment to reduce the RBC and platelet count. With chlorambucil treatment, elective cholecystectomy should be performed to avoid the possible need to perform the operation on an emergency basis when the patient is not fully prepared.
325
Q

A 56-year-old man underwent prostatectomy.
He bled excessively and urgently required
blood over and above what had been requested
before surgery. In deciding on an appropriate
blood transfusion protocol, what should be
kept in mind?

(A) Group AB is the universal donor.

(B) Serum from the recipient stored for
1 week is suitable for testing.

(C) Hypothermia is indicated if cryoglobulin
is found.

(D) Cross-matching should be done before
dextran administration.

(E) Fresh-frozen plasma can be given instead of 4 U of packed cells.

A
  1. (D) Cross-matching should be done before dex- tran administration. Group O is the universal donor, and if there is insufficient time to do appropriate cross-matching of blood, this type of blood should be used. Serum of the recipient should be <24 hours old, because antigenicity may be altered in blood stored for a longer time. Before hypothermia is undertaken, the patient’s (recipient’s) blood should be tested for cold agglutinin titer. Cryoglobulin may be present in patients with lymphoma or leukemia. Blood must be given at room temperature to such patients.
326
Q

A 60-year-old man with carcinoma of the
esophagus is admitted with severe malnutrition. Nutritional support is to be initiated.

What should be his daily caloric intake?

(A) 1 kcal/kg body weight/day

(B) 5 kcal/kg body weight/day

(C) 15 kcal/kg body weight/day

(D) 30 kcal/kg body weight/day

(E) 100 kcal/kg body weight/day

A
  1. (D) In general, total caloric needs for the majority of patients ranges between 25 and 35 kcal/kg/d. An alternative formula for calcu- lating daily caloric requirements is the Harris- Benedict equation, which is based on sex, age, weight, and height. The caloric requirements of humans also varies by amount of activity, degree of stress of surgery, trauma, sepsis, or burns.
327
Q

TPN is initiated in a 44-year-old woman with
Crohn’s disease. In parenteral alimentation,
carbohydrates should be provided in an optimal ratio of which of the following?

(A) 1 kcal/g nitrogen

(B) 5 kcal/g nitrogen

(C) 10 kcal/g nitrogen

(D) 100 kcal/g nitrogen

(E) 1000 kcal/g nitrogen

A
  1. (D) The baseline protein requirements are cal- culated as 1 g/kg/d. Following stress, there is an increased protein requirement, and protein intake should be 1.5 g/kg/d after surgery, 2 g/kg/d after polytrauma, and after sepsis. Glucose and amino acids must be infused simultaneously to appropriately utilize nitrogen.
    The ideal ratio is 100 nonprotein kcal/g of nitrogen. In starvation, the nonprotein calorie- to-nitrogen ratio of 150 kcal/g is adequate.
328
Q

After undergoing subtotal gastrectomy for carcinoma of the stomach, a 64-year-old woman is
receiving peripheral parenteral nutrition. To
increase calories by the peripheral route, what
should be prescribed?

(A) D5W in normal saline

(B) Multivitamin infusion

(C) D25W (25% dextrose in water)

(D) Soybean oil

(E) Lactulose

A
  1. (D) Lipid emulsions derived from soybean or safflower oils are widely used. One of the real advantages of lipid emulsion is that a large amount of calories can be provided through a peripheral vein. The 10% solution provides 4.62 kJ/mL and the 20% solution, 9.24 kJ/mL. Dextrose concentration in peripheral route is 10%. Concentrations >10% require administra- tion into a central vein to prevent phlebitis owing to hypertonicity of the solutions. Lactulose is used to treat hepatic encephalopathy.
329
Q

A 35-year-old man with duodenal stump leak
after partial gastrectomy is receiving central parenteral nutrition containing the standard
D25W, 4.25% amino acid solution. Which is TRUE of essential fatty acid deficiency seen after hyperalimentation?

(A) It occurs if soybean oil is given only once weekly.

(B) It is usually noted at the end of the first week.

(C) It causes dry scaly skin with loss of hair.

(D) It is accompanied by
hypercholesterolemia.

(E) It is treated with insulin.

A
  1. (C) Essential fatty acid deficiency usually occurs if hyperalimentation is extended for more than 1 month and when soybean oil is not administered at least twice a week. There is a decrease in linolenic, linoleic, and arachidonic acids and an increase in oleic and palmitoleic acid. In addition to the skin changes, there may be poor wound healing, increased susceptibil- ity to infection, lethargy, and thrombocytope- nia. It is characterized by a triene-to-tetraene ratio >0.4.
330
Q

In metabolic alkalosis, there is which of the
following?

(A) Gain in fixed acid

(B) Loss of base

(C) Hyperkalemia

(D) Rise in base excess

(E) Hyperchloremia

A
  1. (D) In metabolic alkalosis, there may be a loss of fixed acids or excess of base. It is associated with hypokalemia because of renal conser- vation of H+ ions and urinary potassium loss. Loss of hydrochloric acid as seen in vomiting in patients with pyloric obstruction results in hypochloremic, hypokalemic, metabolic alkalosis.
331
Q

Following urinary tract infection associated
with extraction of a bladder stone, a 64-year-old woman developed gram-negative septicemia.

Which statement is true for gram-negative bacterial septicemia?

(A) Pseudomonas is the most common
organism isolated.

(B) Many of the adverse changes can be accounted for endotoxin release.

(C) The cardiac index is low.

(D) Central venous pressure (CVP) is high.

(E) Endotoxin is mainly a long-chain peptide.

A
  1. (B) Many of the adverse changes can be accounted for by endotoxin release. Escherichia coli is the most common organism involved in gram-negative septicemia, followed by Klebsiella, Aerobacter, Proteus, and Pseudomonas. The car- diac index is high, peripheral resistance is decreased, and CVP is low to normal. The most common conditions leading to gram-negative sepsis are those of the urinary tract, followed by respiratory and biliary tract and abdominal visceral infections. Endotoxins are lipopolysac- charide complexes. The lipid A portion is prob- ably responsible for the toxicity.
332
Q

In septic shock, which of the following is true?

(A) The mortality rate is between 10% and 20%.

(B) Gram-negative organisms are involved
exclusively.

(C) The majority of patients are elderly.

(D) The most common source of infection is
the alimentary tract.

(E) Two or more organisms are responsible
in most cases.

A
  1. (C) Most patients are elderly. The underlying conditions leading to septic shock occur more commonly in elderly patients. The mortality is higher in this patient population. The overall mortality rate exceeds 40–50%. Gram-positive organisms, parasites, or fungi also may be responsible. The genitourinary and respiratory tracts are more common sources for initiating sepsis. Two or more organisms are found in 10–20% of cases.
333
Q

A 68-year-old man has a history of myocardial
infarction. He undergoes uneventful left hemicolectomy for carcinoma of the colon. In the recovery room, he is hypotensive and given a
fluid bolus of 500 mL Ringer’s lactate over
30 minutes. He is intubated, and his neck veins are distended. His HR is 130 bpm, his BP is
80/60 mm Hg, and his urine output is 20 mL
over the last hour. What should be the next
step in his management?

(A) Administration of Ringer’s lactate,
500 mL over 1 hour

(B) Administration of dopamine

(C) Insertion of a Swan-Ganz catheter

(D) Administration of Lasix

(E) Extubation of the patient

A
  1. (C) The patient’s clinical picture is suggestive of cardiogenic shock. However, he may still be hypovolemic, because distension of neck veins does not accurately reflect the filling pressures of the heart. A Swan-Ganz catheter should be inserted for appropriate assessment of hemody- namic status and institution of appropriate ther- apy. Fluid therapy will worsen cardiogenic shock, and Lasix will make the patient hypov- olemic. Dopamine will increase BP but is delete- rious to the heart. The patient should not be extubated until he is stable.
334
Q

A 75-year-old woman who is in the ICU after undergoing cholecystectomy for acute cholecystitis is hypotensive and tachycardic.

Pulmonary capillary wedge pressure (PCWP)
is elevated to 18 mm Hg, and cardiac output is
3 L/min. She is in shock best described as
which of the following?

(A) Hypovolemic shock

(B) Septic shock

(C) Cardiogenic shock

(D) Anaphylactic shock

(E) Neurogenic shock

A
  1. (C) Low cardiac output in the presence of ele- vated filling pressures is characteristic of cardio- genic shock. PCWP is decreased in all the other types of shock.
335
Q

A 40-year-old woman with deep vein thrombosis is being treated with IV heparin, 1000U/h. On the seventh day of treatment, her laboratory values are hemoglobin, 14 g/dL; WBC count, 7600/mm3; platelet count, 30,000/mm3;
PT, 13 seconds (control, 12.5 seconds); and PTT, 50 seconds (control, 26 seconds). What management would be appropriate?

(A) Continue with heparin at the same
dosage

(B) Increase heparin

(C) Decrease heparin

(D) Discontinue heparin

(E) Continue heparin and start warfarin (Coumadin)

A
  1. (D) Thrombocytopenia is a common complica- tion of heparin therapy. The most common form, type I (seen in up to 30% of patients), is a milder form that occurs after 2–3 days of heparin therapy. The platelet count remains over 50,000/mm3 and has no clinical significance. Type II, seen in 1–2%, usually occurs 7–10 days after heparin treatment. It is immune mediated and can be caused by heparin therapy in any form, in any dose, including heparin flushes and heparin-bonded intravenous catheters. Treat- ment consists of immediate cessation of heparin administration in any form.
336
Q

A 55-year-old man involved in an automobile
accident is unresponsive and is intubated at the
scene. On arrival in the emergency department,
he responds to painful stimulation. His systolic
BP is 60 mm Hg, his HR is 140 bpm, his neck
veins are distended, and his breath sounds are
absent on the left side. Immediate management
should involve which of the following?

(A) Insertion of a central venous line on the
right side

(B) Insertion of an 18-gauge needle in the
left second intercostal space

(C) Pericardiocentesis

(D) Peritoneal lavage

(E) CT scan of head

A
  1. (B) The patient has tension pneumothorax, as evidenced by distended neck veins and absent breath sounds. Increased intrathoracic pressure interferes with venous return to the heart, resulting in shock. Immediate management should be insertion of a large-bore needle in the left second intercostal space, followed by insertion of a chest tube. In a trauma patient, venous access should be achieved by inserting two large-bore (16-gauge) angiocatheters in the cubital veins. Insertion of a central venous line on the right side should not be done, because it carries the risk of producing pneumothorax in the opposite side.
337
Q

A 25-year-old man sustained laceration of the
liver and rupture of the spleen in an automobile accident. He was hypotensive for more
than 1 hour and received 10 L of crystalloids
and 10 U of blood. On the second postoperative
day, he is intubated, his HR is 120 bpm, his BP
is 110/60 mm Hg, his urine output is 40 mL/h,
and his CVP is 13 cm H2
O. His ABGs on 70%
oxygen reveal a pH of 7.42, a PO2 of 58 mm Hg,
and a PCO2 of 35 mm Hg. What is the most
appropriate management?

(A) Increase the fraction of inspired oxygen
(FiO2).

(B) Increase the tidal volume (VT).

(C) Administer Lasix, 20 mg IV.

(D) Institute positive end-expiratory pressure
(PEEP).

(E) Decrease FiO2.

A
  1. (D) Institute positive end-expiratory pressure (PEEP). This patient has developed ARDS, which is associated with a significant decrease in functional residual capacity (FCR) of the lungs from collapse of alveoli and increased shunt from perfusion of unventilated alveoli. The most appropriate way to improve his oxy- genation is by instituting PEEP.
338
Q

A 40-year-old paraplegic is taken to the OR for
cholecystectomy for acute cholecystitis. She is given succinylcholine before intubation. Immediately after induction of anesthesia, she develops cardiac arrest. What is the most likely cause?

(A) Esophageal intubation

(B) Hyperkalemia

(C) Perforation of gallbladder

(D) Hypovolemic shock

(E) Myocardial infarction

A
  1. (B) Administration of a depolarizing anesthetic agent such as succinylcholine in quadriplegics, in paraplegics, or after burns and severe trauma can result in life-threatening hyperkalemia from release of intracellular potassium.
339
Q

A 70-year-old woman has low cardiac output
with increased PCWP and increased systemic
vascular resistance. What should be the drug of
choice?

(A) Dopamine

(B) Norepinephrine

(C) Dobutamine

(D) Epinephrine

(E) Phenylephrine

A
  1. (C) Dobutamine is the drug of choice for improving cardiac function. It is a b -receptor
    1
    agonist and increases myocardial contractibil-
    ity and also reduces afterload by b2 effect. Dopamine at low doses (1–3 mg/kg/min) stim- ulates dopaminergic receptors and increases renal blood flow. At moderate doses (3–10 mg/kg/min), it stimulates b-receptors, resulting in a positive inotropic and chronotropic effect. Systolic and mean BP are increased; whereas, diastolic BP is usually unchanged. At higher doses (10–20 mg/kg/min), stimulation of a- receptors occurs and it significantly increases systemic vascular resistance. Norepinephrine, epinephrine, and phe-nylephrine are powerful vasoconstrictors.
340
Q

A 60-year-old man had undergone exploratory
laparotomy for perforated gastric ulcer with
severe peritoneal contamination. Six hours after surgery, he is tachycardic, hypertensive, and has shallow respirations. Intubation and institution of ventilatory support is indicated in the
presence of which of the following?

(A) Respiratory rate of 23 breaths/min

(B) PaCO2 of 45 mm Hg

(C) PaO2 of 55 mm Hg on room air

(D) HR of 140 bpm

(E) BP of 150/100 mm Hg

A
  1. (C) The criteria for need for ventilatory support are apnea, respiratory rate >30 breaths/min, PaO2 <60 mm Hg on room air, and PaCO2 >55 mm Hg (except in patients with chronic obstructive pulmonary disease [COPD]).
341
Q

A 60-year-old man is being weaned from a ventilator in the ICU. The likelihood that weaning
is going to fail is suggested by the presence of
which of the following?

(A) A respiratory rate of 24 breaths/min

(B) A PaO2 of 80 mm Hg on FiO2 of 40%

(C) A vital capacity (VC) of 5 mL/kg body
weight

(D) A minute ventilation of 8 L/min

(E) A maximum negative inspiratory
pressure of −30 cm H2O

A
  1. (C) Vital capacity (VC) of 5 mL/kg body weight. See Answer 76.
342
Q

A patient is being weaned from mechanical
ventilation. Weaning parameters are obtained
prior to deciding on extubation. Successful
weaning from a ventilator is suggested by the
presence of which of the following?

(A) An alveolar arterial gradient of more
than 350 mm Hg

(B) A PaO2/FiO2 ratio of <200

(C) A PaCO2 over 55 mmHg

(D) A tidal volume of over 5 mL/kg

(E) A minute ventilation of 12 L/min

A
  1. (D) Successful weaning from the ventilator is suggested by the presence of
    (a) PaO2 of 70 mm Hg or more with an FiO2 of 0.35 or less
    (b) An alveolar arterial gradient of <350 mm Hg
    (c) A PaO -to-FiO ratio of >200 22
    (d) A PaCO2 of over 30 mm Hg and <55 mm Hg
    (e) A VC of more than 10–15 mL/kg
    (f) A maximum negative inspiratory force of more than -25 cm H2O
    (g) A minute ventilation of <10 L/min
    (h) A tidal volume of over 5 mL/kg
    (i) A respiratory rate of <30 breaths/min
343
Q

A 55-year-old man with oat cell carcinoma of
the lung is suspected to have SIADH. This is
characterized by which of the following?

(A) Decreased total body water (TBW)

(B) Low serum sodium

(C) Increased urine output

(D) Urine sodium of <10 mEq/L

(E) Low urinary specific gravity

A
  1. (B) Patients with SIADH have low urinary output with hyponatremia. Urine-specific grav- ity or osmolality is increased, urinary excretion of sodium is increased (>20 mEq/L), and TBW is increased as manifested by low serum osmo- lality. SIADH is seen after various CNS disor- ders, in neoplastic disease, pulmonary diseases, and with some drugs and may be idiopathic.
344
Q

Following surgery for a perforated appendix
with generalized peritonitis and multiple intraabdominal abscess, a 25-year-old man is admitted to the ICU. On the third postoperative day, he continues to be febrile and has a nasogastric tube.
What is the metabolic characteristic seen in him?

(A) A decrease in energy expenditure

(B) Fat as his primary fuel

(C) Respiratory quotient of 0.6–0.7

(D) Proteolysis

(E) Decreased hepatic synthesis of protein

A
  1. (D) The metabolic response to stress is different to that seen following starvation, as illustrated in Table 1–2.
345
Q

Increase in energy expenditure by 100% over normal, or two times greater than normal, is
seen in a patient with which of the following?

(A) Pyloric obstruction from chronic
duodenal ulcer

(B) Fractured femur

(C) Perforated diverticulitis of colon

(D) Severe thermal burns of more than 30%
total body surface area (BSA)

(E) Right inguinal herniorrhaphy for
incarcerated inguinal hernia

A
  1. (D) Resting energy expenditure is decreased fol- lowing starvation (e.g., in the patient with pyloric obstruction) and increased after the stress of sur- gery, trauma, or sepsis. The increase in energy expenditure correlates with the severity of insult being 1.2 times greater after minor operation (e.g., right inguinal herniorrhaphy), 1.35 times greater after skeletal trauma (e.g., fractured femur), 1.6 times greater after major sepsis (e.g., perforated diverticulitis), and 2 times greater after severe thermal burns.
346
Q

A 30-year-old man with a gunshot wound to
the abdomen has severe injuries involving the
liver, duodenum, pancreas, and colon.

Why is parenteral nutrition support preferred over
enteral nutrition support?

(A) It is less expensive.

(B) It preserves gut mucosal mass and
mucosal immunity.

(C) It prevents gut permeability and
translocation.

(D) It is easy to start and administer nutrient
requirement rapidly.

(E) It attenuates hypermetabolic response to surgery.

A
  1. (D) It is easy to start and administer nutrient requirements rapidly. Parenteral nutrition should be administered when enteral access cannot be obtained, when enteral nutrition support fails to meet nutritional requirements, or when feeding into the GI tract is contraindicated. Current evi- dence suggests that in addition to safety, con- venience, and cost, enteral feeding is well tolerated, preserves gut mucosal mass and normal gut flora, prevents increased gut per- meability to bacteria and other toxins, main- tains mucosal immunity, and attenuates the hypermetabolic response to surgery. As com- pared to parenteral nutrition, enteral nutri- tion is also associated with significantly reduced septic complications. Therefore, enteral feeding is preferred over TPN when feasible.
347
Q

A 24-year-old man with multiple injuries is
receiving standard TPN. The following is true
regarding glutamine.

(A) It is a major fuel for the brain.

(B) It is an essential amino acid.

(C) It is a major fuel for the gut.

(D) It is synthesized de novo in the kidney.

(E) It is a component of TPN solutions.

A
  1. (C) It is a major fuel for the gut. It is readily syn- thesized de novo in skeletal muscle, lung, and liver. Glutamine is a nonessential amino acid. It is not a component of presently available TPN solutions because of its lack of stability. Glutamine is a major fuel for the small intes- tinal mucosa and other replicating cells such as lymphocytes, macrophages, fibroblasts, and endothelial cells. Glucose is the primary source of fuel for the brain.
348
Q

A 50-year-old man with small-bowel fistula has
been receiving TPN for the previous 3 weeks
through a single-lumen central venous catheter.
He is scheduled for exploratory laparotomy
and closure of fistula. On the morning of the
day of surgery, TPN is discontinued and intravenous infusion with balanced salt solution
(Ringer’s lactate) is started. An hour later, the
patient is found to be anxious, sweating, and
tachycardic. What is the most likely cause?

(A) Anxiety

(B) Hypoglycemia

(C) Hypovolemia

(D) Unexplained hemorrhage

(E) Hyperglycemia

A
  1. (B) Patients on TPN with hypertonic glucose solutions have elevated islet-cell production of insulin. Sudden cessation of TPN can lead to rebound hypoglycemia, because pancreatic islet-cell insulin secretion is not immediately downregulated. Symptoms are attrbutable to high catecholamine release secondary to hypo- glycemia. In general, the TPN rate should be reduced to 50 mL/h during surgery. This pre- vents both hypoglycemia and the hyper- glycemia seen with higher infusion rates. Weaning from TPN should be done gradually over 24–48 hours. In instances where TPN is
    discontinued suddenly, a solution of D10W should be administered in the interim.
349
Q

A 40-year-old woman with inflammatory bowel
disease has been receiving TPN for over 3 weeks. Workup reveals pelvic abscess. She undergoes exploratory laparotomy, resection of small bowel with anastomosis, and drainage of pelvic
abscess. During surgery, TPN is maintained at
the original rate of 125 mL/h. In the recovery room, the patient is found to have a urine output
of 200 mL/h. CVP is 1, and laboratory results are
Na, 149; K, 3.5; Cl, 110; HCO3
, 18; BUN, 40; and
creatinine, 1 mg/dL. Which of the following
statements is true regarding this condition?

(A) The patient’s urine output is secondary
to fluid overload during surgery.

(B) The patient is in high-output renal failure.

(C) Hyperosmolar-nonketotic coma will
develop if the condition is not aggressively treated.

(D) Diuresis is a normal response to stress
of surgery.

(E) Potassium supplementation is not
indicated.

A
  1. (C) Hyperosmolar-nonketotic coma is a seri- ous complication seen when an excessive amount of glucose is given, especially in the presence of sepsis, steroids, or inadequate insulin. Furthermore, the combination of sur- gery and sepsis results in an increased insulin- resistant state. The increased urine output is secondary to osmolar load from blood glu- cose. Low CVP, hypernatremia, and BUN-to- creatinine ratio over 20 suggest hypovolemia and not fluid overload. Normal creatinine level and BUN-to-creatinine ratio over 20 rules out high-output renal failure. The stress of surgery is characterized by water retention and not diuresis. Management consists of aggressive hydration, discontinuation of TPN, and insulin drip. Insulin drives the potassium intracellu- larly and potassium must be replaced.
350
Q

A 42-year-old man who weighs 60 kg is receiving 3 L of standard hypertonic 25% glucoseamino acid solution. He has no history of smoking or bronchial asthma. In the ICU, he is
alert, afebrile, and hemodynamically stable,
but he remains intubated and attempts to wean
him off the ventilator have been unsuccessful.
What is the most likely cause?

(A) Copper deficiency

(B) Excess fat calories

(C) Excess glucose calories

(D) Excess amino acids

(E) Inadequate glucose calories

A
  1. (C) Glucose infusion should not exceed 4–5 mg/kg/min, equivalent to 365–432 g for this patient. The patient is receiving 750 g of glucose. Glucose has a respiratory quotient of 1. Excess glucose results in increased produc- tion of CO2, making it difficult to wean the patient off ventilator. Treatment consists of reducing glucose load and providing fat calo- ries (up to 40% of total calories). Fat has a res- piratory quotient of 0.7, resulting in decreased production of CO2.
351
Q

An 85-year-old male is admitted to the ICU in
septic shock. A pulmonary artery (PA) catheter is placed. The PA catheter does not directly
measure which one of the following?

(A) PA systolic pressure

(B) PCWP

(C) Systemic vascular resistance

(D) Right ventricular diastolic pressure

(E) Right atrial pressure

A
  1. (C) Systemic vascular resistance (an approxi- mation of afterload) is a calculated value. All the other choices are directly measured.
352
Q

A 50-year-old woman with adult respiratory
distress syndrome (ARDS) is intubated. The oxyhemoglobin curve is shifted to the right with increased oxygen delivery by which of
the following?

(A) Metabolic acidosis

(B) Older age

(C) Decreased 2,3-diphosphoglycerate (DPG)

(D) Decreased thyroid hormone level

(E) Hypothermia

A
  1. (A) The oxyhemoglobin dissociation curve is a convenient method to study the affinity of hemoglobin for oxygen. It is S-shaped, which provides an efficient method of uptake and release of oxygen. It holds on to the oxygen at high concentrations and as the blood enters the lower pressures encountered in the capillaries, it releases the oxygen. Hemoglobin is 75% satu- rated at a PO2 of 40 mm Hg and 50% saturated at a PO2 of 27 mm Hg. At the peripheral tissues, a right or left shift does have a real impact on the affinity of hemoglobin for oxygen. If the S-curve is shifted to the right, there is a decreased affin- ity of hemoglobin for oxygen (more oxygen is released). A right shift occurs with increase in 2,3-DPG, acidosis, increase in temperature, and increase in hormones (cortisol, thyroid, or aldos- terone). A left shift occurs with a decrease in temperature, alkalosis, low DPG, carboxyhemo- globinemia, and old age.
353
Q

A 70-year-old man was administered 20,000 U
of heparin before femoral artery embolectomy.
Following surgery, he is noted to have generalized bleeding from the wound margins.
Immediate management should consist of
administration of which of the following?

(A) Fresh-frozen plasma

(B) Cryoprecipitate

(C) Platelet transfusion

(D) Intravenous protamine sulfate

(E) Intravenous sodium bicarbonate

A
  1. (D) Intravenous protamine sulfate. The cause of bleeding is circulating heparin. The antico- agulative effect of heparin can be immediately neutralized by intravenous protamine sulfate. One milligram of protamine sulfate usually neutralizes 100 U of heparin. Fresh-frozen plasma is given to counteract the effect of war- farin (Coumadin). Cryoprecipitate is useful in treating patients with hemophilia. Intravenous sodium bicarbonate is indicated after mis- matched blood transfusion to alkalinize the urine. Platelet transfusions are necessary to cor- rect dilutional thrombocytopenia seen after massive blood transfusion.
354
Q

A 70-year-old female has been admitted to your
ICU in shock. You determine that a PA catheter
is needed. Which of the following is not a
known complication associated with the insertion of PA catheter?

(A) Transient arrhythmias such as ventricular
tachycardia

(B) Right bundle branch block

(C) Pneumothorax

(D) Mural thrombus

(E) Cardiac perforation

A
  1. (D) Transient arrhythmias and right bundle branch block are seen during the insertion of the PA catheter as it may hit the wall of the right ventricle causing these electrical distur- bances. Pneumothorax is a risk associated with the insertion of the introducer for the PA catheter. Cardiac perforation is certainly a risk anytime a catheter is being placed through the heart. Mural thrombus is not a known complication.
355
Q

A 34-year-old male has serum sodium of
114 mEq/L. Correction of hyponatremia can be
done by raising serum sodium by what amount?

(A) 1 mEq/L/h

(B) 3 mEq/L/h

(C) 5 mEq/L/h

(D) 7 mEq/L/h

(E) 10 mEq/L/h

A
  1. (A) Rapid correction of hyponatremia >1–2 mEq/L/h can lead to central pontine myeli- nolysis. Serum sodium level should not be raised >25 mEq/L within 48 hours of starting therapy. Only symptomatic hyponatremia requires treatment with hypertonic saline, oth- erwise fluid restriction is sufficient.
356
Q

A 47-year-old woman with chronic renal failure has been maintained on chronic dialysis for several years. She had undergone kidney transplantation but because of rejection, she was placed back on dialysis. She had repeated bouts of pain in the RUQ and was intolerant to fatty meals. Ultrasound showed cholelithiasis.

Following elective cholecystectomy, severe
bleeding occurred. This was most likely attributed to which of the following?

(A) Elevated PT

(B) Elevated PTT

(C) Low platelet count

(D) Decreased platelet aggregation

(E) Sepsis

A
  1. (D) Abnormal hemostasis, common in chronic renal failure, is characterized by prolonga- tion of bleeding time, decreased activity of platelet factor 3, abnormal platelet aggrega- tion, and adhesiveness. The prolonged bleeding time is related to failure of platelet interaction with von Willibrand’s factor (factor 8-VWF)
    This interaction can be corrected by using desmopressin or by transfusing cryoprecipitate.
357
Q

A 47-year-old woman with chronic renal failure has been maintained on chronic dialysis for several years. She had undergone kidney transplantation but because of rejection, she was placed back on dialysis. She had repeated bouts of pain in the RUQ and was intolerant to fatty meals. Ultrasound showed cholelithiasis.

The most appropriate management of this
patient is the administration of which of the following?

(A) Heparin

(B) Protamine sulfate

(C) Fresh-frozen plasma

(D) Desmopressin

(E) Factor VIII concentrate

A
  1. (D) The coagulation changes can be reversed with desmopressin or cryoprecipitate.
358
Q

A 70-year-old man, who weighs 70 kg, is admitted with acute cholecystitis. His calculated daily fluid requirement for maintenance is
approximately which of the following?

(A) 1 L

(B) 2 L

(C) 2.5 L

(D) 3 L

(E) 4 L

A
  1. (C) Daily maintenance fluid requirements are calculated on the basis of 100 mL/kg for the first 10 kg of body weight, 50 mL/kg for the second 10 kg of body weight, and 20 mL/kg for each additional kg of body weight (i.e., 100 × 10 + 50 × 10 + 20 × 50 = 2500 mL). Hourly fluid requirement can be calculated using the 4, 2, 1 rule as follows: 4 mL/kg, for the first 10 kg, 2 mL/kg for second 10 kg, and 1 mL/kg for each additional kg of body weight (i.e., 4 × 10 + 2 × 10 + 1 ( 50 = 110 mL/h).
359
Q

A 90-year-old woman with a fractured neck
of femur is receiving low-molecular-weight
heparin (LMWH). Which of the following statements regarding LMWH is true?

(A) It has molecular weight below 4000 d.

(B) Its anticoagulant effect is by binding to
antithrombin III.

(C) It should be administered two to three times a day

(D) It has lower bioavailability than standard heparin.

(E) It has a greater rate of heparin-associated
thrombocytopenia.

A
  1. (B) Low molecular weight heparins (LMWH) are fragments of unfractionated standard heparin with mean molecular weights between 4000 and 64,000 d. They bind to and accelerate the activity of antithrombin III. LMWH has greater bioavailability, more effective antico- agulant effect, lower incidence of heparin- associated thrombocytopenia, and can be administered once daily.
360
Q

DIRECTIONS (Questions 94 through 100): Each set
of matching questions in this section consists of a list of lettered options followed by several numbered items. For each numbered item, select the appropriate lettered option(s). Each lettered option may be selected once, more than once, or not at all.

(A) Copper deficiency
(B) Chromium deficiency
(C) Zinc deficiency
(D) Manganese deficiency
(E) Vitamin A deficiency
(F) Vitamin D deficiency
(G) Vitamin E deficiency
(H) Vitamin K deficiency
(I) Vitamin C deficiency

A 45-year-old man receiving TPN has signs of retarded wound healing. SELECT ONLY THREE.

A
  1. (C, E, I) Zinc deficiency, vitamin A deficiency, and vitamin C deficiency. Zinc is a metalloen- zyme involved in protein and nucleic acid metabolism. Deficiency results in diminished wound strength and healing rates. Vitamin A deficiency results in delayed wound healing, specifically epithelization. Vitamin C deficiency results in defective sulfonated mucopolysac- charides and chondroitin sulfate with retarded wound healing.
361
Q

DIRECTIONS (Questions 94 through 100): Each set
of matching questions in this section consists of a list of lettered options followed by several numbered items. For each numbered item, select the appropriate lettered option(s). Each lettered option may be selected once, more than once, or not at all.

(A) Copper deficiency
(B) Chromium deficiency
(C) Zinc deficiency
(D) Manganese deficiency
(E) Vitamin A deficiency
(F) Vitamin D deficiency
(G) Vitamin E deficiency
(H) Vitamin K deficiency
(I) Vitamin C deficiency
A
  1. (B, D) Chromium is an insulin cofactor. Defi- ciency state results in hyperglycemia. Manganese is a cofactor of enzyme of energy and protein metabolism and also of fat synthesis. Besides causing glucose intolerance, manganese defi- ciency also causes hypocholesterolemia.
362
Q

DIRECTIONS (Questions 94 through 100): Each set of matching questions in this section consists of a list of lettered options followed by several numbered items. For each numbered item, select the appropriate lettered option(s). Each lettered option may be selected once, more than once, or not at all.

(A) Copper deficiency
(B) Chromium deficiency
(C) Zinc deficiency
(D) Manganese deficiency
(E) Vitamin A deficiency
(F) Vitamin D deficiency
(G) Vitamin E deficiency
(H) Vitamin K deficiency
(I) Vitamin C deficiency

A 42-year-old man with small-bowel fistula has
been receiving TPN with standard hypertonic
glucose-amino acid solution for the previous 3 weeks. The patient is noticed to have scaly,
hyperpigmented lesions over the acral surfaces
of elbows and knees, similar to enterohepatic
acrodermatitis. What is the most likely cause of
this condition? SELECT ONE.

A
  1. (C) Zinc is one of the metalloenzymes involved in lipid, carbohydrate, protein, and nucleic acid metabolism. Skin lesions similar to enterohepatic acrodermatitis are the most common signs seen in zinc deficiency. Other manifestations include hypogonadism, diminished wound healing, and immunodeficiencies. Copper deficiency is characterized by microcytic hypochromic anemia.
363
Q

DIRECTIONS (Questions 94 through 100): Each set of matching questions in this section consists of a
list of lettered options followed by several numbered items. For each numbered item, select the appropriate lettered option(s). Each lettered option may be selected once, more than once, or not at all.

(A) Factor II (prothrombin)
(B) Factor V
(C) Factor VII
(D) Factor VIII
(E) Factor IX
(F) Factor X
(G) Factor XII
(H) Calcium
(I) Fibrin split products

A 72-year-old man requires blood transfusion.
He was initially given stored plasma. He is
most likely to show a deficiency of what?
SELECT TWO.

A
  1. (B, D) Factor V and VIII are deficient in stored plasma. In contrast, fresh-frozen plasma con- tains all the coagulation factors. The major dis- advantage of plasma administration, however, is the risk of hepatitis.
364
Q

DIRECTIONS (Questions 94 through 100): Each set
of matching questions in this section consists of a
list of lettered options followed by several numbered items. For each numbered item, select the appropriate lettered option(s). Each lettered option may be selected once, more than once, or not at all.

(A) Factor II (prothrombin)
(B) Factor V
(C) Factor VII
(D) Factor VIII
(E) Factor IX
(F) Factor X
(G) Factor XII
(H) Calcium
(I) Fibrin split products

What is the coagulation factor involved exclusively in the extrinsic coagulation system?
SELECT ONE.

A
  1. (C) There are two coagulation pathways— extrinsic and intrinsic. In the extrinsic system, tissue thromboplastin (a lipoprotein) interacts with factor VII. The intrinsic pathway requires factors XII, XI, IX, and VIII. Factor XII is the ini- tial step in the coagulation cascade. Factor XII, activated by contact with a nonendothelial sub- stance, will activate factor XI (plasma throm- boplastin antecedent). However, factor XI can be activated even when factor XII is deficient. Calcium is required for nearly all of the enzyme reactions in both the intrinsic and extrinsic sys- tems. The amount of ionized calcium required for these reactions is extremely small, and clin- ical hypocalcemia itself is not a cause of abnor- mal bleeding. Fibrin split products are not part of the normal pathway in either the intrinsic or extrinsic system. The excessive breakdown of fibrinogen results in measurable amounts of the breakdown products of fibrinogen in the blood. Their presence may signal DIC if the PT and platelet count are deranged. In pure fibri- nolysis, fibrinogen breakdown product levels also may be increased.
365
Q

DIRECTIONS (Questions 94 through 100): Each set
of matching questions in this section consists of a
list of lettered options followed by several numbered items. For each numbered item, select the appropriate lettered option(s). Each lettered option may be selected once, more than once, or not at all.

(A) Factor II (prothrombin)
(B) Factor V
(C) Factor VII
(D) Factor VIII
(E) Factor IX
(F) Factor X
(G) Factor XII
(H) Calcium
(I) Fibrin split products

A 48-year-old man with severe liver cirrhosis is
admitted to the hospital with hematemesis.
What coagulation factors are not synthesized in
the liver? SELECT TWO.

A
  1. (D, H) All the coagulation factors except throm- boplastin, calcium, and factor VIII are synthe- sized in the liver. Factors II, VII, IX, and X are vitamin K dependent.
366
Q

A 20-year-old man has undergone appendectomy
for perforated appendicitis with generalized peritonitis. Seven days postoperatively, his temperature
continues to spike to 103.8°F despite antibiotic therapy with ampicillin, gentamicin, and metronidazole.
A CT scan reveals a large pelvic abscess. Soon afterward, he has bleeding from the mouth and nose with
increasing oozing from the surgical wound and all intravenous puncture sites. What is the most likely diagnosis?

(A) Anaphylactoid reaction to intravenous
dye

(B) Disseminated intravascular coagulation
(DIC)

(C) Antibiotic-induced coagulopathy

(D) Liver failure

(E) Congenital bleeding disorder

A
  1. (B) Disseminated intravascular coagulation is characterized by diffuse intravascular coagu- lation, thrombosis, and fibrinolysis. It results in thrombocytopenia, hypofibrinogenemia, pro- longation of PT and PTT, and increased con- centration of fibrin degradation products in plasma. Sepsis is a major factor that can trigger DIC.
367
Q

During the treatment of septic shock, a 28-yearold male remains hypotensive despite adequate
volume replacement; PA occlusion pressure is 18 mm Hg. When dopamine is started , ventricular tachycardia develops and this is unresponsive to lidocaine. The V-tach converts back
to sinus rhythm once the dopamine is stopped.

At this point, which of the following treatments
are most appropriate for this hypotensive patient?

(A) Amrinone

(B) Dobutamine

(C) Epinephrine

(D) Phenylephrine

(E) Intra-aortic balloon pump

A
  1. (D) Dopamine activates b -receptors and this 1
    was probably the reason for the arrhythmia. Amrinone will inhibit phosphodiesterase and result in an increased cyclic AMP level, pro- ducing the same result as b-receptor stimu- lation. Dobutamine and epinephrine also stimulate the b- receptors. The only choice which stimulates only a-adrenergic receptors is phenylephrine. Intra-aortic balloon pump is invasive¯, therefore, less appropriate as a choice.
368
Q

A 65-year-old male is resuscitated using hydroxyethyl starch (hetastarch). Which of the following is associated with the use of hetastarch ?

(A) Thrombotic thrombocytopenia

(B) Elevated levels of factor VIII

(C) Elevation of serum creatinine

(D) Hyperbilirubinemia

(E) Hyperamylasemia

A
  1. (E) Hetastarch is a synthetic colloid that is metabolically inert and can be infused IV. A 6% solution of hetastarch has the same osmotic properties as 5% albumin. Relatively few com- plications are associated with hetastarch. Large volumes do cause dilution of plasma proteins as well as coagulation and platelet function disorders. Hetastarch binds to amylase and impairs renal excretion of amylase causing hyperamylasemia.
369
Q

A 28-year-old female several minutes after
receiving an intravenous dose of ampicillin for
dental prophylaxis against endocarditis develops diffuse pruritis, cutaneous erythema, and
hypotension (BP = 60/40 mm Hg). All of the
following hemodynamic parameters are typical
of this type of shock initially, except

(A) Increased HR

(B) Intravascular hypovolemia

(C) Vasodilation

(D) Increased cardiac output

(E) Decreased preload

A
  1. (D) In anaphylactic shock, which this patient is showing, the HR would reflexively increase due to the drop in BP. The intravascular hypovolemia and decreased preload are clearly present along with the vasodilation; however, the cardiac output is initially decreased not increased.
370
Q

A 55-year-old male presents to the emergency
room (ER) with a history suggestive of myocardial infarction, but without a diagnostic ECG
pattern of ST-segment elevation. Which of the
following ECG patterns strongly suggests that
thrombolytic therapy should be administered.

(A) Right bundle branch block

(B) Left bundle branch block

(C) Second-degree AV block (Wenckebach
type)

(D) Complete artrioventricular (AV) block

(E) Runs of V tachycardia

A
  1. (B) The only ECG rhythm, which can obscure ST-segment changes seen in acute myocardial infarction, is left bundle branch block. All the other rhythms would allow visualization of the ST-segment changes.
371
Q

You are called to the emergency department to
evaluate a 55-year-old woman following motor
vehicle crash with associated head trauma. She withdraws to pain and is intubated for airway
protection. In order to calculate the Glasgow
Coma Scale score, which of the following components of the neurologic examination are necessary?

(A) Motor response, verbal response, corneal reflexes

(B) Motor response, eye opening, verbal response

(C) Eye opening, pupillary light reflexes, motor response

(D) Pupillary light reflexes, motor response,
verbal response

(E) Corneal reflexes, pupillary light reflexes,
motor response

A
  1. (B) The Glasgow Coma Score scale is made up of eye opening, verbal response, and motor response.
372
Q

A40-year-old woman is given a routine injection
of ragweed allergen immunotherapy by her
family physician. She developed a shortness of
breath and a sensation of throat swelling. She was
taken to the emergency department where she
was noted to be flushed and sweating profusely
and in moderate distress. She was also noted to
be wheezing, tachycardic and hypotensive.
Which of the following interventions is most
appropriate at this time?

(A) Ranitidine 50 mg PO

(B) Diphenhydramine 50 mg PO

(C) Ringer’s lactate, 250 ml over 1 hour

(D) Methylprednisolone 125 mg PO

(E) Epinephrine 0.5 mL intramuscular (IM)

A
  1. (E) The patient is suffering from anaphylaxis, and the treatment of choice is epinephrine. Epinephrine IM has been shown to be more effective than SC for the treatment of anaphy- laxis.
373
Q

A 67-year-old man with severe ARDS is receiving pressure assisted control ventilation. He is
requiring an FiO2 of 100% to maintain the following blood gas levels: pH = 7.26, PCO2 = 60,
PO2= 58. You decide to put the patient in prone
position. Fifteen minutes later, on the same vent
settings, the patient’s tidal volume is now decreased and his blood gas values are pH = 7.09,
PCO2 = 76, PO2 = 89. He is hemodynamically
unchanged and his chest x-ray (CXR) is also
unchanged. The most likely cause of his worsening respiratory acidosis in the prone position is

(A) Pneumothorax

(B) Increased dead space

(C) Decreased cardiac filling

(D) Reduced chest wall compliance

(E) Pulmonary edema

A
  1. (D) The prone positioning reduces the dispar- ity in mechanics between the dependent and nondependent regions of the lungs. This reduces the collapsing of the alveoli in the dependent portions of the lungs and overdis- tention in the nondependent portions of the lungs. The prone position also has other effects—it allows a more normal curvature of the diaphragm and allows better function. It also stiffens the chest wall allowing a more even distribution of ventilation and reduction in overventilation of nondependent alveoli.
374
Q

You are asked to see a 70-year-old male admitted
to the ICU with anterior chest pain radiating to
the back described as “a tearing sensation.” The
pain reached maximum intensity within 30 minutes. The patient has a history of hypertension
(noncompliant with medications). His BP in the ICU is 170/110 mm Hg, HR = 110/min. Physical
examination reveals a 2/6 diastolic murmur and
unequal femoral pulses. A CXR of this patient
was normal and the CT chest, which was
obtained, is shown in Fig. 1–2. Which of the following statements regarding his treatment and prognosis are correct?

(A) The patient will require nitroprusside and beta blockade, but will not require surgical intervention.

(B) The patient will require only nitroprusside but will not require surgical intervention.

(C) The patient will require nitroprusside and
b-blockade prior to emergency surgical
intervention.

(D) Neither nitroprusside nor b-blockade is
required prior to surgical intervention.

(E) Nitroprusside and b-blockade are required initially, but surgery may be done electively within 4–6 weeks.

A
  1. (C) The patient has a diagnosis of dissecting aortic aneurysm. This requires emergent med- ical as well as surgical intervention. The BP needs to be lowered by using nitroprusside as well as a b-blocker to reduce dp/dt (the force with which the heart is pumping). Once BP is controlled the surgical intervention is needed to correct the problem.
375
Q

ATLS 10th Ed Guidelines on Shock?

A

BALANCED RESUSCITATION

  • Initial bolus of 1L may be required.
  • Fluids are administered judiciously, as aggressive resuscitation before control of bleeding has been demonstrated to increase mortality.

MINIMAL RESUSCITATION
- Minimum 18gauge peripheral access x 2.

PICC
- Choice of site for alternate access based on clinician experience and skill.

EARLY BT
- Early resuscitation with blood and blood products must be considered in patients with evidence of class III and IV hemorrhage.

MTP

  • Coagulopathy associated with severe trauma can be fueled by resuscitative measures.
  • Use of massive transfusion protocols with blood components administered at predefined low ratios may mitigate this.
  • 1:1:1 PRBC:Plasma:Plt

EARLY TXA

  • Administer tranexamic acid in prehospital setting to severely injured patients within 3h of injury.
  • The first dose (1g) is usually given over 10 minutes and is administered in the field; the follow up dose of 1g is given over 8h.
376
Q

Hemodynamic parameters in shock?

A

1) HYPOVOLEMIC
- Decreased MAP, CO, PAOP
- Increased SVR

2) CARDIOGENIC
- Decreased MAP, CO
- Increased PAOP, SVR

3) OBSTRUCTIVE
(Tension pneumothorax, ACS, PE, Pericardial tamponade, auto-PEEP, severe aortic stenosis, dissecting AA, severe aortic coarctation)
- Decreased MAP, CO
- Unchanged/increased PAOP
- Increased SVR
4) DISTRIBUTIVE
(Sepsis)
- Decreased SVR
- Unchanged/increased CO
- Unchanged/decreased PAOP

Hypotension leads to:

1) Low CO
- > a. Low stroke volume
- –> i. Decreased venous return
- –> ii. Ventricular dysfunction

  • > b. Low heart rate
    2) Low vascular tone
377
Q

Necessary MAP to maintain end organ perfusion in most cases?

A

≥60mmHg

Higher MAP may be necessary in select cases:

  • Intracranial HTN (hemorrhage, cerebral edema, closed head trauma, large strokes)
  • Chronic HTN

MAP
= (CO x SVR)
= ((HR x SV) x SVR)

378
Q

Differentiate multiple organ dysfunction versus failure.

A

DYSFUNCTION
The organ is incapable of maintaining homeostasis.

  • GI: Ileus, intolerance of enteral feeds
  • HPB: High serum bilirubin, liver function tests that are twice normal values

FAILURE
The organ cannot meet minimal demands, and is not considered viable.

  • GI: Stress ulcers, acalculous cholecystitis
  • HPB: Clinical jaundice, TB >8-10mg/dL